Vous êtes sur la page 1sur 194

Ph.D.

Qualifying Examination
c _2001-2010 ODT

U Physics Department
Last updated: May 29, 2012
Contents
1 Nov 2011 1
1.1 ANALYTICAL MECHANICS . . . . . . . . . . . . . . . . . . . . . . . . . . . . . . . . . . 2
1.2 ELECTROMAGNETIC THEORY . . . . . . . . . . . . . . . . . . . . . . . . . . . . . . . 4
1.3 METHODS OF MATHEMATICAL METHODS . . . . . . . . . . . . . . . . . . . . . . . 6
1.4 QUANTUM MECHANICS . . . . . . . . . . . . . . . . . . . . . . . . . . . . . . . . . . . 7
1.5 STATISTICAL MECHANICS . . . . . . . . . . . . . . . . . . . . . . . . . . . . . . . . . . 9
1.6 MISCELLANEOUS PHYSICS PROBLEMS . . . . . . . . . . . . . . . . . . . . . . . . . . 10
2 May 2011 12
2.1 ANALYTICAL MECHANICS . . . . . . . . . . . . . . . . . . . . . . . . . . . . . . . . . . 13
2.2 ELECTROMAGNETIC THEORY . . . . . . . . . . . . . . . . . . . . . . . . . . . . . . . 14
2.3 METHODS OF MATHEMATICAL PHYSICS . . . . . . . . . . . . . . . . . . . . . . . . 15
2.4 QUANTUM MECHANICS . . . . . . . . . . . . . . . . . . . . . . . . . . . . . . . . . . . 16
2.5 STATISTICAL MECHANICS . . . . . . . . . . . . . . . . . . . . . . . . . . . . . . . . . . 18
2.6 MISCELLANEOUS PHYSICS PROBLEMS . . . . . . . . . . . . . . . . . . . . . . . . . . 19
3 Nov 2010 22
3.1 ANALYTICAL MECHANICS . . . . . . . . . . . . . . . . . . . . . . . . . . . . . . . . . . 23
3.2 ELECTROMAGNETIC THEORY . . . . . . . . . . . . . . . . . . . . . . . . . . . . . . . 24
3.3 METHODS OF MATHEMATICAL PHYSICS . . . . . . . . . . . . . . . . . . . . . . . . 26
3.4 QUANTUM MECHANICS . . . . . . . . . . . . . . . . . . . . . . . . . . . . . . . . . . . 28
3.5 STATISTICAL MECHANICS . . . . . . . . . . . . . . . . . . . . . . . . . . . . . . . . . . 30
3.6 MISCELLANEOUS PHYSICS PROBLEMS . . . . . . . . . . . . . . . . . . . . . . . . . . 31
4 May 2010 34
4.1 ANALYTICAL MECHANICS . . . . . . . . . . . . . . . . . . . . . . . . . . . . . . . . . . 35
4.2 ELECTROMAGNETIC THEORY . . . . . . . . . . . . . . . . . . . . . . . . . . . . . . . 37
4.3 METHODS OF MATHEMATICAL PHYSICS . . . . . . . . . . . . . . . . . . . . . . . . 38
4.4 QUANTUM MECHANICS . . . . . . . . . . . . . . . . . . . . . . . . . . . . . . . . . . . 39
4.5 STATISTICAL MECHANICS . . . . . . . . . . . . . . . . . . . . . . . . . . . . . . . . . . 41
4.6 MISCELLANEOUS PHYSICS PROBLEMS . . . . . . . . . . . . . . . . . . . . . . . . . . 42
5 Nov 2009 45
5.1 ANALYTICAL MECHANICS . . . . . . . . . . . . . . . . . . . . . . . . . . . . . . . . . . 46
5.2 ELECTROMAGNETIC THEORY . . . . . . . . . . . . . . . . . . . . . . . . . . . . . . . 47
5.3 METHODS OF MATHEMATICAL PHYSICS . . . . . . . . . . . . . . . . . . . . . . . . 48
5.4 QUANTUM MECHANICS . . . . . . . . . . . . . . . . . . . . . . . . . . . . . . . . . . . 49
5.5 STATISTICAL MECHANICS . . . . . . . . . . . . . . . . . . . . . . . . . . . . . . . . . . 51
5.6 MISCELLANEOUS PHYSICS PROBLEMS . . . . . . . . . . . . . . . . . . . . . . . . . . 52
6 May 2009 55
6.1 ANALYTICAL MECHANICS . . . . . . . . . . . . . . . . . . . . . . . . . . . . . . . . . . 56
6.2 ELECTROMAGNETIC THEORY . . . . . . . . . . . . . . . . . . . . . . . . . . . . . . . 58
6.3 METHODS OF MATHEMATICAL PHYSICS . . . . . . . . . . . . . . . . . . . . . . . . 60
6.4 QUANTUM MECHANICS . . . . . . . . . . . . . . . . . . . . . . . . . . . . . . . . . . . 62
c ODT

U Physics Department. (CONTENTS) i


6.5 STATISTICAL MECHANICS . . . . . . . . . . . . . . . . . . . . . . . . . . . . . . . . . . 63
7 Nov 2008 64
7.1 ANALYTICAL MECHANICS . . . . . . . . . . . . . . . . . . . . . . . . . . . . . . . . . . 65
7.2 ELECTROMAGNETIC THEORY . . . . . . . . . . . . . . . . . . . . . . . . . . . . . . . 67
7.3 METHODS OF MATHEMATICAL PHYSICS . . . . . . . . . . . . . . . . . . . . . . . . 69
7.4 QUANTUM MECHANICS . . . . . . . . . . . . . . . . . . . . . . . . . . . . . . . . . . . 71
7.5 STATISTICAL MECHANICS . . . . . . . . . . . . . . . . . . . . . . . . . . . . . . . . . . 73
8 May 2008 74
8.1 ANALYTICAL MECHANICS . . . . . . . . . . . . . . . . . . . . . . . . . . . . . . . . . . 75
8.2 ELECTROMAGNETIC THEORY . . . . . . . . . . . . . . . . . . . . . . . . . . . . . . . 76
8.3 METHODS OF MATHEMATICAL PHYSICS . . . . . . . . . . . . . . . . . . . . . . . . 77
8.4 QUANTUM MECHANICS . . . . . . . . . . . . . . . . . . . . . . . . . . . . . . . . . . . 78
8.5 STATISTICAL MECHANICS . . . . . . . . . . . . . . . . . . . . . . . . . . . . . . . . . . 80
9 Nov 2007 82
9.1 ANALYTICAL MECHANICS . . . . . . . . . . . . . . . . . . . . . . . . . . . . . . . . . . 83
9.2 ELECTROMAGNETIC THEORY . . . . . . . . . . . . . . . . . . . . . . . . . . . . . . . 85
9.3 METHODS OF MATHEMATICAL PHYSICS . . . . . . . . . . . . . . . . . . . . . . . . 88
9.4 QUANTUM MECHANICS . . . . . . . . . . . . . . . . . . . . . . . . . . . . . . . . . . . 89
9.5 STATISTICAL MECHANICS . . . . . . . . . . . . . . . . . . . . . . . . . . . . . . . . . . 92
10 May 2007 93
10.1 ANALYTICAL MECHANICS . . . . . . . . . . . . . . . . . . . . . . . . . . . . . . . . . . 94
10.2 ELECTROMAGNETIC THEORY . . . . . . . . . . . . . . . . . . . . . . . . . . . . . . . 95
10.3 METHODS OF MATHEMATICAL PHYSICS . . . . . . . . . . . . . . . . . . . . . . . . 97
10.4 QUANTUM MECHANICS . . . . . . . . . . . . . . . . . . . . . . . . . . . . . . . . . . . 98
10.5 STATISTICAL MECHANICS . . . . . . . . . . . . . . . . . . . . . . . . . . . . . . . . . . 99
11 Nov 2006 100
11.1 ANALYTICAL MECHANICS . . . . . . . . . . . . . . . . . . . . . . . . . . . . . . . . . . 101
11.2 ELECTROMAGNETIC THEORY . . . . . . . . . . . . . . . . . . . . . . . . . . . . . . . 102
11.3 METHODS OF MATHEMATICAL PHYSICS . . . . . . . . . . . . . . . . . . . . . . . . 104
11.4 QUANTUM MECHANICS . . . . . . . . . . . . . . . . . . . . . . . . . . . . . . . . . . . 105
11.5 STATISTICAL MECHANICS . . . . . . . . . . . . . . . . . . . . . . . . . . . . . . . . . . 107
12 May 2006 108
12.1 ANALYTICAL MECHANICS . . . . . . . . . . . . . . . . . . . . . . . . . . . . . . . . . . 109
12.2 ELECTROMAGNETIC THEORY . . . . . . . . . . . . . . . . . . . . . . . . . . . . . . . 110
12.3 METHODS OF MATHEMATICAL PHYSICS . . . . . . . . . . . . . . . . . . . . . . . . 112
12.4 QUANTUM MECHANICS . . . . . . . . . . . . . . . . . . . . . . . . . . . . . . . . . . . 113
12.5 STATISTICAL MECHANICS . . . . . . . . . . . . . . . . . . . . . . . . . . . . . . . . . . 115
13 Nov 2005 116
13.1 ANALYTICAL MECHANICS . . . . . . . . . . . . . . . . . . . . . . . . . . . . . . . . . . 117
13.2 ELECTROMAGNETIC THEORY . . . . . . . . . . . . . . . . . . . . . . . . . . . . . . . 119
13.3 METHODS OF MATHEMATICAL PHYSICS . . . . . . . . . . . . . . . . . . . . . . . . 121
13.4 QUANTUM MECHANICS . . . . . . . . . . . . . . . . . . . . . . . . . . . . . . . . . . . 123
13.5 STATISTICAL MECHANICS . . . . . . . . . . . . . . . . . . . . . . . . . . . . . . . . . . 124
c ODT

U Physics Department. (CONTENTS) ii


14 May 2005 125
14.1 ANALYTICAL MECHANICS . . . . . . . . . . . . . . . . . . . . . . . . . . . . . . . . . . 126
14.2 ELECTROMAGNETIC THEORY . . . . . . . . . . . . . . . . . . . . . . . . . . . . . . . 128
14.3 METHODS OF MATHEMATICAL PHYSICS . . . . . . . . . . . . . . . . . . . . . . . . 130
14.4 QUANTUM MECHANICS . . . . . . . . . . . . . . . . . . . . . . . . . . . . . . . . . . . 131
14.5 STATISTICAL MECHANICS . . . . . . . . . . . . . . . . . . . . . . . . . . . . . . . . . . 133
15 Nov 2004 134
15.1 ANALYTICAL MECHANICS . . . . . . . . . . . . . . . . . . . . . . . . . . . . . . . . . . 135
15.2 ELECTROMAGNETIC THEORY . . . . . . . . . . . . . . . . . . . . . . . . . . . . . . . 137
15.3 METHODS OF MATHEMATICAL PHYSICS . . . . . . . . . . . . . . . . . . . . . . . . 138
15.4 QUANTUM MECHANICS . . . . . . . . . . . . . . . . . . . . . . . . . . . . . . . . . . . 139
15.5 STATISTICAL MECHANICS . . . . . . . . . . . . . . . . . . . . . . . . . . . . . . . . . . 140
16 May 2004 141
16.1 ANALYTICAL MECHANICS . . . . . . . . . . . . . . . . . . . . . . . . . . . . . . . . . . 142
16.2 ELECTROMAGNETIC THEORY . . . . . . . . . . . . . . . . . . . . . . . . . . . . . . . 143
16.3 METHODS OF MATHEMATICAL PHYSICS . . . . . . . . . . . . . . . . . . . . . . . . 144
16.4 QUANTUM MECHANICS . . . . . . . . . . . . . . . . . . . . . . . . . . . . . . . . . . . 146
16.5 STATISTICAL MECHANICS . . . . . . . . . . . . . . . . . . . . . . . . . . . . . . . . . . 147
17 Dec 2003 148
17.1 ANALYTICAL MECHANICS . . . . . . . . . . . . . . . . . . . . . . . . . . . . . . . . . . 149
17.2 ELECTROMAGNETIC THEORY . . . . . . . . . . . . . . . . . . . . . . . . . . . . . . . 151
17.3 METHODS OF MATHEMATICAL PHYSICS . . . . . . . . . . . . . . . . . . . . . . . . 152
17.4 QUANTUM MECHANICS . . . . . . . . . . . . . . . . . . . . . . . . . . . . . . . . . . . 153
17.5 STATISTICAL MECHANICS . . . . . . . . . . . . . . . . . . . . . . . . . . . . . . . . . . 155
18 May 2003 156
18.1 ANALYTICAL MECHANICS . . . . . . . . . . . . . . . . . . . . . . . . . . . . . . . . . . 157
18.2 ELECTROMAGNETIC THEORY . . . . . . . . . . . . . . . . . . . . . . . . . . . . . . . 158
18.3 METHODS OF MATHEMATICAL PHYSICS . . . . . . . . . . . . . . . . . . . . . . . . 159
18.4 QUANTUM MECHANICS . . . . . . . . . . . . . . . . . . . . . . . . . . . . . . . . . . . 160
18.5 STATISTICAL MECHANICS . . . . . . . . . . . . . . . . . . . . . . . . . . . . . . . . . . 161
19 Nov 2002 162
19.1 ANALYTICAL MECHANICS . . . . . . . . . . . . . . . . . . . . . . . . . . . . . . . . . . 163
19.2 ELECTROMAGNETIC THEORY . . . . . . . . . . . . . . . . . . . . . . . . . . . . . . . 164
19.3 METHODS OF MATHEMATICAL PHYSICS . . . . . . . . . . . . . . . . . . . . . . . . 165
19.4 QUANTUM MECHANICS . . . . . . . . . . . . . . . . . . . . . . . . . . . . . . . . . . . 166
19.5 STATISTICAL MECHANICS . . . . . . . . . . . . . . . . . . . . . . . . . . . . . . . . . . 168
20 May 2002 169
20.1 ANALYTICAL MECHANICS . . . . . . . . . . . . . . . . . . . . . . . . . . . . . . . . . . 170
20.2 ELECTROMAGNETIC THEORY . . . . . . . . . . . . . . . . . . . . . . . . . . . . . . . 171
20.3 METHODS OF MATHEMATICAL PHYSICS . . . . . . . . . . . . . . . . . . . . . . . . 172
20.4 QUANTUM MECHANICS . . . . . . . . . . . . . . . . . . . . . . . . . . . . . . . . . . . 173
20.5 STATISTICAL MECHANICS . . . . . . . . . . . . . . . . . . . . . . . . . . . . . . . . . . 174
c ODT

U Physics Department. (CONTENTS) iii


21 Nov 2001 175
21.1 ANALYTICAL MECHANICS . . . . . . . . . . . . . . . . . . . . . . . . . . . . . . . . . . 176
21.2 ELECTROMAGNETIC THEORY . . . . . . . . . . . . . . . . . . . . . . . . . . . . . . . 177
21.3 METHODS OF MATHEMATICAL PHYSICS . . . . . . . . . . . . . . . . . . . . . . . . 178
21.4 QUANTUM MECHANICS . . . . . . . . . . . . . . . . . . . . . . . . . . . . . . . . . . . 179
21.5 STATISTICAL MECHANICS . . . . . . . . . . . . . . . . . . . . . . . . . . . . . . . . . . 180
22 May 2001 181
22.1 ANALYTICAL MECHANICS . . . . . . . . . . . . . . . . . . . . . . . . . . . . . . . . . . 182
22.2 ELECTROMAGNETIC THEORY . . . . . . . . . . . . . . . . . . . . . . . . . . . . . . . 183
22.3 METHODS OF MATHEMATICAL PHYSICS . . . . . . . . . . . . . . . . . . . . . . . . 185
22.4 QUANTUM MECHANICS . . . . . . . . . . . . . . . . . . . . . . . . . . . . . . . . . . . 187
22.5 STATISTICAL MECHANICS . . . . . . . . . . . . . . . . . . . . . . . . . . . . . . . . . . 189
Nov 2011
Nov 2011
ANALYTICAL MECHANICS
1. Consider a disk with mass m and the mass is distributed so that one of the principle axes of inertia
is parallel to the axis of the disk and at a distance a from it. (The center of mass CM is at a
distance a from the geometric center GC and the moment of inertia about the principal axis is I.)
Assume that the disk is free to roll without slipping.
(a) Construct the Lagrangian for small oscillations.
(b) Using the Lagrange equation, nd the angular frequency for small oscillations.
(c) If the CM coincides with the GC of the disk, namely a0, what will be the angular frequency?
Is it an expected result? Why?
2. Consider a small ring of mass m that is free to move on a horizontal rod without friction. Another
mass m is attached to this rod with a massless string of length L. Using the generalized coordinates
x and shown in the gure above, answer the following questions.
(a) What is the Lagrangian of the system ?
(b) Write down the equations of motion for the generalized coordinates ?
(c) What are the canonical momenta ?
(d) What is the Hamiltonian of this system ?
(e) Write down the Hamiltons equation of motion for this system.
In the remaining questions, assume that the system is doing small oscillations.
c ODT

U Physics Department. (Nov 2011) 2


(g) What are the normal modes of the system? Sketch or describe how the system looks like in
each one of the modes.
(h) Given the initial conditions: x(t = 0) = 0, x(t = 0) = 0, (t = 0) =
0
and

(t = 0) = 0, what
is the expression for (t) and x(t)?
c ODT

U Physics Department. (Nov 2011) 3


Nov 2011
ELECTROMAGNETIC THEORY
1. The electric eld component of a communication satellite signal traveling in free space is given by

E(z, t) = [ x y(1 + i)] 12e


i50(zct)
V/m
(a) Find the corresponding

H(z, t).
(b) Find the time-average power per unit area carried by this wave.
(c) Determine the polarization (both type and sense) of the wave.
2. Two identical point charges with charge +q
0
are held at positions (a, 0, 0) and (2a, 0, 0) near an
innite grounded conducting plane at x = 0.
(a) Find the total electrical force acting on each charge.
(b) Find the electrical potential energy of this conguration.
(c) Find the surface charge density at the point (0,0,0) on
(d) What is the total induced charge on the plane?
3. Consider a high energy particle with Lorentz factor 1, mass M and charge Z which is moving
at constant speed v along the x

-axis as shown in the gure. If this high energy particle passes


without being deected from the close vicinity of a stationary electron with charge e and mass
m
e
M , some fraction of the kinetic energy of the particle is transferred to the electron. Answer
the following questions in terms of the Lorentz factor , speed v, time t and the closest distance b
between the particles with respect to the xed observer.
(a) Find the force on the electron with respect to the rest frame of the high energy particle.
(b) Find the force on the electron with respect to the rest frame of an observer xed in the lab
frame.
(c) Assuming that the impulse on the electron is along the z direction calculate the kinetic energy
transfered to the electron using the force you found in i) part (a) and ii) part (b); compare
your results.
c ODT

U Physics Department. (Nov 2011) 4


Hint: Use following Lorentz transformations for the electric eld and magnetic ux density.
E
x
= E
x

E
y
= (E
y
+ vB
z
)
E
z
= (E
z
vB
y
)
B
x
= B
x

B
y
=
_
B
y

v
c
2
E
z

_
B
z
=
_
B
z
+
v
c
2
E
y

_
c ODT

U Physics Department. (Nov 2011) 5


Nov 2011
METHODS OF MATHEMATICAL METHODS
1. Consider a at circular plate of unit radius which is initially at constant temperature. At time
t = 0, heat starts to escape from the surface of the plate according to the Newtons law of cooling,
so that temperature U(r, t) satises the equation,

2
U =
U
t
+ (U U
0
)
where U
0
is the temperature of the surrounding medium and , are constants.
(a) (15 pts.) When U
0
= 0 and U = R(r)T(t), show that the given partial dierential equation
can be reduced to the ordinary dierential equations,
d
2
R
dr
2
+
1
r
dR
dr
+ R = 0, (i)
dT
dt
+ (
2
+ )T = 0. (ii)
(b) (20 pts.) Setting = k
2
> 0 and kr = , show that the equation (i) can be written as
d
2
R
d
2
+
1

dR
d
+ R = 0.
Find a power series solution for this equation about = 0 and show that it can be written as

n=0
(1)
n
(n!)
2
_

2
_
2n
(c) (15 pts.) What is the solution of equation (ii)? Write down the temperature distribution of
the circular plate at any time t.
Note:

2
U =

2
U
r
2
+
1
r
U
r
+
1
r
2

2
U

2
2. (a) (15 pts.) Find the roots of the equation z
6
+ 1 = 0, and locate them on the complex plane.
(b) (35 pts.) Evaluate the value of the integral using residue calculus:
_

0
dx
x
6
+ 1
3. (a) (25 pts) Find the function y(x) which extremizes the integral
I[y] =
1
2
_
1
0
dx
(y

(x))
2
x
, (1)
given that y(0) = 0, y

(0) = 0, y(1) = 1, y

(1) = 0. Here y

(x) =
dy
dx
etc.
(b) (25 pts) Solve the same problem given above subject to the following constraint
_
1
0
y(x)dx =
1
2
. (2)
c ODT

U Physics Department. (Nov 2011) 6


Nov 2011
QUANTUM MECHANICS
1. (a) (10 pts) Consider a particle in a state described by the wavefunction (x, t) that is a solution
of the Schrodinger equation, i

t
(x, t) =

2
2m

2
x
2
(x, t) +V (x)(x, t). Show that the prob-
ability of nding the particle somewhere is independent of time,
d
dt
_

(x, t)(x, t)dx = 0.


(b) (10 pts) Consider a particle of mass m conned to the interior of a spherical well with impen-
etrable walls,
V (r) =
_
0 r a
r > a
The radial equation in the domain r a for = 0 (s state) has two linearly independent
solutions
sin(kr)
r
and
cos(kr)
r
. Write the physically acceptable solution of the wavefunction for
this particle and explain your results (i.e., which property of the wave function did you use?).
(c) (10 pts) An electron in the Coulomb eld of a proton is in a state described by the normalized
wave function
(r, , ) =
1

11
_

3
21,1
(r)
210
(r) + i

5
211
(r) +

2
311
(r)
_
,
where here,
nm

(r) represents the energy eigenfunctions. What is the probability that a


measurement of

L
z
yields 1?
(d) (10 pts) For a single-particle (spin 1/2 particle), the spin-orbit potential is given by V =
2

2

L

S
where is a constant. Calculate the expectation values of this potential for = 1 states.
(e) (10 pts) Suppose an electron is initially in the following spin state
(t = 0) = N
_
3i
4
_
.
It is put into a uniform magnetic eld that points in the z direction,

B = B
0

k. The Hamil-
tonian of the electron in a magnetic eld is given by H = (e/m
e
c)

S. Find the normalized


spin wave-function (t) at a later time t. (Hint: use the time-dependent Schrodinger equation
for H.)
S
z
=

2
_
1 0
0 1
_
2. Consider an anharmonic oscillator with the Hamiltonian
H =
p
2
2m
+
1
2
m
2
x
2
+ x
3
.
Here, the parameter , which expresses the degree of anharmonicity, is a suciently small parameter
so that the energy levels can be investigated with perturbation theory.
(a) (40 pts) Find the lowest order correction (lowest order in ) to the energies E
n
= E
n
() of
the oscillator.
(b) (10 pts) Consider transitions between two successive levels of the oscillator, n + 1 n.
Compute the (angular) frequency of the emitted photon.
c ODT

U Physics Department. (Nov 2011) 7


Hints:
(1) a =
_
m
2
_
x + i
p
m
_
(2) E
(2)
n
=

k(=n)
[V
kn
[
2
E
(0)
n
E
(0)
k
3. Consider the Dirac equation.
(a) (10 pts) Compute d

L/dt and d

/dt in the Heisenberg picture.


(b) (10 pts) Can you construct a conserved operator involving

L and

?
(c) (10 pts) Construct the stationary state solutions of free Dirac equation in the form =
u(p)e
ipx/
by explicitly determining the normalized u(p).
(d) (10 pts) Using the explicit solution found in part (c), compute d

L/dt) and d

/dt), where
for an operator A, A) is dened as A) =

A.
(e) (10 pts) Compare the results in (a) and (d) and interpret.
c ODT

U Physics Department. (Nov 2011) 8


Nov 2011
STATISTICAL MECHANICS
1. A one-dimensional, non-relativistic ideal Fermi gas is conned to a line of length L. The mass of
each particles is m and the spin of each particle is s.
(a) (20 pts) Find a general relation between ln of the Grand Partition Function and the pressure
P, volume V and temperature T, for the gas.
(b) (15 pts) Calculate ln , in terms of the s, the thermal wavelength
T
and the Fermi function
f

(z) =
1
()
_

0
x
1
dx
z
1
e
x
+ 1
(c) (15 pts) Calculate the pressure P in terms of the temperature T, the chemical potential (or
equivalently and z), and the Fermi function f
3/2
(z)
2. (a) (15 pts) The Solid A with heat capacity C
A
at temperature T
A
is placed in thermal contact
with solid B with heat capacity C
B
at temperature T
B
. For parts (a)-(c), assume that the
volumes of the two systems are xed, the combined system (A + B) is a closed system and
the heat capacities are independent of temperature.
i. (5 pts) What is the equilibrium temperature?
ii. (10 pts) What is the change in entropy of the combined system until thermal equilibrium
is reached?
(b) (12 pts) Suppose now that solid B is much larger than solid A. Under this condition prove
that the change in entropy (evaluated in part (a)) can be approximated as
S = C
A
_
ln
T
B
T
A
+
T
A
T
B
1
_
(c) (10 pts) Use the expression in part (b) to show that the total entropy increases both in the
cooling and the heating processes.
(d) (13 pts) Consider now two systems: system 1 with heat capacity C
1
at temperature T
1
and
system 2 with heat capacity C
2
at temperature T
2
(T
1
,= T
2
). If necessary, neglect the
dierence between the heat capacities at constant volume and at constant pressure. Assume
that the heat capacities are independent of temperature, nite, and are of the same order of
magnitude. What is the common nal temperature given that a cycle is devised to extract
the maximum possible work from the two systems?
c ODT

U Physics Department. (Nov 2011) 9


Nov 2011
MISCELLANEOUS PHYSICS PROBLEMS
1. In this question you are asked describe an experiment that proves the wave nature of visible light.
For this purpose:
(a) (4 pts) Draw a schematic diagram of the experimental set up.
(b) (7 pts) Write the names of the apparatus. Write a typical realistic range for the basic properties
of each apparatus (for example: the plane screen at a distance of at least . . . meters from the
source).
(c) (7 pts) Discuss how the parameters of the experiment aect the experimental results (include
the necessary equations).
(d) (7 pts) Explain how the analysis of the experimental results proves the wave nature of light.
2. (a) (10 pts) Describe the Compton eect by drawing it in a simple diagram (indicate the energy
and momenta of particles).
(b) (15 pts) Using the conservation of momentum and energy, derive the expression

=
h
mc
(1 cos())
which relates the dierence in the wavelength of the interacting photon with the Compton
wavelength of the electron and the scattering direction of the photon .
3. Consider the vibrations of the masses m and M (m<M) in one dimensional diatomic crystal struc-
ture (with period a) by force constant C.
(a) (10 pts) Write down the equations of motions of the atoms for [100] directional traveling waves
by considering only the nearest neighbor interactions.
(b) (10 pts) Show that the light atoms are stationary in the acoustical branch and the heavier
atoms are stationary in the optical branch with the acoustic and optic vibration modes of

A
=
_
2C
M
and
O
=
_
2C
m
respectively, at the Brillouin zone boundary.
(c) (5 pts) Describe the acoustic oscillation types of the atoms around k=0 if

2
A

Ck
2
a
2
2(m + M)
4. The dwarf planet Pluto and its largest moon Charon constitute a dwarf planetary binary system
as they move around the center of mass in almost circular orbits (see the Figure, C.M. denotes the
center of mass of the system).
(a) (5 pts) Show that
m
P
m
C
=
R
C
R
P
(Hint: consider center of mass position)
(b) (10 pts) Show that T
P
= T
C
(period of Pluto equals period of Charon) utilizing the relation
derived in part (a). (Hint: assume uniform circular motion)
(c) (10 pts) Calculate the binary period T of the Pluto-Charon system in terms of M
C
M
P
R
C
R
P
(and other expected constants).
c ODT

U Physics Department. (Nov 2011) 10


5. Assume you have n and p-type semiconductors.
(a) (10 pts) What are the dierences of these semiconductors from each other and metal (or what
make them dierent from each other and metal)? Write and express their conductivities,
carrier concentrations, mobilities and also plot the energy band diagram by showing each
energy level.
(b) (5 pts) When you make n-p junction, explain ow of charges and energy band diagram of this
system in details.
(c) (10 pts) Assume you have p-n (or n-p) junction produced by using these semiconductors with
dierent band gap values (one has 1.24 eV and the other one 2.48 eV) are placed on top
of each other and you are making photoresponse measurement which is the measurement of
voltage or current as a function of wavelength of incident light. As result of this measurement,
what could be the shape of the photocurrent versus wavelength graph of this sample? Plot
and explain.
6. (25 pts) A pion travelling at speed v decays into a muon and a neutrino,

. If the
neutrino emerges at 90 degrees to the original pion direction, at what angle does the come o ?
(Denote the masses of the pion, muon, and neutrino as m

, m

, and m

, respectively.)
7. (25 pts) In a steady state situation, both ions and electrons will follow the Boltzmann relation
n
i
= n
0
exp(q
i
/KT
i
)
For the case of innite, trasparent grid charged to a potential,, show that the shielding distance
is given approximately by

2
D
=
ne
2

0
_
1
KT
e
+
1
KT
i
_
Show that
D
is determined by the tempereature of the colder species.
c ODT

U Physics Department. (Nov 2011) 11


May 2011
May 2011
ANALYTICAL MECHANICS
1. Consider a mass under the inuence of a central potantial V (r) = Ar
n
where A and n are some
real constants.
(a) (10 pts) Write down the Lagrangian of the system in spherical coordinates r, and . What
are the symmetries of the Lagrangian, and corresponding conserved quantities?
(b) (5 pts) What is the Hamiltonian of the system?
(c) (10 pts) Obtain the equations of motion for the coordinates of the system.
(d) (10 pts) Using the conserved quantities of the system, prove that the problem can be reduced
to a 2 dimensional problem. Using this freedom, choose = /2. Obtain the equation
governing the time dependence of and r.
(e) (10 pts) By eliminating the time variable using the two equations, obtain the equation gov-
erning the trajectory r().
(f) (5 pts) Show that for n = 1 the trajectory is closed. (Hint: Consider the dierential equation
for 1/r())
2. Consider the system in the Figure, where two masses m
1
and m
2
are attached to each other by a
spring that has spring constant k. Each mass m
i
is also attached to a xed wall by springs that
have spring constant k
i
. For simplicity, assume that k
1
/k
2
= m
1
/m
2
. Let x
i
be the displacement
of the mass m
i
from its equilibrium position.
(a) (5 pts) What is the Lagrangian of the system?
(b) (10 pts) Find the conjugate momenta to the coordinates x
i
, and the Hamiltonian of the
system.
(c) (10 pts) One can dene the CM coordinate and the relative coordinate as
r =
m
1
x
1
+ m
2
x
2
m
1
+ m
2
y = x
1
x
2
For these transformations to be canonical, what should be the corresponding momenta?
(d) (10 pts) Obtain the new Hamiltonian in terms of the new coordinates and momenta.
(e) (5 pts) Obtain the Hamiltons equations for these new coordinates.
(f) (10 pts) Write down (do not solve) the Hamilton Jacobi equation in terms of the new coordi-
nates.
c ODT

U Physics Department. (May 2011) 13


May 2011
ELECTROMAGNETIC THEORY
1. A parallel plate capacitor has plate separation a much smaller than the plate size. Between the
plates there is a linear, uniform and isotropic material with dielectric permittivity .
(a) (15 pts) Starting from Laplace equation, nd the potential between the plates assuming that
the capacitor is charged uniformly with a surface charge density
o
. (Neglect the edge eects.)
(b) (5 pts) Conrm your result by nding electric eld using Gauss law.
(c) (20 pts) Find the current density through the material as a function of time (Assume that the
material obeys Ohms law and use I = dq/dt).
Assume now the dielectric material has a resistivity and the charge
o
is suddenly placed
to one of the plates at time t = 0.
(d) (15 pts) Find the power per unit volume delivered to the material.
2. (a) (15 pts) Use Biot-Savart law to nd that the magnetic eld produced by an innite wire
carrying current I is given as (assume that the wire is along z axis and the current is in the
z direction):

B =

0
I
2(x
2
+ y
2
)
(y x x y).
(b) (5 pts) Show that this eld obeys Maxwell Equations in free space.
(c) (20 pts) An observer is moving with a speed v = v z not necessarily small. Find the electric
and magnetic elds that she will measure. Show that

B
2


E
2
is Lorentz invariant.
You may use the followings:

= (

E +v

B)

2
+ 1
v(v

E)

B

= (

B v

E)

2
+ 1
v(v

B)
3. Consider a plane wave incident on a layered interface as shown in the gure. The index of refraction
of the three media are n
1
, n
2
, n
3
where
1
=
2
=
3
=
0
. The thickness of the intermediate layer
is d.
(a) (20 pts) Write all the boundary conditions.
(b) (10 pts) Find the transmission coecient for normal incidence as
T
1
=
v
3
E
2
T
v
1
E
2
I
=
1
4n
1
n
3
_
(n
1
+ n
3
)
2
+
(n
2
1
n
2
2
)(n
2
3
n
2
2
)
n
2
1
sin
2
(
n
2
d
c
)
_
(c) (10 pts) What is the minimum (non zero) thickness of the layer (d) in order to have maxi-
mum transmission? (Use the result of part 3b)
Hint: In the rst layer, there is an inci-
dent wave and a reected wave. In the
third layer, there is a transmitted wave.
Inside the intermediate layer there is a
wave going to the right and a wave going
to the left. E
2
I
, E
2
T
are the absolute square
of amplitudes of incident and transmitted
electric elds respectively. v
1
, v
3
are the
speeds of waves in these regions.
c ODT

U Physics Department. (May 2011) 14


May 2011
METHODS OF MATHEMATICAL PHYSICS
1. (a) Show that x = 0 is an irregular singular point of
d
2
y
dx
2
+ 2x
2
dy
dx
+

x y = 0, x 0
(b) Show that the change of variable x = t
2
, with t =

x 0, transforms the given equation into


the form
t
d
2
y
dt
2
+ (4t
6
1)
dy
dt
+ 4t
4
y = 0
with a regular singular point at t = 0.
(c) Obtain a power series for y(t) valid near t = 0, by using the smaller root of indicial equation.
2. (a) Find the Laurent series of
f(z) =
e
z
z(z + 2)
2
about the singularities z = 0 and z = 2.
(b) Verify that z = 0 is a pole of order 1 and z = 2 is a pole of order 2.
(c) Find the residue of f(z) at each pole.
(d) Using Cauchys integral formula, evaluate
_
C
e
z
z(z + 2)
2
dz
where C is the circle [z[ = 3.
3. Note that part a and part b are not related.
(a) In 1 space dimension, consider the quantum mechanical bound state problem in a potential
V (x). The wave functions obey the time-independent Schrodinger equation


2
2m

(x) + V (x)(x) = E(x),


where V () = 0 and E < 0. Show that there cannot be two linearly independent solutions
for the same value of E. Hint: Calculate the Wronskian.
(b) Carry out the following integrals.
i.
I =
_
d
3
r
1
r
2
+ a
2

3
(r

b), (1)
where a is a non-zero number and

b is a non-zero constant vector. The integration is over


the entire space,
3
(r

b) is the three dimensional Dirac-delta function.


ii.
I =
_

dx(x + 1)
2
(cos(x)). (2)
c ODT

U Physics Department. (May 2011) 15


May 2011
QUANTUM MECHANICS
1. A particle of mass m is moving in a harmonic oscillator potential so that its Hamiltonian is H =
1
2m
p
2
+
1
2
m
2
x
2
.
(a) (10 pts) Using uncertainy principle, prove that the lowest energy of the oscillator is

2
.
(b) (10 pts) Show that in terms of the operators A =
_
m
2
_
x + i
p
m
_
and A

, the Hamiltonian
operator becomes H =
_
A

A +
1
2
_
(c) (10 pts) Consider the eigenvalue problem for the operator
N = A

A: N [ n >= n [ n >.
Show that
A [ n >=

n [ n 1 >, A

[ n >=

n + 1 [ n + 1 >
(d) (10 pts) Prove that the eigevalues n are nonnegative integers, n=0, 1, 2, 3, . . .
(e) (10 pts) If the state vector of the particle at t = 0 is given by
[ , t = 0 >=
5

50
[ 0 > +
4

50
[ 1 > +
3

50
[ 2 >
. What is the average energy in this state? Calculate the state vector [ , t > for t > 0.
2. A spin-
1
2
particle of mass m and magnetic moment =
0

S is moving in an innite potential well


V (x) with walls at x =
L
2
and x =
L
2
: V (x) =
_
_
_
x <
L
2
0
L
2
< x <
L
2
x >
L
2
(a) (10 pts) Calculate the energy eigenvalues and the corresponding energy eigenfunctions. Is the
energy spectrum degenerate, why ?
(b) (15 pts) The particle is in the ground state. Suddenly the walls of the potential well are moved
to x = L and x = L. What is the probability that the particle will remain in the ground
state of the new potential? Explain your method of solution.
(c) (25 pts) Now, an external weak magnetic eld

B =
_
B z L x 0
B x 0 x L
is introduced. Using rst-order degenerate perturbation theory, calculate the energy of the
n
th
excited state of this particle.
Note:
x
=
_
0 1
1 0
_

y
=
_
0 i
i 0
_

z
=
_
1 0
0 1
_
3. Under a Lorentz transformation (LT), x

= x, the Dirac spinor wavefunction transforms as

(x

) = S()(x) where S() is a 4 4 matrix in the Dirac spinor space.


(a) (10 pts) Show that the Dirac equation is covariant (invariant in form),
i.e., (i

mc)

(x

) = 0 provided that S
1
()

S() =

c ODT

U Physics Department. (May 2011) 16


(b) (10 pts) Consider the innitesimal transformation

=
_
e

.
Show that

is anti-symmetric.
(c) (10 pts) For the innitesimal case, one can parametrize S() as
S() = I
i
4

.
Show that

= (i/2)[

] satises the equation in part (a).


(d) (10 pts) Show that, in the nite case S() can be expressed as
S() = e

i
4

.
(e) (10 pts) Work out S() explicitly in the 4 4 matrix form for the Lorentz boost in the
x-direction.
c ODT

U Physics Department. (May 2011) 17


May 2011
STATISTICAL MECHANICS
1. A system of N bosons in two dimensions has an energy-momentum relationship E=cp
2/3
and
number density n=N/A (A is the area).
(a) (25 pts) Show that at low temperatures the system will Bose condense and that the Bose
condensation temperature goes like T
C
n

. Find .
(b) (25 pts) Show that the entropy below T
C
is S=T

, and the pressure (in two dimensions


has units of force per unit length) is P=T

. Find and .
2. Consider an ideal gas of N spin-1/2 fermions of mass m in two space dimensions. Let A be the
area of the gas so that the particle density is n=N/A.
(a) (5 pts) Determine the Fermi wave vector k
F
, the Fermi energy E
F
, and the Fermi temperature
T
F
for this gas.
(b) (5 pts) Determine the density of states (E) for this gas.
(c) (10 pts) Obtain a closed form expression for the chemical potential for this gas as a function
of temperature. Show that in the limit of high temperatures (T >> T
F
) the fugacity is
approximately given by z
2
/l
2
where is the thermal wavelength and l =
_
A/N is the
mean particle spacing.
(d) (10 pts) Obtain the specic heat at constant area of this gas in both the low temperature T
<< T
F
and high temperature T >> T
F
limits.
c ODT

U Physics Department. (May 2011) 18


May 2011
MISCELLANEOUS PHYSICS PROBLEMS
1. Consider the Youngs double slit experiment. Let the width of the fringes observed around the
optical center on the screen be W. We use either a white light source or a laser beam of wavelength
. The distance between the slits (d) and the width of the slits (b) are adjustable. Answer the
following questions; write the necessary relations and use.
(a) (6 pts) For the same d, compare the values of W that would be obtained with white light and
with laser light. Explain why.
(b) (6 pts) How would the value of W change with d ? Answer for both kinds of light sources.
Explain why.
(c) (6 pts) Describe the colors of the fringes when white light is used. Explain your answer by
making simple calculations.
(d) (6 pts) Assume that one of the slits is wider than the other and the laser is used. Compare
the resulting interference pattern with that would be obtained with identical slits.
(e) (6 pts) Under the laser illumination, one of the slits is closed. Compare the resulting interfer-
ence pattern with that would be obtained with two open identical slits.
(f) (6 pts) Consider the laser illumination and two identical open slits. Assign realistic values to
, d, L. Calculate the position of the rst bright fringe following the central one.
(g) (14 pts) Consider the laser illumination and two identical open slits. An optical element that
shifts the phase of the wave by 180
o
, and having a transmission coecient of 4/9 is placed in
front of one of the slits. Find the expression of the intensity on the screen as a function of
the distance measured from the optical center and plot I-y graph. On your plot, indicate the
positions of maxima and minima and their values. (Let the intensity due to single slit with
no phase shifter be I
o
).
2. Explain and discuss the following briey.
(a) (5 pts) The correspondence principle in Bohr atomic theory.
(b) (10 pts) The introduction of the concept of quanta or quantum by Planck in an attempt to
resolve the ultraviolet catastrophe in a black body radiation.
(c) (5 pts) The mechanism of producing X-rays and their discovery by Wilhelm Conrad Roentgen.
If a light of wavelength 600 nm and intensity 0.020 W/m
2
falls onto a metal surface (cathode) of
area A = 5 cm
2
and the stopping potential is 0.5 volts, nd
d (20 pts) the photo current produced in this process, assuming an eciency of 25 percent.
c ODT

U Physics Department. (May 2011) 19


d (10 pts) the cut o wavelength.
3. The two dimensional square crystal sample (surface) with an interatomic distance of a and
the length of L along each direction is given.
(a) (4 pts) Write down the basis vectors, a,

b, and

A,

B for the direct and indirect (reciprocal)
lattices, respectively.
(b) (5 pts) Draw the 1
st
Brillioun zone.
In Free electron theory,
c (25 pts) nd the free electron energies of the sample at

k =

0 and along [0 k
y
0] direction, by
taking only the rst and second nearest neighbor atoms into account,
c (10 pts) plot the calculated energy bands of the free electron along [0 k
y
0] direction in the
reduced zone scheme.
c (6 pts) Explain, why Free electron theory can not explain the distinction between the metals,
semiconductors and insulators.
4. (a) Consider a star which is completely supported by radiation pressure (P
rad
=aT
4
/3 where a is
the radiation constant and T is the temperature). If this star is in hydrostatic equilibrium
nd its luminosity (called Eddington luminosity) in terms of
gr
, M, c and G where
gr
is
the opacity in cm
2
/gr, M is the mass of star, c is the speed of light and G is the gravitational
constant. The radiative temperature gradient and the equation of hydrostatic equilibrium are
given as
(dT/dr)
rad
= 3
gr
L/4acT
3
4r
2
and (dP/dr) = g
where g is the gravitational acceleration.
(b) (10 pts) Obtain a numerical estimate for this luminosity as a function of (M/M
sun
) by as-
suming that the surface of star is hot enough for the opacity to be dominated by electron
scattering (
e,gr
=
e
n
e
/ where
e,gr
is the electron scattering opacity,
e
is the Thomson
cross section and n
e
is the number of free electrons per cm
3
). Take the fractional content (by
mass) of the hydrogen as 0.7.
(c) (15 pts) What would happen to star if its luminosity exceeds this luminosity?
(d) (10 pts) The mean opacity for the Sun is 1 cm
2
/gr. How luminous is the Sun as a fraction of
the Eddington luminosity? Comment on your value.
(e) (15 pts) Derive the central temperature of a hot star supported only by radiation pressure. Use
the equation of hydrostatic equilibrium to estimate the central pressure. Hot Main-Sequence
stars of mass M have radius of R=(M/M
sun
)
1/2
R
sun
.
Some constants: Boltzmann constant= 1.38x10
16
erg/K, Plancks constant=6.63x10
27
erg/s,
Gravitational constant= 6.67x10
8
dyn cm
2
/g
2
, radiation constant= 7.6x10
15
erg/cm
3
K
4
, mass
of Sun= 2x10
33
g, radius of Sun= 7x10
10
cm, luminosity of Sun= 3.90x10
33
erg/s, mass of hydrogen
atom= 1.67x10
24
gr, Thomson cross section= 6.6x10
25
cm
2
.
5. Particle physicists are so used to setting the speed of light c = 1 that they measure mass in units of
energy. In particular, they tend to use electron volts (eV) or its multiples (keV, MeV, GeV, etc...).
You may already know that a muon, just like an electron, has unit negative charge 1.6 10
19
C.
The muon at rest has been measured to have a mass of 0.106 GeV, but for simplicity let us take it
to be just 0.100 GeV for this problem.
c ODT

U Physics Department. (May 2011) 20


(a) (10 pts) What is the mass of the muon at rest in SI units of kg?
In what follows, let us consider a muon which is moving in the circular storage ring of a
particle accelerator, such that the mouns total energy is 1000 GeV.
(b) (10 pts) What is the factor with which such a muon is moving?
(c) (10 pts) If the rest-frame lifetime of a muon is 2.2 10
6
s, how long does a muon in the
storage ring live from the point of view of an experimenter?
(d) (20 pts) Finally, if the radius of the storage ring is 1 km, what is the proper acceleration of
the muon in the ring?
6. (50 pts) The half life of a

meson at rest is 2.510


8
s. A beam of

meson is generated at a
point 15m from a detector. Only 1/2 of the

mesons live to reach the detector. What is the


speed of the

mesons? Express your answer in terms of the speed of light, c.


7. (50 pts) The Debye length
D
and Debye shielding are important concepts in plasma physics.
(a) Prove that

2
D
=
kT
4ne
2
.
(b) Suppose that a point charge q is introduced into a thermal uniform plasma. After the plasma
settle down to steady state nd the electrical potential (r). Assume that both electron and
ion densities obey Maxwell-Boltzmann distribution
n
e
= n
e,o
exp
_
e
kT
e
_
, n
s
= n
s,o
exp
_
Z
s
e
kT
s
_
, n
e,o
=

Z
s
n
s,o
s shows several ions, and use Poissons equation

2
=
1
r
d
2
dr
2
(r) = 4
_
e(n
e
n
e,o
) +

eZ
e
(n
s
n
s,o
)
_
8. (50 pts)
(a) What are the physical identication methods that come out as a result of interaction of light
with the materials to be used for the analysis of material properties? Write and explain each
of them.
Assume you have n and p-type semiconductors
(b) What makes them dierent from each other and metals ? Write and express their conduc-
tivities, carrier concentrations, mobilities and also plot the energy band diagram by showing
each energy level explicitly.
(c) When you make n-p junction, explain ow of charges and energy band diagram of this system
in details.
(d) Assume you have p-n (or n-p) junction produced by using these semiconductors with dierent
band gap values (one has 1.24 eV and the other one 2.48 eV) are placed on top of each
other and you are making photoresponse measurement which is the measurement of voltage
or current as a function of wavelength of incident light. As result of this measurement, what
could be the shape of the photocurrent versus wavelength graph of this sample? Plot and
explain.
c ODT

U Physics Department. (May 2011) 21


Nov 2010
Nov 2010
ANALYTICAL MECHANICS
1. Consider a charged particle constraint to move in the xy-plane. Assume that there is a constant
magnetic eld in the z direction given by

B = B
0
z, where B
0
is a constant. This magnetic eld
can be described by a vector potential

A =
1
2
r

B, such that

B =

A. The Lagrangian of
the particle can be written as
L =
1
2
m

r
2
+ q

r

A (1)
where

r = x x + y y
(a) (10 pts) Obtain the equations of motion for this particle.
(b) (5 pts) What are the canonical momenta, P
x
and P
y
, for the particle?
(c) (5 pts) Obtain the Hamiltonian for the particle.
(d) (10 pts) Consider the transformations
P
1
=
_
P
y

qB
0
2
x
_
1

m
, Q
1
=
_
P
x
+
qB
0
2
y
_

m
qB
0
P
2
=
_
P
x

qB
0
2
y
_

m
qB
0
Q
2
=
_
P
y
+
qB
0
2
x
_
1

m
,
(2)
Show that these transformations are canonical?
(e) (5 pts) Obtain a generating function for these transformations.
(f) (5 pts) Write down the new Hamiltonian in terms of the new coordinates and momenta.
(g) (10 pts) Obtain the solution for the trajectory of the particle by rst solving the equations of
motion in terms of the new coordinates, and then transforming back to the original coordi-
nates. What is the shape of the trajectory?
2. Consider a point like mass, m, inside a sphere of radius R. The mass is constraint to move on the
interior surface. Denote the gravitational acceleration by g.
(a) (5 pts) What is the Lagrangian of the system?
(b) (10 pts) What are the symmetries and the corresponding conserved quantities?
(c) (10 pts) What is the Hamiltonian of the system?
(d) (5 pts) Write down the Hamiltons equations for the system?
(e) (5 pts) For very small deviations from the bottom of the sphere, what is/are the oscillation
frequency/ies?
(For the following questions, assume small oscillations)
(f) (10 pts) What is the Hamilton-Jacobi equation for the system?
(g) (5 pts) Obtain the action as a solution of Hamilton-Jacobi equation. (You can use separation
of variables)
c ODT

U Physics Department. (Nov 2010) 23


Nov 2010
ELECTROMAGNETIC THEORY
1. Consider a spherical cavity in a material of innite extend with magnetic permeability . A uniform
magnetic eld is applied to the material as shown in the gure.
(a) (20 pts) Find the magnetic scalar potential inside and outside the cavity.
(b) (10 pts) Find the magnetic eld inside and outside the cavity.
(c) (10 pts) Find the magnetization inside and outside the cavity.
(d) (10 pts) Find the magnetic surface charge density.
2. A nonconducting thin circular ring of radius a lies on xy plane. A linear charge density (x

) is
placed on the ring.
(a) (10 pts) What is the electric dipole moment of this charge density?
Suppose now the ring is rotated counterclockwise with a constant angular velocity about the z
axis. What is the time averaged power radiated per unit solid angle if
start=b (10 pts) =
0
, which is constant;
stbrt=b (10 pts) =
0
cos , where is azimuthal angle (consider only the electric dipole moment
elds in the far zone).
stcrt=b (10 pts) For part 2a nd the electric eld for the far points (Use far point approximation,
that is: [x x

[ [x[ = r >> [x

[).
stdrt=b (10 pts) For part 2b nd the vector potential for the far zone due to electric dipole.
You may use the following information:
p(x) =
_
x(x)d
3
x
dP
d
=
1
2
Rer
2
n EH

Electric dipole elds for an oscilating charge:


H =
ck
2
4
(n p)
e
ikr
r
(1
1
ikr
)
E =
1
4
0
k
2
(n p) n
e
ikr
r
+ [3n(n p) p](
1
r
3

ik
r
2
)e
ikr

A =

o
4
e
ikr
r
_
J(x

)d
3
x

n =
x
r
c ODT

U Physics Department. (Nov 2010) 24


3. The relativistic Lagrangian for a charged particle in external electromagnetic eld is given as
L = mc
2
_
1
u
2
c
2
+
e
c
u

Ae
(a) (20 pts) Using the relativistic Lagrangian derive the Hamiltonian.
(b) (15 pts) Using part 3a dene the 4-vector momentum.
(c) (15 pts) Show that scalar product of 4-momentum is invariant.
c ODT

U Physics Department. (Nov 2010) 25


Nov 2010
METHODS OF MATHEMATICAL PHYSICS
1. (a) (30 pts) The residue theorem can often be used to sum various types of series. You are given
that

n=
f(n) = [sum of the residues of ( cot z)f(z) at the poles of f(z)].
Using this information calculate the sum

n=
1
n
2
+ a
2
where a > 0,
and

n=1
1
n
2
+ a
2
(b) (20 pts) Evaluate the integral
I =
_
|z|=4
e
z
(z
2
+
2
)
2
dz
2. (a) (20 pts) It can be shown that the Laguerre functions are orthogonal on the interval (0, )
with respect to the weighting function e
x
, thus
_

0
e
x
L
n
(x)L
m
(x)dx = c
n

n,m
for some constant c
n
. Using the generating function for the Laguerre functions, which is
dened as,
g(x, t) =
e
xt/(1t)
1 t
=

n=0
L
n
(x)t
n
Find the value of constant c
n
.
(b) (10 pts) Using the result of part (a), derive the closure relation for the Laguerre functions,
i.e, evaluate

n=0
L
n
(x)L
n
(x

)
(c) (10 pts) Verify the identity
x

2
g
x
2
+ (1 x)
g
x
+ t
g
t
= 0
and derive the dierential equation satised by L
n
(x).
(d) (10 pts) Verify the following recursion relations:
L

n+1
L

n
+ L
n
= 0
xL
n
nL
n
+ nL
n1
= 0
3. f (x) = x
3
x is dened on the interval x [0, 1].
c ODT

U Physics Department. (Nov 2010) 26


(a) (15 pts) Determine the sine Fourier series of this function.
(b) (15 pts) Determine the cosine Fourier series of this function.
(c) (10 pts) Calculate

k=0
(1)
k
(2k+1)
3
.
(d) (10 pts) Calculate

n=1
1
n
6
.
c ODT

U Physics Department. (Nov 2010) 27


Nov 2010
QUANTUM MECHANICS
1. (a) (20 pts) A particle in a spherically symmetric potential is known to be in an eigenstate of

L
2
and L
z
with eigenvalues
2
(+1) and m, respectively. Starting from the angular momentum
commutation relations, prove that the expectation values between [ m) states satisfy
L
x
) = L
y
) = 0 L
2
x
) = L
2
y
) =

2
2
[( + 1) m
2
]
(b) (10 pts) Now, consider a particle which is initially in the state
[ ) =
1

5
[ 1, 1) +
3

5
[ 1, 0) +
1

5
[ 1, 1)
If L
z
is measured what values will one obtain and with what probabilities.
(c) (20 pts) If after measuring L
z
on the state [ ), we nd m = 1, calculate the uncertainties
L
x
and L
y
and their product L
x
L
y
2. Consider a particle of charge q and mass m in a magnetic eld

B(x) where

B(x) =


A(x)
with

A(x) being the vector potential.
(a) (5 pts) Write the time dependent Schrodinger equation.
(b) (10 pts) Calculate [
x
,
y
] where = p
q
c

A.
(c) (20 pts) Show that if we perform a gauge transformation

A(x)

A(x) +

(x), this trans-
formation must be accompanied by a phase change in the wavefunction which is
(x, t) exp[
iq
c
(x)](x, t).
(d) (15 pts) Using this result, discuss the Aharonov-Bohm eect. Can this eect be explained
classically?
3. By considering the expectation value of the operator
1
r
r p in a stationary state, one can prove the
following general virial theorem:
_
1
r
_
p
2

1
r
2
(r p)
2
__
= m
_
1
r
r

V
_
. (1)
You can use this expression without proof in the following.
(a) (12.5 pts) Compute

L
2
explicitly in terms of the r and p. Here

L = r p is the orbital angular
momentum operator.
(b) (15.0 pts) Using the result in 3 together with the equation 1, show that for a particle bound
to a xed center by a spherically symmetric potential V (r)
[(0)[
2
=
m
2
2
_
dV (r)
dr
_
(2)
for all s states ( = 0), ground or excited.
c ODT

U Physics Department. (Nov 2010) 28


(c) (22.5 pts) One can show that one of the ne structure correction terms one gets for the
Coulomb Hamiltonian from the Dirac equation is the so-called Darwin term:
H
D
=
e
2

2
2m
2
c
2

(3)
(r) . (3)
st1rt=1 Show that, this term indeed leads to a correction of order
4
.
st2rt=2 Compute the shift to the unperturbed energy, by using equations 2 and 3
E
(0)
n
=

2
2n
2
mc
2
to the lowest order by treating H
D
as a perturbation. Show that this contribu-
tion is nonzero only for the s states by recalling that for small r, the unperturbed
wavefunction behaves like R(r) r

as r 0.
c ODT

U Physics Department. (Nov 2010) 29


Nov 2010
STATISTICAL MECHANICS
1. A system having three states equally spaced by energy is pumped into an internal equilibrium
condition such that P
1
=2P
0
. Here, P
n
is the probability of occupation of state n (n=0, 1, 2) and
n=0 is the ground state with energy E
0
(E
0
<E
1
<E
2
).
(a) (10 pts) What is the equilibrium temperature of the system?
(b) (10 pts) What is the average energy of the system?
(c) (20 pts) Now suppose that the equilibrium temperature of the system is , calculate the
following quantities: average energy, Helmholtz free energy, and entropy.
(d) (10 pts) Find the equilibrium temperature of the resulting closed system if two systems in
part 1a/1b and 1c are brought into thermal contact.
2. (50 pts)
(a) Find the chemical potential of a two-dimensional gas of fermions.
(b) Calculate the low temperature chemical potential of a two-dimensional gas of fermions by the
Sommerfeld expansion method. Observe that the temperature series expansion terminates.
(c) Compare result for part 1a w4th the exact result of the previous question. Discuss the
dierence between the two results..
c ODT

U Physics Department. (Nov 2010) 30


Nov 2010
MISCELLANEOUS PHYSICS PROBLEMS
1. (50 pts) A beam of light of wavelength = 600nm passes through a long narrow slit of width
b=0.01 mm and strikes a screen normal to the beam, placed a distance D=2m from the slit. If the
slit is extending from y=-b/2 to y=+b/2, the dierential amplitude dE of the wave in the direction
(measured from the optical axis) can be written as
dE e
iky
sin dy
where, k = 2 is the propagation constant.
(a) Using the above information drive an expression for the intensity distribution on the screen
as a function of .
(b) Now, suppose that the screen is coated with phosphor and a beam of electrons were to be
used instead of light. Through what potential dierence should the electrons be accelerated
before they reach the slit, if the spreading of the beam is to be reduced by a factor of 10
3
below that found for light?
2. (a) (15 pts) Explain the blackbody radiation by plotting a curve of energy distribution against
wavelength for dierent temperatures and the Wien displacement law; and considering that
the cosmic background radiation is that of blackbody at 2.7K, what is the value of
max
at
which the distribution has a maximum? (Take the constant in Wiens law as 0.29 10
2
in
SI units.) Discuss briey the following forms of interaction of radiation with matter, and list
them in terms of their eectiveness as the energy of radiation increases.
Discuss briey the following forms of interaction of radiation with matter, and list them in
terms of their eectiveness as the energy of radiation increases.
i. (10 pts) Photoelectric eect
ii. (10 pts) Compton eect
iii. (10 pts) Pair production
(b) (5 pts) A metal has work function 4.7 eV. What is the kinetic energy of a photo-electron if
radiation of wavelength 200 nm falls on the surface of the metal?
3. Consider a lattice in one dimension with the periodicity a and the crystal potential,
V (x) = Ucos
2
a
x
where, U is constant.
(a) (10 pts) Find the rst Brillouin zone of the sample.
(b) (20 pts) Write and solve the central equations,
(
k
E)C(

k) +

G
V

G
C(

k

G) = 0,
in nearly free electron model in terms of
k
and V

G
at the 1
st
Brillouin zone boundary,
where

k
=

2
k
2
2m
c ODT

U Physics Department. (Nov 2010) 31


and
V

G
=
1
a
_
a
0
V (x)e
i

G.x
dx
(c) (10 pts) Find the energy gap at the 1
st
Brillouin zone boundary explicitly as a function of U.
Hint: Remember that cos
2
x =
1+cos2x
2
(d) (10 pts) Explain the existence of the energy gaps at the Brillouin zone boundaries by Braggs
reection.
4. (a) (10 pts) Explain the brightness temperature.
(b) (15 pts) Find the brightness temperature of a supernova remnant which has a measured
intensity of I

= 1.3 10
13
erg/s/cm
2
/sr/Hz at 100 MHz. Assume the emission is thermal.
You can use Rayleigh-Jeans approximation.
(c) (10 pts) Verify that Rayleigh-Jeans approximation is valid.
(d) (10 pts) Suppose the supernova remnant has an angular radius of = 2.15 arcminutes.
Calculate the ux at 100 MHz (F

= I

where is the solid angle).


(e) (5 pts) Find the wavelength
max
at which this objects radiation will be maximum, if the
emission is blackbody emission (T=T
b
).
Boltzmann constant= 1.38x10
16
erg/K, Plancks constant=6.63x10
27
erg/s, c=3x10
10
cm/s,
max
T=
0.289 K cm, I

= 2kcT
b
/
4
, I

d=I

d, I

= B

= (2h
3
/c
2
) 1/(e
h/kT
1).
(Solid angle is given as
2
where is in radians.)
5. An alien spaceship traveling at speed 0.6c toward the Earth launches a landing craft. The landing
craft travels in the same direction with a speed of 0.8c relative to the mother ship. As measured
on the Earth, the spaceship is 0.2 light-years from the Earth when the landing craft is launched.
(The speed of light is c = 3 10
8
m/s.)
(a) (15 pts) What speed do the Earth-based observers measure for the approaching landing craft?
(b) (15 pts) What is the distance to the Earth at the moment of the landing crafts launch as
measured by the aliens?
(c) (10 pts) What travel time is required for the landing craft to reach the Earth as measured by
the aliens on the mother ship?
(d) (10 pts) If the landing craft has a mass of 4 10
5
kg, what is its kinetic energy as measured
in the Earth reference frame?
6. (50 pts) If a proton with energy E (as measured in the lab) collides with a proton at rest in the
lab to produce never-before-seen particle X through the reaction p+p X, what is the maximum
possible rest mass M
X
of this particle X? Express your answer in terms of E and the rest mass of
the proton.
Answer the same question for a collision of a proton with an antiproton of the same energy E
traveling in the opposite direction.
7. (50 pts) The concept of the magnetic dipole moment is useful in the description of the motion of
a charged particle in a slowly increasing cylindiracally symmetric magnetic eld in the z-direction
(dB
z
/dz is small and positive). What would be the trajectory of a charged particle of charge q
and mass M in such a magnetic eld? Explain this trajectory qualitatively and quantitatively.
One can call this type of magnetic eld; magnetic bottle or magnetic mirror. Also Van Allen Belts
of the Earth poses a good example to this type of magnetic elds.
c ODT

U Physics Department. (Nov 2010) 32


Take the magnetic eld as:

B = B
r
r + B
z
z
8. (50 pts) Assume you have n and p-type semiconductors.
(a) What make them dierent from each other and metal? Write and express their conductivities,
carrier concentrations, mobilities and also plot the energy band diagram by showing each
energy level.
(b) When you make n-p junction, explain ow of charges and energy band diagram of this system
in details.
(c) Assume you have p-n (or n-p) junction produced by using these semiconductors with dierent
band gap values (one has 1.24 eV and the other one 2.48 eV) are placed on top of each
other and you are making photoresponse measurement which is the measurement of voltage
or current as a function of wavelength of incident light. As result of this measurement, what
could be the shape of the photocurrent versus wavelength graph of this sample? Plot and
explain.
c ODT

U Physics Department. (Nov 2010) 33


May 2010
May 2010
ANALYTICAL MECHANICS
1. Consider two point masses m
1
and m
2
. The rst mass, m
1
, is attached to a xed point by a
massless rod of length l
1
. The second mass m
2
is attached to the rst mass by another massless
rod of length l
2
(see the Figure above). Both of the masses are restricted to move in a plane,
keeping the length of the rods xed. (Let g be the constant gravitational acceleration)
(a) (10 pts) What is the Lagrangian of this system?
(b) (5 pts) What are the conserved quantities for this system?
In the following questions, take the small oscillations limit, i.e., the angle that the rods make
with the vertical is small, and also assume that l
1
= l
2
= l and m
1
= m
2
= m.
(c) (10 pts) Obtain the Hamiltonian corresponding to this system?
(d) (10 pts) Write down the Hamiltons equations.
(e) (10 pts) What are the normal modes?
(f) (5 pts) Assume that at t = 0, the rst mass is straightly below the suspension point, and the
rod connecting the second mass to the rst mass makes an angle
0
<< 1 with the vertical.
Both masses are initially at rest. Find the position of both the masses as a function of time.
2. A particle of mass M moves with angular momentum l in the eld of xed force center with
F(r) =
k
r
2
+

r
3
,
where k and are positive constants.
(a) (15 pts) Use the eective potential energy to discuss the qualitative nature of orbits for
dierent values of the energy.
(b) (15 pts) Solve the orbital equation and show that the orbit has the form,
r() =
c
1 + cos()
,
where c, and are positive constants.
(c) (8 pts) Find c and in terms of the given parameter.
(d) (12 pts) For what values of , the orbit is closed? What happens to your result as 0?
c ODT

U Physics Department. (May 2010) 35


NOTE: The dierential equation of the orbit for a central force is
d
2
d
2
(
1
r
) +
1
r
=
r
2
l
2
F(r).
c ODT

U Physics Department. (May 2010) 36


May 2010
ELECTROMAGNETIC THEORY
1. A sphere of dielectric contant and radius a is located under the uniform external electric eld
along z-axis.
(a) (35 pts) Find the polarization surface charge density.
(b) (10 pts) If the sphere starts to rotate along z-axis with constant angular frequency nd the
magnetization density.
(c) (5 pts) Using the solution of part (b) dene the magnetic scalar potential and induced magnetic
eld.
2. (a) (5 pts) Write the Maxwell equations in vacuum for a charge density and current density free
medium ( = 0 and

J = 0).
(b) (5 pts) Show that the electric eld and magnetic eld satises the wave equation.
(c) (5 pts) Write down the solutions of wave equations (plane waves) for the electric eld and
magnetic eld. How are they related to each other? Find the group velocity and phase velocity
of the electromagnetic waves.
(d) (35 pts) Show that an electromagnetic wave satises the relation

E

B = 0 and E
2
= c
2
B
2
in
any Lorentz frame. (Hint: express the Lorentz scalars F

and F

in terms of the
electric and magnetic eld.)
F

=
_
_
_
_
0 E
x
/c E
y
/c E
z
/c
E
x
/c 0 B
z
B
y
E
y
/c B
z
0 B
x
E
z
/c B
y
B
x
0
_
_
_
_
,
F

can be obtained by the substition



E/c

B and

B

E/c.
3. Four point charges q and 2q are placed at x = a and y = a as shown in the gure. The
charges oscillate at frequency in phase.
(a) (5 pts) Find the electric dipole moment of the charge distribution.
(b) (10 pts) Find the electric quadrupole moment of the charge distribution.
(c) (35 pts) Find the emitted power per unit solid angle at the far eld zone.
c ODT

U Physics Department. (May 2010) 37


May 2010
METHODS OF MATHEMATICAL PHYSICS
1. The dierential equation for transverse vibrations of a string whose density increases linearly from
one end to the other is
d
2
Y
dx
2
+ (x + )Y = 0,
where and are nonzero constants.
(a) (30 pts) By dening the new independent and dependent variables
z = x + ,
Y (z) =

zU(z),
= (2/3)z
3
2
successively, show that the given dierential equation can be transformed to

2
d
2
U
d
2
+
dU
d
+ (
2

1
9
)U = 0.
(b) (20 pts) Write down the general solution of this system in terms of Bessels functions.
2. (a) (30 pts) By contour integration, verify that
_
2
0
d
a + b cos
=
2

a
2
b
2
,
where [ b [< a, a > 0.
(b) (20 pts) Then, evaluate the following integral,
_
2
0
d
(a + b cos )
2
.
3. Note that part a and part b are not related.
(a) (25 pts) Find the Greens function for the following boundary value problem.
y

(x) k
2
y(x) = f(x), x (, ) k > 0,
where y(x) remains nite as [x[ .
(b) (25 pts) In 3 dimensions nd the Greens function (that vanishes at innity) for the following
operator
L =
2
k
2
(1)
Hint: Use the Fourier transform technique for part b.
c ODT

U Physics Department. (May 2010) 38


May 2010
QUANTUM MECHANICS
1. Consider a particle of mass m in one dimension subject to the potential V (x) = V
0
(x) where
V
0
> 0.
(a) (15 pts) Find the wave function and the binding energy of the ground state. Are there excited
bound states?
(b) (15 pts) What is the probability that the particle remains bound when suddenly the strength
of the potential V
0
becomes (i)
1
2
V
0
, (ii) 4V
0
?
(c) (20 pts) Consider the scattering case (E > 0), and calculate the reection and transmission
coecients as a function of the wave number k.
2. Consider an electron, denote its spin operator by

S =

2
where are Pauli spin matrices.
(a) (15 pts) Calculate the eigenvalues and the eigenspinors of the operator S
x
cos + S
y
sin .
(b) (15 pts) An electron is subjected to an external static uniform magnetic eld in the z-direction
so that its Hamiltonian is H = S
z
. If the initial state of the electron is given as (0) =
_
a
b
_
, calculate (t).
(c) (15 pts) If initially the electron is in the eigenstate of S
x
with eigenvalue

2
, what is the
probability that it will be found in an eigenstate of S
x
with eigenvalue

2
, and

2
for t > 0?
(d) (15 pts) For the initial state in part c, calculate < S
x
> and < S
y
> for t > 0. Interpret your
result physically.
Note:
x
=
_
0 1
1 0
_

y
=
_
0 i
i 0
_

z
=
_
1 0
0 1
_
3. Consider the free Dirac equation for a particle with mass m and charge e
(,p mc)(x) = 0 .
One can introduce the electromagnetic interaction [described by the potentials A

= (,

A)] via
the minimal substitution p

(e/c)A

.
(a) (10 pts) Work out the continuity equation and identify the probability density and the prob-
ability current explicitly in the
i. free case,
ii. and the interacting case.
(b) (10 pts) Work out the velocity operator, v
op
, in the Heisenberg picture.
(c) (10 pts) What are the eigenvalues of the operator (v
op
i
)
2
(for any i)? Compare this against
the corresponding classical expression by rst proving that
(v
cl
i
)
2
=
c
2
p
2
i
p
2
+ m
2
c
2
< c
2
for any i.
c ODT

U Physics Department. (May 2010) 39


(d) (10 pts) Work out the commutator [v
op
i
, v
op
j
] in the free and interacting cases explicitly. Based
on this result, discuss whether you can measure two dierent components of v
op:
simultane-
ously. If not, work out the Heisenberg uncertainty product using
(A)(B)
1
2
[[A, B])[
for the components v
op
x
and v
op
y
explicitly.
(e) (10 ptws) Prove that, for a free Dirac particle , H
D
= 2c p.
i. Using this expression show that
c
i
) =
_
c
2
E
p
i
_
in energy eigenstates. Is this consistent with the corresponding classical expression?
ii. If so, interpret your result by recalling how you dene the state of a free particle with
xed positive energy p
0
= E/c and momentum p.
iii. Compute the commutator [
i
, H
D
] for the free case. Using this result, discuss whether
the velocity operator has a sharp eigenvalue in the energy eigenstate.
iv. Recalling how one denes the state of a (say) free electron with xed positive energy p
0
and momentum p, try to reconcile the results in (3c) and (3(e)i).
c ODT

U Physics Department. (May 2010) 40


May 2010
STATISTICAL MECHANICS
1. A crystalline sample consists of a large number (M>>1) of molecules in equilibrium at temperature
T (in K). Every molecule in the system behaves like a 1-dimensional simple harmonic oscillator
of frequency
o
. (i.e., E
n
= n.h
o
, n=0, 1, 2,.. . . . . . ..,). Let P
n
be the probability of nding a
molecule at the n
th
energy level. The plot shows P
0
versus =(k
B
T)
1
. Use the plot to answer
the following questions.
(a) (10 pts) What is the value h
o
in meV ? (up to 1 signicant digits)
(b) (10 pts) What is the partition function of the sample at =0.3 meV
1
?
(c) (10 pts) What is the average energy of the sample at =0.6 meV
1
?
(d) (20 pts) What is the heat capacity at constant volume per molecule at =0.7 meV
1
? Express
in terms of k
B
.
2. Consider a white dwarf that has a mass M and consists purely of helium atoms (with atomic mass
6.410
27
kg). The temperature of the star is typically 10
7
K, but nevertheless we assume that
this temperature is much smaller than the Fermi energy of the electrons in the star.
(a) (5 pts) If the radius of the star is R, what is the Fermi energy of the electrons in it ? Assume
that the electrons are non-relativistic, and are not bound to the helium nuclei. Express your
answer in terms of M and R and other physical constants.
(b) (5 pts) What is the total energy of the electron gas?
(c) (10 pts) What is the gravitational energy of the star? (G=6.6710
11
m/J.kg
2
).
(d) (10 pts) Minimize the total energy (gravitational+electronic) to nd the equilibrium radius
of the star. (typically, the mass of the white dwarf is of the order of a Solar mass, 10
30
kg.)
(e) (10 pts) Assume the opposite limit of ultra-relativistic electrons with E=cp=ck as their
kinetic energy. What is the energy of the gas now? Show that this energy has the same
dependence on R, the radius of the star, as the gravitational energy.
(f) (10 pts) From the conditions of stability nd a condition on the mass of the white dwarf.
Compare this critical mass with the mass of the Sun.
c ODT

U Physics Department. (May 2010) 41


May 2010
MISCELLANEOUS PHYSICS PROBLEMS
1. Consider a glass sphere of radius of curvature R and refractive index n. In dark, a laser beam with
a diameter of 2mm is sent along the optical axis from a 15.7 mW diode laser pointer emitting
=6505 nm.
(a) (10 pts) What must be the refractive index n of the glass, if we want the beam exiting from
the front surface, after suering one reection from the back surface, to be almost at the same
width and parallel to the incident beam.
(b) (5 pts) Calculate the average irradiance, and the amplitude of its electric and magnetic elds
for the incident beam.
Now, assume a normal incidence at each surface.
(c) (20 pts) Calculate the number of photons/m
2
.s in air at the back (N
B
) and in front (N
F
) of
the sphere.
(h=6.6210
34
J.s,
0
=8.8510
12
(C.s)
2
/kg.m
3
,
0
=410
7
N.A
2
)
(d) (15 pts) Is it possible to see interference eects due to beams reected from the back and
front surface of the sphere if R=10 cm? Explain your answer in terms of coherence and other
related concepts.
2. The energy of the electron in a three dimensional crystal structure is derived to be
E
k
= E
o

i=1
e
i

k.r
i
(1)
by Tight binding model. Here E
o
and are the constant energies.

k and r
i
are the wave vector
and the position vectors of the atoms, respectively. The summation is taken over the rst nearest
neighbor atoms.
(a) (10 pts) Find the velocity of the electron at k=(

a
,

a
,

a
) point.
(b) (10 pts) Explain the value of the velocity at k=(

a
,

a
,

a
) point by Braggs reection.
(c) (10 pts) Find the eective mass of the electron at k=(

a
,

a
,

a
) point.
(d) (10 pts) Explain the meaning of the negative nite eective mass.
c ODT

U Physics Department. (May 2010) 42


3. Top quarks are the most massive particles with mass m
t
= 175 GeV/c
2
. Suppose many top quarks
are produced in an accelerator lab with an average total energy of 245 GeV by special reactions.
Top quarks decay with a half life of about 7 10
26
sec.
(a) (20 pts) What is the average speed of a top quark in an accelerator lab?
(b) (15 pts) What is the half life measured in an accelerator lab?
(c) (15 pts) Those top quarks travel in the lab and decay as they go. Suppose 1/2 of them have
not decayed after travelling a certain distance. What is that distance?
4. (a) (15 pts) A spaceship travels to a star 5 light-years away, in a time its crew considers to be 5
years. What is the speed of the ship?
(b) (15 pts) A stellar object at some known large distance ejects a jet at speed v towards an
observer obliquely, making an angle with the line of sight. To the observer the jet appears
to be ejected sideways at speed V . Find V in terms of v, and the speed of light c. What is
the value of V when = /4 ? Comment on the result you have found and its meaning.
(c) (10 pts) Two particles move along the x-axis of an inertial frame S at velocities 0.8 c and 0.9
c, respectively, the faster one momentarily 10 m behind the slower one. How many seconds
elapse before collision?
(d) (15 pts) A rod of proper length 1 m moves longitudinally along the x-axis of an inertial frame
S at speed c/2. How long (relative to the inertial frame S) does it take a particle, moving
oppositely at the same speed, to pass the rod?
5. A plasma medium consist of a gas of free electrons and ions. Since the ions are heavier than the
electrons, this gas has natural oscillations of plasma frequency
p
.
(a) (20 pts) Prove that it is given by
_
Ne
2

o
m
e
, where N is the number of free electrons per unit
volume, e and m
e
is the charge and mass of an electron, respectively.
(b) (20 pts) An electromagnetic wave propagates through the plasma medium mentioned above.
Neglecting damping, show that index of refraction of the plasma is given by:
n =
_
1

2
p

2
where
p
is the plasma frequency and is the angular frequency of the electromagnetic wave.
(c) (10 pts) In the Earths ionosphere the typical free electron density is 10
5
cm
3
. For which
type of signals does the ionosphere more transperent to TV (10
9
Hz) or AM (10
5
Hz) waves?
Discuss it.

0
=8.8510
12
C
2
/N.m
2
, e=1.610
19
Coul., m
e
=9.3110
31
kg.
6. A photon of energy 214 MeV undergoes a Compton collision with an electron at rest, and the
scattered photon makes an angle of = 60

with the +x axis.


(Hint: - = (h/mc) ( 1- cos ))
(a) (10 pts) Determine the energy of the scattered photon, and argue if this photon can create a
pair (e

and e
+
) or no?
(b) (10 pts) Calculate the kinetic energy of the scattered electron, and after justifying that it is
relativistic or nonrelativistic, determine its speed in terms of c.
(c) (10 pts) Find the de Broglie wavelength of the scattered electron.
c ODT

U Physics Department. (May 2010) 43


(d) (10 pts) If the scattered photon falls upon a metal surface that has a work function of 5 eV,
determine what will be the kinetic energy of the photoelectron.
(e) (10 pts) Let a beam of electrons having the same kinetic energy as the one scattered in part
(b) pass through a crystal and exhibit an interference pattern. If the separation between
successive maxima in the interference pattern is 6.410
2
radians and the condition for a
constructive interference is
n = 2 d sin(), where n = 1,2,3,...
What is the separation of Bragg planes of the crystal?
h = Planck constant = 6.6310
34
j.s, c = speed of light = 310
8
m/s, m = mass of the electron
= 9.1110
31
kg.
7. (a) (10 pts) Explain thermodynamic equilibrium condition briey. Is this room (you are presently
in) in thermodynamic equilibrium? Why?
(b) (10 pts) Which of energy transfer mechanisms (radiative, convective, conductive) depends on
opacity of stellar material? Explain.?
(c) (10 pts) A stellar atmosphere has a temperature of 28000 K. The full width at half maximum
of SiIV line at 1350

A is measured to be 1.5

A. Is this due to thermal broadening? Assume
the turbulent velocity is zero.
(d) (10 pts) Using the concept of escape velocity estimate the Suns interior temperature. Assume
it is made of pure hydrogen.
(e) (10 pts) Making a rough approximation of hydrostatic equilibrium condition and using equa-
tion of state for an ideal gas obtain the relation between temperature T, total mass M and
radius R of a star.
Boltzmann constant= 1.3810
16
erg/K, Plancks constant=6.6310
27
erg/s, Gravitational con-
stant= 6.6710
8
dyn cm
2
/g
2
, radiation constant= 7.610
15
erg/cm
3
K
4
, mass of electron=
9.110
28
g, mass of proton= 1.6710
24
g, mass of Sun= 210
33
g, radius of Sun= 710
10
cm,
atomic mass of silicon= 24 amu., eective temperature of the Sun = 5800 K, Earth-Sun distance
= 1.510
13
cm.
8. (a) (15 pts) What are the main physical properties of direct and indirect semiconductors and plot
their energy band diagrams by showing each energy level? And how can you dene their band
gap values by means of transmission spectrum?
(b) (20 pts) What are the dierences between n and p-type semiconductors from each other and
metal? Write and express their conductivities, carrier concentrations, mobilities and plot the
energy band diagram by dening each energy level.
(c) (15 pts) Assume you have p-n (or n-p) junction produced by using these semiconductors with
dierent band gap values (one has 1.24 eV and the other one 2.48 eV) are placed on top
of each other and you are making photoresponse measurement which is the measurement of
voltage or current as a function of wavelength of incident light. As result of this measurement,
what could be the shape of the photocurrent versus wavelength graph of this sample? Plot
and explain.
c ODT

U Physics Department. (May 2010) 44


Nov 2009
Nov 2009
ANALYTICAL MECHANICS
1. A particle of mass m slides on a smooth inclined plane of mass M. The inclined plane is also free
to slide on a smooth horizontal surface as shown in the gure.
(a) (15 pts) Using the generalized coordinates (u(t), v(t)), write down the Lagrangian for the
system.
(b) (10 pts) Find the equations of motion.
(c) (15 pts) Solving the equations of motion for the initial conditions that at t = 0, v(0) =
v
0
, u(0) = 0, and v(0) = u(0) = 0, nd the velocity of the inclined plane at the instant the
mass m reaches the horizontal surface.
(d) (10 pts) Discuss the conservation of linear momentum and the total energy of the system.
2. Let Q
1
= q
2
1
, Q
2
= q
1
+ q
2
, P
i
= P
i
(q, p), i = 1, 2 be a canonical transformation in two degrees
of freedom.
(a) (15 pts) Complete the transformation by nding the most general expression for the P
i
.
(b) (10 pts) Find a generating function.
(c) (13 pts) Find a particular choice for the P
i
that will reduce the Hamiltonian
H = (
p
1
p
2
2q
1
)
2
+ p
2
+ (q
1
+ q
2
)
2
to a Hamiltonian K
K = P
2
1
+ P
2
.
(d) (12 pts) Using the canonical equations of the new Hamiltonian K obtain expressions for
q
1
, q
2
, p
1
and p
2
as functions of time and their initial values.
c ODT

U Physics Department. (Nov 2009) 46


Nov 2009
ELECTROMAGNETIC THEORY
1. Two positive point charges q
1
and q
2
are placed on the x axis at points x=L and x=2L, respectively
as shown.
(a) (20 pts) Find the electric potential and the electric led at point P(2L, L, 0).
Suppose now a conducting grounded innite plane is inserted at x=0 while the point charges are
kept at their positions.
(b) (15 pts) Find the electric potential at point P using the image method.
(c) (15 pts) Using Greens theorem, nd the potential at any point in the region x > 0 .
(x) =
1
4
0
_
V
(x

)G(x,x

)d
3
x

+
1
4
_
S

_
G(x,x

(x

)
G
n

_
da

2. A thin, straight conducting wire is centered on the origin, along the z axis and carries a current
I = I
o
cos(wt) everywhere along its length L.
(a) (10 pts) What is the electric dipole moment of the wire?
(b) (20 pts) What is the vector potential everywhere outside the source region? State your choice
of gauge and make no assumption about the size of emitted wavelength of radiation and
the size of wire L.
(c) (20 pts) Find the radiation pattern at far eld zone r >> >> L .
3. An electron is released from rest at a large distance r
0
from a nucleus of charge Ze and then falls
toward the nucleus. Assuming that the electrons velocity is non-relativistic and the radiative
reaction force on the electron is negligible,
(a) (15 pts) Find the angular distribution of the emitted radiation.
(b) (10 pts) What is the polarization of the emitted radiation?
(c) (15 pts) What is the radiated power as a function of the separation between electron and the
nucleus?
(d) (10 pts) What is the total energy radiated by the time the electron is a distance r < r
0
from
the nucleus?
c ODT

U Physics Department. (Nov 2009) 47


Nov 2009
METHODS OF MATHEMATICAL PHYSICS
1. (a) (10 pts) Find the Laurent series expansion of the function
f(z) =
e
z
1 z
for [z[ < 1 .
(b) (40 pts) Evaluate
_

0
cos
2
(x/2)
(1 x
2
)
2
dx
by using complex integration and the Residue Theorem.
2. Consider the second order ordinary dierential equation
x
d
2
y
dx
2
+ (c x)
dy
dx
a y = 0 ,
where c RZ, i.e. c is a non-integer real number, and a R.
(a) (5 pts) Locate and classify the singular points of this dierential equation.
(b) (25 pts) The solution of this dierential equation which is regular at x = 0 and which has
value 1 at that point is called the conuent hypergeometric function, and is usually denoted
by M(a, c; x). Using the method of Frobenius, determine M(a, c; x) as a power series in x.
(c) (20 pts.) Show that the most general solution of this dierential equation is of the form
y(x) = c
1
M(a, c; x) + c
2
x
1c
M(1 + a c, 2 c; x) ,
for arbitrary real constants c
1
and c
2
.
3. Solve the following integral equations.
(a) (25 pts)
e
ax
2
=
_

0
sin(xt)(t)dt,
where a > 0.
(b) (25 pts)
_
1 + x
2
(x) = 1 +
_
x
0
t(t)dt,
where is a constant.
c ODT

U Physics Department. (Nov 2009) 48


Nov 2009
QUANTUM MECHANICS
1. A particle of mass m is in the innite cubic potential well
V (x, y, z) =
_
0 if 0 < x < L, 0 < y < L, 0 < z < L
otherwise
(a) (15 pts) Calculate the energies and the wavefunctions for the ground state and the rst excited
state.
(b) (15 pts) Now, the following perturbation is added
V

= V
0
L
3
(x
L
4
)(y
3L
4
)(z
L
4
) .
Using rst-order perturbation theory, calculate the energy of the ground state.
(c) (20 pts) Using rst-order degenerate perturbation theory, calculate the energy of the rst
excited state.
2. When the Dirac equation, (i

mc)(x) = 0, is written in the form


i

t
= H
D
,
a formulation in the Schrodinger Picture emerges. One can study the theory in the Heisenberg
Picture also, where the state vectors are constant and the observables change in time as
dA
H
(t)
dt
=
i

[H
D
, A
H
(t)] .
By rst explicitly working out the Hamiltonian for a charged particle in an external EM-eld
described by the potentials A

= ( = 0,

A),
(a) (12.5 pts) Compute d
i
/dt in the Heisenberg Picture.
(b) (12.5 pts) Then, show that
H
D
, d
i
/dt = 2ec
ijk

j
B
k
.
(c) (12.5 pts) In energy eigenstates, show that this equation reduces to the following in the non-
relativistic (NR) limit,
_
d

dt
_

e
mc
_


B .
(d) (12.5 pts) Comparing with the corresponding classical (NR) equation describing the dynamics
of angular momenta (d

L/dt = =torque), interpret this equation. What does it say for the
gyromagnetic ratio g, dened via the magnetic moment expression, = ge/(2mc)

S, recalling
that

S = (/2)

, in analogy with the NR case?


c ODT

U Physics Department. (Nov 2009) 49


Hints:

= 2g

I ,

=
_
0
0
_
,
=
_
0
0
_
,

0
= =
_
I 0
0 I
_
.
The Hamiltonian
H =
p
2
2m
Fx
describes the motion of a particle in one dimension under the eect of a uniform, constant force F.
(a) (5 pts) Compute [H, x] and [H, p].
(b) (10 pts) Compute the Heisenberg operators for position x
H
(t) and momentum p
H
(t). Give
brief comments on the expressions you obtained.
(c) (5 pts) Evaluate the following operator
1
2m
p
H
(t)
2
Fx
H
(t) .
Is it time-dependent? Briey explain your result.
(d) (10 pts) Let [(t)) denote the state of the particle at time t. Let A)
t
be the expectation
value of an observable A under the condition that it is measured at time t. Write down the
expression that is used in evaluating A)
t
i. in the Heisenberg picture,
ii. in the Schrodinger picture.
Show clearly which state is used in each.
(e) (10 pts) Find the time-dependent expectation values x)
t
and p)
t
for this particle in terms
of some initial expectation values.
(f) (10 pts) Find the time-dependent uncertainties p
t
and x
t
at time t.
c ODT

U Physics Department. (Nov 2009) 50


Nov 2009
STATISTICAL MECHANICS
1. (a) (15 pts) Consider a general thermodynamic system with S=S(T,P) and P=P(V,T).
Show that C = C
P
C
V
= T(
S
P
)
T
(
P
T
)
V
.
S: entropy, T: temperature, P: pressure, V: volume, C
P
: the heat capacity at constant pressure,
C
V
: the heat capacity at constant volume.
(b) (20 pts) Show that C = T.V

2

.
: thermal expansion coecient,
1
V
(
V
T
)
P
,
: isothermal compressibility,
1
V
(
V
P
)
T
.
(c) (15 pts) Now consider two experiments in which 2 moles of a 3-dimensional monatomic ideal gas
are heated from temperature T to temperature T + T. In the rst experiment the volume is
kept constant. In the second experiment the pressure is kept constant. In which experiment
more heat is needed to raise the temperature by the given amount T, and by how much?
2. Consider a two-dimensional ideal non-relativistic gas of N spin-1/2 fermions in area A.
(a) (10 pts) Find the Fermi energy.
(b) (10 pts) Find the total energy at T=0

K.
(c) (15 pts) Find the total energy at low temperatures, keeping only the rst correction.
(d) (15 pts) Calculate the chemical potential at low temperatures.
c ODT

U Physics Department. (Nov 2009) 51


Nov 2009
MISCELLANEOUS PHYSICS PROBLEMS
1. The vertical plane surface of a glass (n =

2) quarter sphere is totally illuminated by uniform,
horizontal monochromatic beam of light. The horizontal plane of the quarter sphere is placed on
a grounded glass table for viewing purposes. The radius of the spherical surface is R=10 cm and
is coated with a thin lm such that it is nearly 50% reecting. It is observed that various regions
with dierent illuminations exits on the table.
(a) (35 pts) Describe the illumination (i.e. dark or bright) in the regions OA, AB, BC and to
the right of C. Make necessary calculations and give your reasoning for each case. Find the
length of each region. Note; neglect the refractions due to grounded glass table.
(b) (15 pts) Describe what an observer looking at the spherical surface sees. Explain why.
2. The simple cubic (sc) crystal sample with the lattice constant of a and length of L along each
direction is dened by the direct lattice basis vectors,
a = ai

b = aj
c = ak
The corresponding reciprocal (indirect lattice) basis vectors are

A =
2
a
i

B =
2
a
j

C =
2
a
k
In Free electron model,
(a) (5 pts) write down the equation of motion of an electron,
(b) (5 pts) use the form of a traveling plane wave,

k
(r) = exp(i

k.r)
to derive the energy of the electron,
(c) (10 pts) nd the components of the wave vector

k that satisfy the periodic boundary conditions


of the crystal sample,
c ODT

U Physics Department. (Nov 2009) 52


(d) (15 pts) nd the energies of the electron in the rst four bands, at

k =

0 and along k
x
axis in
the 1
st
Brillouin zone,
(e) (5 pts) plot the rst four bands of the free electron along [k
x
0 0] direction in the reduced
zone scheme.
(f) (10 pts) By considering Bragg reection, discuss the use of the free electron model in the
energy band structure calculations of a crystal sample.
3. The average lifetime of muons at rest is 2.2 sec. A lab measurement on the decay in ight of
muons yield an average lifetime of 6.6 sec.
(a) (10 pts) What is the speed of these muons in the lab frame?
(b) (5 pts) What is the energy of the muons in their rest frame? (Take the rest mass of muon is
m

100MeV/c
2
)
(c) (10 pts) What is the energy of the muons in the lab frame?
(d) (5 pts) What is the kinetic energy of the muons in the lab frame?
(e) (10 pts) What is the momentum of the muons in the lab frame?
(f) (10 pts) How far do they travel before decaying in the lab?
4. (a) (25 pts) A rocket propels itself rectilinearly by emitting radiation (i.e. emitting photons with
velocity c relative to its instantaneous rest-frame) in the direction opposite to its motion. It
continues to do so until it attains a velocity V relative to its initial rest-frame. Find the ratio
of the initial to the nal rest-mass of the rocket.
(b) (25 pts) If one neutron and one pion are to emerge from the collision of a photon with a
stationary proton, nd the threshold frequency of the photon in terms of the rest-mass n of
a proton (or a neutron, assuming their masses to be equal) and that, m, of a pion. [Recall
that the threshold frequency is the minimum frequency that the photon must have so that
the reaction can happen.]
5. (a) (20 pts) A uniform, innite cylindrical plasma with radius a (cylinder oriented along the z-axis)
carries a steady current of I
p
MA. Calculate the magnetic eld at any radius r in magnitude
and direction due to this current. You may assume that the current density,

j =
_
I
p
a
2
_
z, for
r a and vanishes for r > a.
(b) (30 pts) Estimate the gyroradius r
L
of an alpha particle (Helium 4 nucleus) with mass number
A = 4, atomic number, Z = 2 and energy 4 MeV in the Earths magnetic eld (approx. 10
4
T). You can take the rest mass of the alpha particle as 41.6710
27
kg, e=1.610
19
C and
1 MeV=1.610
13
J.
6. Below you will nd 5 questions where each one is worth 10 points. Give brief but complete answers.
No derivation is necessary; if you know the answer you can write it down directly.
(a) Let

S denote the total spin of 3 electrons. What are the possible results if S
2
is measured?
(b) You want to describe the spin-orbit interaction by using the relativistic Dirac equation. Do
you need to modify the Dirac equation to do this? If yes, how would you modify it? If no,
why?
(c) Write down the uncertainty relation for the product L
y
L
z
where

L is an angular momen-
tum operator.
(d) Let

J =

L +

S be the total angular momentum of an electron. Write down the value of the
commutator [J
y
, S
x
].
c ODT

U Physics Department. (Nov 2009) 53


(e) Suppose that a hydrogen atom is placed inside a uniform electric eld. Give an example of
levels for which the degenerate perturbation theory is essential.
7. A particle of mass m moves in a central potential given by V = V
0
_
r
a
_
k
.
(a) (25 pts) Use the Bohr quantization rule for momentum, to calculate the quantized energies of
allowed orbits of the particle.
(b) (15 pts) What value of k corresponds to the hydrogen atom? Find values for V
0
and a (or a
suitable combination of these constants) which give the correct potential for hydrogen.
(c) (10 pts) Use your results from parts (a) and (b) to calculate the energy levels of the hydrogen
atom.
(m
e
= 9.1 10
31
kg, e = 1.6 10
19
C, h = 6.62 10
34
J.s,
0
= 8.85 10
12
C
2
/N.m
2
)
8. Stars are formed by the gravitational collapse of gas clouds.
(a) (25 pts) Derive the quasi-hydrostatic equilibrium equation (ignoring all other force aects like
magnetic and rotation), which is the condition of approximately zero acceleration for a star of
mass M(r), pressure P(r) and density (r) where r is the distance from the center of the star.
Also, nd an expression for the pressure P(r), if we assume a uniform density throughout the
star and the pressure goes to zero at the surface r=R
surface
.
(b) (5 pts) Assuming that stars radiate like a blackbody, determine the surface temperature of a
star with a wavelength of 2.897 10
5
cm at the maximum intensity.
(c) (5 pts) Calculate the energy ux, i.e. energy per unit area per unit time for the star in part
(b).
(d) (5 pts) If the radius of the star is twice the Suns radius, what is the luminosity of the star?
(e) (5 pts) If the star is at a distance of 1.0 light year, what unabsorbed energy ux is measured
at the Earth?
(f) (5 pts) If the star in question rotates about its axis with a period of 814.17 hours, what Doppler
shift due to rotation, one would measure in its spectrum at the wavelength mentioned in part
(b)?
R
sun
= 7 10
10
cm, = 5.7 10
5
erg cm
2
deg
4
s
1
, ly = 10
18
cm,
c = 3 10
10
cm s
1
, M
sun
= 2 10
33
gr, L
sun
= 4 10
33
erg s
1
, G = 6.7 10
8
Dyn.cm
2
/gr
2
9. Assume you have n and p-type semiconductors
(a) (15 pts) What are the dierences of these semiconductors from each other and metal or what
make them dierent from each other and metal ? Write and express their conductivities,
carrier concentrations, mobilities and also plot the energy band diagram by showing each
energy level.
(b) (15 pts) When you construct an n-p junction, explain the ow of charges and the energy band
diagram of this system in detail.
(c) (20 pts) Assume you have p-n (or n-p) junction produced by using these semiconductors
with dierent band gap values (for p-type, E
g
=1.24 eV and for n-type, E
g
=2.48). They are
placed on top of each other and you are making photoresponse measurement which is the
measurement of voltage or current as a function of wavelength of incident light. As a result
of this measurement, what could be the shape of the photocurrent versus wavelength graph
of this sample? Plot and explain.
c ODT

U Physics Department. (Nov 2009) 54


May 2009
May 2009
ANALYTICAL MECHANICS
1. Two blocks, each of unit mass (m
1
= m
2
= 1 unit), are connected to each other by a spring of
spring constant k, and by springs of unit spring constants (k
1
= k
2
= 1 unit) to the walls. They
can move on a smooth surface as shown in the gure:
(a) Write down the Lagrangian for the system in terms of the generalized coordinates x
1
and x
2
.
(b) Obtain the Hamiltonian H. Is H conserved?
(c) Perform a canonical transformation to the center of mass coordinates: That is, let one new
coordinate be the position of the center of mass of the system and let the other new coordinate
be the distance between the two masses. Obtain the explicit form of the Hamiltonian in terms
of the new coordinates and momenta.
(d) Write down the Hamilton-Jacobi equation and nd the natural frequencies of the system by
using the method of action-angle variables.
2. A disk of mass m and radius a (moment of inertia I
cm
=
1
2
ma
2
) rolls without slipping inside a
xed cylinder of radius b (b > a). Assume that the motion takes place on the plane of the paper
and there is a uniform gravitational eld in the direction indicated in the gure:
(a) How many degrees of freedom does the system have?
(b) Determine the constraint equations.
c ODT

U Physics Department. (May 2009) 56


(c) Write down the Lagrangian for the system and nd the frequency of small oscillations at the
lowest position.
(d) Using the method of Lagrange multiplier, nd the forces of the constraints. What is the
physical interpretation of these forces?
(e) What is the total energy of the system? With what minimum speed should the center of the
disk start at the bottom of the cylinder in order to go completely around without leaving the
surface?
3. A particle of mass m moves on the smooth, inner surface of the paraboloid of revolution x
2
+y
2
= bz,
where b is a positive constant. A uniform gravitational eld acts in the negative z-direction.
(a) Write down the Lagrangian for the system in terms of the cylindrical coordinates r, , z.
(b) Obtain the equations of motion and reduce the problem to an equivalent one dimensional
problem.
(c) Calculate the eective potential energy and discuss the nature of the motion.
(d) Prove that the particle will describe a horizontal circle in the plane z = constant provided
that it is given an angular frequency w =
_
2g/b.
(e) Calculate the period of small radial oscillations, that is, the period of the motion when the
particle is slightly disturbed from this circular orbit.
c ODT

U Physics Department. (May 2009) 57


May 2009
ELECTROMAGNETIC THEORY
1. (a) Write Maxwell Equations in dierential forms for free space and explain each in a statement.
Start from one or two of Maxwell Equations to answer the following parts.
(b) Derive Gauss law :
_
S

E da =
q
en

0
. Using Gauss law, nd the electric eld

E everywhere
due to a charge distribution (x) = Ar for r R and 0 elsewhere. Here, r is the spherical
coordinate and A is a constant.
(c) Derive Amperes law:
_
S

B d

l =
0
I
enc
for a steady current. Using Amperes law nd the
magnetic eld

B everywhere for a surface current density on xy plane and of innite extend
given as :

k = k
0
( x + y).
(d) Show that the magnetic eld can be written in terms of a gradient of a scalar function for the
regions where there is no current source (including the displacement current). For a current
loop, this scalar function can be written as
m
=

0
I
4
, where is the solid angle subtended
by the loop surface at the eld point. Use this fact to calculate the scalar function
m
and the
magnetic eld

B on the symmetry axis of a circular current loop. (You may use the cylindrical
coordinates.)
(e) Show that the dynamic magnetic and electric elds vectors can generally be written in terms
of a scalar and a vector eld.
You may use:
_
V


AdV =
_
S

A da
_
S


A da =
_
C

A d

l
d =
da r
r
2
2. An incident electromagnetic wave in air (index of refraction n
1
= 1) is reected and refracted at
the interface of a glass (n
2
= 2 ) as shown in the Figure. The polarization of the incident wave is
parallel to the plane of incidence (the x-z plane in the gure).
(a) Using Maxwell equations derive and write the boundary conditions at the interface.
(b) Find the reected and transmitted electric elds in terms of incident elds (That is the Fresnel
equations).
(c) Find the critical angle at which the reected wave is completely extinguished (Brewsters
angle). (Take
1
=
2
=
0
).
c ODT

U Physics Department. (May 2009) 58


(d) Find the transmission and reection coecients for the transmitted and reected energies.
3. If an incident monochromatic wave is scattered from a system whose individual dimension is small
compared with its wavelength then the scattered (radiated) electric eld can be expressed in terms
of electric and magnetic dipole moments ( p and m) as follows:
(a) Using the scattered electric eld nd the dierential scattering cross section per unit solid
angle.
(b) If the incident linearly polarized wave is scattered by a point charge q nd the dierential
scattering cross section per unit solid angle.
(c) Find the total scattering cross section.
c ODT

U Physics Department. (May 2009) 59


May 2009
METHODS OF MATHEMATICAL PHYSICS
1. Laguerres equation is given by (L+n)y(x) = 0, where n N and the second order linear dierential
operator L reads
L = x
d
2
dx
2
+ (1 x)
d
dx
.
Clearly L is not a self-adjoint operator since it is not of the form
L
s
=
d
dx
_
p(x)
d
dx
_
+ q(x) ,
for some functions p(x) and q(x).
(a) (10 pts.) Find a suitable function (the so-called weighting function) (x) such that Laguerres
equation can be rewritten as
_
L
s
+ n(x)
_
y(x) = 0 , (1)
where L
s
is self-adjoint. What are the functions (x), p(x) and q(x) in (1)?
(b) (10 pts.) Using your answer to part a), justify with some simple arguments that the solutions
of (1), i.e. the Laguerre functions L
n
(x), are orthogonal on a suitable interval (a, b) R with
respect to the weighting function (x), thus
_
b
a
L
n
(x) L
m
(x) (x) dx = c
n

nm
, where n, m N,
for some constant c
n
. What are the values of a and b? How does one determine their values?
(c) (10 pts.) From a dierent perspective, one can think of the Rodrigues formula
L
n
(x) =
e
x
n!
d
n
dx
n
_
x
n
e
x
_
as dening the Laguerre functions. Show that the Laguerre functions can be expanded in a
series as
L
n
(x) =
n

m=0
(1)
m
n!
(n m)! (m!)
2
x
m
.
(d) (20 pts.) Using the Rodrigues formula given in part c), nd the value of the constant c
n
in
part b).
2. (a) Let f(z) be analytic in some region of the complex plane. [It follows that the real and
imaginary parts of f(z) satisfy the Cauchy-Riemann (CR) conditions in that region.] Let
u(r, ) and v(r, ) be the real and imaginary parts of f(z), respectively, written in polar
coordinates.
i. (15 pts.) What are the CR conditions that u and v satisfy in polar coordinates?
ii. (10 pts.) Calculate the derivative df/dz in terms of u(r, ), v(r, ) and their derivatives
with respect to r.
(b) (10 pts.) Find the Laurent expansion of the function z/(z 1) about the origin for its entire
region of analyticity.
c ODT

U Physics Department. (May 2009) 60


(c) (15 pts.) Let C be the circle [z[ = 3. Evaluate
_
C
e
z
z
2
(z i)
dz .
3. (a) (15 pts) Find the Greens function for
Ly(x) (
d
2
dx
2
k
2
)y(x), (2)
with the boundary conditions y(0) = y() = 0. Here k is a real number.
(b) (15 pts) Using this Greens function, solve
_
d
2
dx
2
k
2
_
y(x) = e
x
. (3)
What happens in the k = 1 case?
(c) (15 pts) With the boundary conditions given as above (namely, y(0) = y() = 0), nd the
integral I whose extremization would give (3).
(d) (5 pts) Suppose neither y(0) nor y() is prescribed (that means y(x) at the boundaries is not
xed),then what are the boundary conditions that one should impose for I to be a minimum
or a maximum?
c ODT

U Physics Department. (May 2009) 61


May 2009
QUANTUM MECHANICS
1. (a) Consider the eigenvalue equation in the quantum-mechanical formalism. What is the physical
signicance of the operator, of the eigenket, of the eigevalue? What is the physical signicance
of an expectation value?
(b) Show that the Hamiltonian of the simple harmonic oscillator H =
p
2
2m
+
1
2
mw
2
x
2
can be
written in terms of the operator a =
_
mw
2
(x + i
p
mw
) and a
+
as H = w(a
+
a +
1
2
).
(c) Consider the eigenvalue equation N[n) = n[n) for the operator N = a
+
a. Show that a[n) =

n[n 1), a
+
[n) =

n + 1[n + 1).
(d) Obtain the eigenvalues and eigenkets of N, hence those of H.
(e) A simple harmonic oscillator is in a state for which a measurement of energy would yield
either
1
2
w or
3
2
w, each with a probability of one-half. A measurement of the momentum of
the particle at t = 0 gives a mean value which is as large a positive value as can be obtained
subject to the above energy condition. Calculate p) and H) as functions of time.
2. (a) Starting from the free equation, construct the Dirac equation for a charged particle in an
electromagnetic eld described by A

= (,

A).
(b) Discuss the electromagnetic gauge invariance in terms of the A

, by noting that the electric and


magnetic elds are the components of the 2nd rank antisymmetric tensor F

.
(c) Demonstrate the gauge covariance of the Dirac equation. (Covariance here means that the
equation has the same form in all gauges. This further means that when you replace A

by
A

= A

f(x, t) and (x, t) by

= U, the equation has the same form.)


(Hint: First show that U is unitary. What is the physical reason for this? Then, parameterize
U as U = exp(ig(x, t)) with g being a real function. Then determine g(x, t) in terms of f(x, t).)
(d) Work out the probability current and check whether it is gauge invariant.
(e) Repeat the same problem, that is the demonstration of gauge covariance, for the Klein-Gordon
equation.
3. An electron with charge e (e > 0) and mass m is moving under the eect of a uniform magnetic
eld,

B = Bz, which is directed along positive z-axis (B > 0). In this question, you will compute the
time dependence of the expectation value of electrons velocity. (You may want to use = eB/mc
in some of your expressions.)
(a) Choose a particular gauge (any gauge you want) and write down the vector potential and the
Hamiltonian. Find the expectation values of the position and momentum operators x) =
[x[) and p) = [p[) in that state. Express these in terms of and its complex conjugate

.
Hint:
H =
1
2m
_
p
(e)
c

A
_
2
.
(b) If you wanted to dene a velocity operator, v = v
x
x+v
y
y+v
z
z, for this system, how would
you dene it? Justify your answer.
(c) Compute the commutators [v
x
, v
y
], [v
x
, v
z
] and [v
y
, v
z
].
(d) Find the rate of change of the time-dependent expectation values v
x
)
t
, v
y
)
t
and v
z
)
t
.
(e) Solve the equations in part (d) to express v
x
)
t
, v
y
)
t
and v
z
)
t
in terms of some initial
expectation values.
c ODT

U Physics Department. (May 2009) 62


May 2009
STATISTICAL MECHANICS
1. N 1 molecules are arranged along a straight line to form a chain of polymer. Each molecule can
be in state a or in state b. In the a state, the energy of the molecule is E
a
(E
a
> 0) and the length
of the molecule is a. In the b state, the energy of the molecule is E
b
= E
a
and the length of the
molecule is b = a/2. Assume that the polymer chain always remains straight. Let E be the total
energy of the polymer chain. k
B
denotes the Boltzmann constant.
(a) Find the temperature at which N
a
/N = 1/3.
(b) What is the length of the polymer chain at temperature T = E
a
/k
B
? (Take e = 2.718)
(c) What is S (the entropy of the polymer chain) when its total energy is equal to E? Express S
in terms of k
B
, (E/E
a
) and N. Do not apply Stirlings approximation. State any assumptions
if necessary.
(d) Calculate the entropy of the polymer chain for N = 10
18
and E/E
a
= (1/2) 10
18
. Express
your answer in terms of k
B
. (Take ln3 = 1.1 and ln4 = 1.4)
2. Consider a classical ideal gas of N particles in a box of volume V .
(a) Find the Canonical partition function Q
N
in terms of V , N and the thermal wavelength

T
= h/

2mkT.
(b) From this result for Q
N
, nd the Grand Canonical partition function Z(, V, T).
(c) Find the entropy S, pressure P, the number of particles N in terms of , T, and V . Find the
equation of state for the gas.
(Possibly useful information :
_
4 p
2
e
p
2
/2mkT
= (2mkT)
3/2
, = kT ln Z.)
3. Consider a gas of non-interacting bosons, each of mass m which are free to move within a volume
V . Find, in the very low temperature limit:
(a) The energy U of the system in terms of temperature T,
(b) heat capacity C of the system in terms of temperature T.
(c) The heat capacity is that of the nearly degenerate (kT <<
F
) electron gas and has the form
C
V
= V T
n
where V is the volume and is a constant,
(d) The surface of the star is a perfect absorber of radiation at all frequencies.
Discuss why it is appropriate at low temperatures to put the chemical potential to zero.
Put all integrals in dimensionless form but do not evaluate them.
c ODT

U Physics Department. (May 2009) 63


Nov 2008
Nov 2008
ANALYTICAL MECHANICS
1. (a) By any method you choose, show that the following transformation
x

= xcos wt + y sin wt,


y

= xsin wt + y cos wt,


z

= z,
p
x
= p
x
cos wt + p
y
sin wt,
p
y
= p
x
sin wt + p
y
cos wt,
p
z
= p
z
,
is canonical where

x

= (x

, y

, z

), x = (x, y, z) are both rectangular cartesian coordinates


and w = (0, 0, w) is the constant angular velocity.
(b) Obtain a generating function of the type F
2
= F
2
(x, y, z, p
x
, p
y
, p
z
, t).
(c) The Hamiltonian of a particle of mass m in a conservative potential V = V (x, y, z) is
H =
1
2m
(p
2
x
+ p
2
y
+ p
2
z
) + V (x, y, z).
Find the new Hamiltonian H

under this transformation.


(d) What is the physical signicance of the new Hamiltonian? Is it a constant of motion? Derive
the new canonical equations of motion and identify physically each of the terms that occurs
in the equations.
2. Two masses m
1
and m
2
move in an attractive central force potential
V ([ r
1
r
2
[) =
1
2
k[ r
1
r
2
[
2
where k is a positive constant, and r
1
r
2
= r is the relative position vector between them.
(a) Without using any formulas, explain briey why the motion will be conned to a plane.
(b) Without using any formulas, explain briey how you could reduce the problem to an equivalent
one-body problem involving a reduced mass ,
=
m
1
m
2
m
1
+ m
2
under a force kr.
(c) Sketch the potential energy V (r), the centrifugal potential energy V
cf
(r) and the eective
potential energy V
eff
(r). Discuss the qualitative nature of the orbits for dierent values of
the total energy.
(Consider the angular momentum l as a known, xed constant.)
(d) Find the condition under which the two particles can circle each other with a constant r = r
0
.
(e) What is the expression for the minimum value of the total energy in terms of the given
parameters?
c ODT

U Physics Department. (Nov 2008) 65


(f) Find the frequency of small radial oscillations about the circular orbit if the particles are
disturbed a little from the equilibrium r
0
.
3. A massless string is suspended vertically from a xed point and the other end is wrapped several
times around a uniform cylinder of mass m and radius R. At time t = 0, the cylinder is dropped
from the rest and rotates without slipping as the string unwinds.
(a) How many degrees of freedom does the system have?
(b) Write down the Lagrangian for the system.
(c) Obtain the equation of motion and nd the linear and angular accelerations of the cylinder.
(d) Determine the velocity of the point O and the speeds of the points A and B.
(e) Find the forces of constraint and give your physical comments.
Note: The moment of inertia for a uniform cylinder is I =
1
2
mR
2
.
c ODT

U Physics Department. (Nov 2008) 66


Nov 2008
ELECTROMAGNETIC THEORY
1. A thin insulating rod is placed on z-axis from z = a to z = +a. If the linear charge density on
the rod is given as =
0
(constant), nd
(a) the electric potential at any point in space, and
(b) the electric potential on the x- and z-axes .
If the linear charge density on the rod is a function of z, nd an approximate expression for the
potential at large distances by calculating the leading term (rst non-zero term) in the multiple
expansion for
(c) =
0
cos(z/2a), and
(d) =
0
sin(z/a).
P
0
(x) = 1 , P
2
(x) =
1
2
(3x
2
1)
P
1
(x) = x , P
3
(x) =
1
2
(5x
3
3x) .
_
du
(u
2
+ c
2
)
1/2
= ln
u + (u
2
+ c
2
)
1/2
c
.
For an axial charge distribution (z

):
(x) =
1
4
0

n=0
_
P
n
(cos )
r
n+1
__
+a
a
z
n
(z

) dz

2. Consider a linear radio antennas with dipole moment p oscillates with a frequency . It is placed
at a distance h from an innite perfectly grounded conducting plane. The antenna is parallel to
the conducting plane as shown in the gure.
(a) Find the electromagnetic eld at far eld zone (r >> ).
(b) Find the emitted power per solid angle at far eld zone.
Hint: The vector potential of a dipole at far eld zone is given as

A =
i
0

4
p
e
ikr
r
.
3. An observer in the laboratory observes a beam of electrons with density , moving at velocity
v = v

k. If the beam has circular radius R nd repulsive force acting on an electron inside the
c ODT

U Physics Department. (Nov 2008) 67


beam (r<R) both with respect to the rest frame of the electrons and with respect to the observer
in the laboratory frame, respectively.
Hint: Transformation of electromagnetic elds with respect to laboratory frame is given as

=
_

E +
v
c


B
_


2
+ 1
v
c
_
v
c


E
_

=
_

B
v
c


E
_


2
+ 1
v
c
_
v
c


B
_
.
c ODT

U Physics Department. (Nov 2008) 68


Nov 2008
METHODS OF MATHEMATICAL PHYSICS
1. The beta function is dened as
B(m + 1, n + 1) =
_
1
0
t
m
(1 t)
n
dt =
(m + 1) (n + 1)
(m + n + 2)
=
m! n!
(m + n + 1)!
for Re(m) > 1 and Re(n) > 1.
(a) (5 pts.) Show that
_
1
0
t
z
(1 t)
z
dt =
(z!)
2
(2z + 1)!
for Re(z) > 1 .
(b) (15 pts.) Let t = (1 + s)/2 on the left hand side of the equality in part (a). Show that
z!
(2z + 1)!
=
1
2
2z+1
(1/2)!
(z + 1/2)!
.
(c) (5 pts.) Recall that the gamma function is also called as the generalized factorial function
and one has (z) = z!/z. Moreover (1/2) =

. Given the identity z! (z)! = z/ sin (z),


nd (1/2)! .
(d) (5 pts.) Show that z! (z + 1/2)! = 2
(2z+1)

(2z + 1)! .
(e) (20 pts.) The power series expansion for the Bessel functions of the rst kind of integral order
is given by
J
n
(x) =

s=0
(1)
s
s!(n + s)!
_
x
2
_
n+2s
.
The spherical Bessel functions are dened as j
n
(x)
_
/(2x) J
n+1/2
(x). Find a power series
representation of j
n
(x).
2. (a) (10 pts.) Expand the function f(z) = 1/(1 z) in powers of z + 1 for 2 < [z + 1[.
(b) (20 pts.) Let x = Re(z). Compute
_
|z|=r
xdz
in two separate ways: i) By parametrizing the contour suitably, and ii) by observing that
x = (z + z)/2 and noting that z z = r
2
on the circle of integration.
(c) (20 pts.) Evaluate the following integral using residue calculus:
_
/2
0
cos
4
d .
Explain and verify the steps you take.
3. In D dimensions Fourier Transform (FT) of a function (r) is dened as
(

k) =
1
(2)
D/2
_
d
D
r (r) e
i

kr
, (1)
and the inverse FT follows as
(r) =
1
(2)
D/2
_
d
D
k (

k) e
i

kr
. (2)
c ODT

U Physics Department. (Nov 2008) 69


(a) (7 pts) Use equations (1) and (2) to evaluate the following integral
I =
_
d
D
k e
i

k(r

)
. (3)
(b) (8 pts) If (

k) is the FT of (r), what is the FT of


2
(r) ?
[Here
2
=



. Note that you need to derive the expression, not just write it. Assume
that as [r[ , (r) 0 sucently fast.]
(c) (15 pts) From now on choose D = 3, If
(

k) =
1
(2)
3/2
1

k
,
Find (r).
Hint: Use spherical polar coordinates.
(d) (15 pts) Using the information that you got in the previous parts, solve the 3-dimensional
equation.

2
(r) = (r)
[Note, your nal answer should be an integral that has (r) in it.]
(e) (5 pts) In part (d), carry out the Integral for (r) =
3
(r

).
c ODT

U Physics Department. (Nov 2008) 70


Nov 2008
QUANTUM MECHANICS
1. Let

S be the spin operator of a spin 1/2 particle. Let n be a unit vector with spherical angles
and (i.e., n = sin (cos + sin ) + cos

k). Let S
n
=

S n = S
x
n
x
+ S
y
n
y
+ S
z
n
z
denote the
component of the spin along n. Use the 2 2 matrix representation of spin operators.
(a) Find the eigenvectors of S
n
. (These are the spin up,
n
, and spin down,
n
, states along
n.)
(b) Suppose that the particle is in state
n
. Find the expectation values of S
x
), S
y
) and S
z
).
Express
_

S
_
in a short form.
(c) Let m be another unit vector with spherical angles

and

. Consider a Stern-Gerlach
experiment in which S
m
is measured when the particle is prepared in spin up along n state,

n
. Which values can be obtained and what are the probabilities of obtaining each? Check
that these probabilities depend only on the angle between n and m.
Hints:

x
=
_
0 1
1 0
_
,
y
=
_
0 i
i 0
_
,
z
=
_
1 0
0 1
_
.
(1 + cos ) = 2 cos
2

2
, (1 cos ) = 2 sin
2

2
, sin = 2 sin

2
cos

2
.
2. (a) Consider the Hamiltonian H = H
0
+V (t). The state ket can be expanded in the Schrodinger
picture as [ (t) >=

n
c
n
(t)e

E
n
t
[ n > where H
0
[ n >= E
n
[ n >. At t=0 when V(t)
is turned on only the state [ i > is populated, that is c
n
(0) =
ni
. Assuming a perturbation
expansion of the form c
n
(t) = c
(0)
n
(t) + c
(1)
n
(t) + ..., show that to rst order
c
(1)
m
(t) =
1
i
_
t
0
dt

< m [ V (t

) [ i > e
i

(E
m
E
i
)t

.
(b) A hydrogen atom in its ground state [ nm >=[ 100 > is placed between the plates of a
capacitor. A time-dependent but uniform electric eld is applied as follows:

E =
_
0 t < 0
(E
0
z)e
t/
t > 0
Using the rst order result in (a) calculate the probability for the atom to be found at t
in each of the three 2p states [ nm >=[ 211 >, [ 210 >, [ 21 1 >. Do not evaluate the radial
integrals but perform all other integrations.
(Note Y
10
=
_
3
4
cos =
_
3
4
z
r
)
3. Consider an isotropic 3 dimensional charged harmonic oscillator with charge q described by the
Hamiltonian
H =
1
2m
p
2
+
1
2
m
2
r
2
.
(a) Are there degeneracies? What are the degeneracies of a general energy level ?
c ODT

U Physics Department. (Nov 2008) 71


(b) This oscillator is now exposed to a uniform magnetic eld

B . Work out the Hamiltonian and
identify the perturbation term, for weak magnetic eld, and express it in terms of the creation
and annihilation operators a
i
, a
+
i
, where i = x, y, z.
(c) Assuming that the perturbation term is small compared to , calculate to lowest order in
perturbation theory, the shifts in the energies of the ground state, and the rst excited state,
using the creation-annihilation operator formalism.
c ODT

U Physics Department. (Nov 2008) 72


Nov 2008
STATISTICAL MECHANICS
1. Consider a two-dimensional Fermi gas.
(a) Find the Fermi energy E
F
in terms of the mean areal density n = N/A.
(b) Find the temperature dependence of the chemical potential in terms of the Fermi energy
E
F
.
(c) One denition of the crossover from the quantum to the classical regimes as temperature is
increased is that becomes negative. Find the temperature at which this happens.
2. Consider a system of N non-interacting spin 1 particles with magnetic moments. These spins are
in a magnetic eld so that each spin has magnetic energy -E when aligned with the eld, +E when
opposite to the eld and 0 when neither happens (perpendicular), E > 0. You can take particles
to be at xed positions.
(a) Without elaborate calculation; nd the energy U and the entropy S at very low temperatures,
T0. Also, nd the energy and entropy at very high temperatures, T.
(b) What is the canonical partition function of this system?
(c) Find the Helmholtz Free energy. entropy and energy of this system at temperature T.
3. A large number (M) of molecules are located in space and are in equilibrium at temperature T (in
K). Every molecule in the system has two non-degenerate states at energies E
0
= 0 and E
1
= .
Let P
n
be the probability of nding a molecule at the n
th
energy level (n=0 and n=1). The plot
shows P
1
versus = (k
B
T)
1
. k
B
is the Boltzmann constant. Take = 40 eV
1
at T=300 K.
Use this plot to answer the following questions.
(a) (15 pts) What is the value of in meV ? (up to 1 signicant digits)
(b) (10 pts) What is the fraction of the molecules in the excited state at T=300K ?
(c) (10 pts) What is the average energy of the sample at = 0.4 meV
1
?
(d) (15 pts) What is the heat capacity of the sample (at constant volume) at = 0.7 meV
1
.
Express in terms of k
B
.
c ODT

U Physics Department. (Nov 2008) 73


May 2008
May 2008
ANALYTICAL MECHANICS
1. A particle of mass M moves in an axially symmetric potential V (
_
x
2
+ y
2
, z).
(a) Consider the canonical transformation induced by the generating function
F = (xcos t y sin t)P
1
+ (xsin t y cos t)P
2
+ zP
3
, = constant
and use it to transform the Hamiltonian
H =
1
2M
(p
2
x
+ p
2
y
+ p
2
z
) + V (
_
x
2
+ y
2
, z)
(b) Is the new Hamiltonian a constant of motion? Is it the energy?
(c) What is the physical signicance of the new Hamiltonian?
(d) Derive the new Hamiltonians equations of motion and identify physically each of the terms
occuring in the equations.
2. A double pendulum with equal lengths and dierent masses m
1
and m
2
performs small oscillations
in a plane. Introduce the transverse displacements of the rst particle from the vertical
1
and of
the second particle from the rst particle
2
.
(a) Find the Lagrangian of the system .
(b) Construct mass matrix / and potential matrix 1, and derive normal-mode frequencies (for
customary denote =
_
m
2
m
1
+m
2
).
(c) Construct the normal mode eigenvectors and describe the motion. Show that these represent
the expected behavior for large and small values of m
1
/m
2
.
(d) Find the explicit form of the modal matrix / and show that it diagonalizes the matrices /
and 1.
(e) Construct the normal coordinates.
(f) Assume that m
2
<< m
1
. If the upper mass is displaced slightly from the vertical and released
from rest, show that subsequent motion is such that at regular intervals one pendulum is
stationary and the other oscillates with the maximum amplitudes.
Hint: The modal matrix / is constructed from the normal mode eigenvectors in the following
way:
/ = [
1
,
2
] .
3. A particle with mass m in one-dimensional space in the presence of the potential energy eld V (q)
in existence by a damping eld 2m q.
(a) Find the Lagrangian of the system.
(b) Find the Hamiltonian.
(c) For the generating function F
2
(q, P, t) = e
t
qP, nd the transformed Hamiltonian K(Q, P, t).
(d) For the harmonic oscillator potential V = m
2
q
2
/2, calculate
dK
dt
and nd time dependence
of Q.
(e) Obtain the solution q(t) for the damped oscillation from the result of the part (d) in the case
that < .
c ODT

U Physics Department. (May 2008) 75


May 2008
ELECTROMAGNETIC THEORY
1. The potential on the surface of a sphere having radius R is given as () = V
0
cos
2
, where V
0
is
constant and is the polar angle. A point charge Q is placed at the center of the sphere (to the
origin).
(a) Find the electric potential everywhere using the solution of Laplace equation in spherical
coordinates.
(b) Find the charge density on the surface of the sphere.
You may use:
P
0
(x) = 1 , P
1
(x) = x , P
2
(x) =
1
2
(3x
2
1) , P
3
(x) =
1
2
(5x
3
3x) .
(x) =

m
m

n=m
(A
n
cos n + A

n
sin n)(B
m
r
m
+ B

m
r
(m+1)
)P
n
m
(cos )
2. (a) Write the Maxwell equations on the covariant form in terms of eld strength tensor.
(b) Under the Lorentz condition derive the four dimensional wave equation.
(c) Derive the Greens function D(x, x

) for the equation


x
D(x, x

) =
4
(x x

),
where
4
(x x

) = (x
0
x

0
)(x x

) is a four dimensional delta function.


(d) Show that 4-vector potential for the radiation elds can be written as
A

(x) =
4
c
_
d
3
xD(x x

) J

(x

)
where J

= (c,

J).
(e) If the charged particle is moving in arbitrary path write the charge and current density in
terms of charges 4- velocity.
(f) Derive Lienard-Wiechert potentials for a moving point charge.
Hint:
[f(x)] =
i
(x x
i
)
[
_
df
dx
_
x=x
i
[
3. Two thin, parallel, innitely long, nonconducting rods, a distance d apart from each other are
carrying identical constant charge density per unit length in their rest frame. If they move with
velocity v parallel to the direction of the wire calculate the force per unit length between them in
a frame of reference that is at rest, and in a frame of reference moving with rods and compare your
results.
Hint:

=
_

E +
v
c


B
_


2
+ 1
v
c
_
v
c


E
_

=
_

B
v
c


E
_


2
+ 1
v
c
_
v
c


B
_
c ODT

U Physics Department. (May 2008) 76


May 2008
METHODS OF MATHEMATICAL PHYSICS
1. Using complex integral theorems evaluate the following integrals:
(a)
I =
_

0
dx
1 + x
5
.
(b)
I =
_
2
0
sin
2
d
a + b cos
, a > b > 0.
(c)
I =
_

0
x
2
dx
(a
2
+ x
2
)
3
, a > 0.
2. Derive two basic recursion relations for the polynomials dened by the generating function
1
1 2xt + t
2
=

n=0
u
n
(x) t
n
, [x[ < 1 , [t[ < 1 .
3. A string has its endpoints clamped at x = 0 and x = . The small vibrations of the string are
described by the wave equation

1
v
2

2
t
2
u(x, t) +

2
x
2
u(x, t) = 0 ,
subject to the initial conditions
u(x, 0) = f(x) and

t
u(x, t)

t=0
= g(x) ,
where f(x) and g(x) are smooth functions in x [0, ]. Find the solution u(x, t).
c ODT

U Physics Department. (May 2008) 77


May 2008
QUANTUM MECHANICS
1. Consider a Hydrogen-like atom in an external Electromagnetic (EM) eld. We can take the EM
eld to be a monochromatic plane wave

A = A
0
cos(

c
n x t) where the unit vectors and n
represent the linear polarization and the propagation directions, respectively.
(a) Work out the Hamiltonian of the atomic electron and identify the time dependent interaction
term V (t), H = H
0
+ V (t), with V (t) = 1 e
it
+ 1
+
e
it
, for the case where

A
2
- term can
be eliminated.
(b) Electric dipole approximation is based on the fact that the wavelength of the radiation eld
is far larger than the atomic dimensions, that is, the EM eld is uniform (does not change)
over the atomic scale. Show that this condition is held to order so that the interaction term
can be simplied substantially to this order. Work out the simplied form of the interaction
term 1
+
, describing the absorption.
(c) In the regime described in (b), you can take the vector potential describing the monochromatic
plane waves as

A = A
0
cos( t), by dropping the position dependent term. Show that

A
2
term, in the time-dependent Shrodinger equation i

t
= H , can be eliminated by the
following transformation:
(x, t)

(x, t) = (x, t) exp


_
i

e
2
2mc
2
_
t
0
dt


A
2
(t

)
_
.
That is, you get the following Shrodinger equation:
i

(x, t)
t
=
_
H
0
+
ie
mc

(x, t) .
(d) Next we perform a further gauge transformation with the gauge function
(x, t) =

A(t) x:
/(x, t)

(x, t) = /(x, t) exp


_
i

e
c
(x, t)
_

= +
1
c

t
,

A

A

=

A

.
Show that the new (transformed) potentials are

A

= 0, and

E x.
Recall:

E =


1
c


A
t
,

B =


A.
Thus, the new time-dependent Shrodinger equation for

becomes;
i

t
= (H
0
+ V (t))

,
with V (t) =

d, where

d = ex is the electric dipole operator of the one-electron Hydrogen-like
atom.
Note: This is the reason why the approximation in which the EM eld is taken to be uniform over
the atomic scale is called electric dipole approximation.
c ODT

U Physics Department. (May 2008) 78


2. (a1) Construct the continuity equation for the free Klein-Gordon equation, and identify the prob-
ability and current densities, and discuss your results.
(a2) Work out the non-relativistic (NR) limits of these quantities (probability density and current
density) for the positive energy case. Do they agree with the corresponding quantities of the
free Shrodinger theory ?
(b1) Construct the continuity equation for the free Dirac equation, and identify the probability
and current densities, and discuss your results.
(b2) Work out the non-relativistic limits of these quantities for the positive energy case. Do they
agree with the corresponding quantities of the free Shrodinger theory ?
3. Consider a simple harmonic oscillator with the following Hamiltonian
H =
p
2
2m
+
1
2
m
2
x
2
.
Let [n) (n = 0, 1, 2, . . .) represent the usual eigenstates of H. For any complex number we dene
the following state
[) = e
||
2

n=0

n!
[n) .
These states are called coherent states.
(a) Find the expectation values of the position and momentum operators x) = [x[) and
p) = [p[) in that state. Express these in terms of and its complex conjugate

.
(b) Find x and p for that state. Do these uncertainties depend on ? Check that the uncer-
tainty relation is satised.
(c) Suppose that the oscillator is initially in a coherent state [(t = 0)) = [
0
) where
0
is a real
number. Find the state [(t)) at time t. Is this a coherent state? If so what is the value of
the parameter at time t?
(d) Find the average position and momentum at time t, x)
t
and p)
t
, for the state given in part
(c). [Note: You can use the result in part (a) for this purpose.]
(e) Check that x
t
p
t
satises the uncertainty relation.
Hint:
a =
_
m
2
_
x + i
p
m
_
.
c ODT

U Physics Department. (May 2008) 79


May 2008
STATISTICAL MECHANICS
1. Consider a system of N, identical (each of frequency
0
), non-interacting, one-dimensional simple
harmonic oscillators (1D- s.h.o). Let the ground state energy of each oscillator be zero.
(a) Let E = Mh
0
be the total energy of the system. Prove that the number of states accessible
to the system is given by
(N1+M)!
(N1)!M!
.
(b) Using the entropy-temperature relationship of the microcanonical ensemble, nd the temper-
ature dependence of the average energy of the system.
Answer parts c and d using the system description below.
Consider a closed system C consisting of two subsystems A and B. A and B can exchange
energy with each other. System A has 4 1D- s.h.o.s and system B has 3 1D- s.h.os. Each 1D-
s.h.o is of frequency
0
. Take the ground state energy of each oscillator at zero and suppose
that C has total energy (E
C
) equal to 4h
0
.
(c) Make a table showing the number of states accessible to A, B, and C for every possible
distribution of the total energy (E
C
= E
A
+ E
B
) between the two subsystems.
(d) What is the average energy of A (in the combined system C)?
2. Consider a system with only three single-particle states with energies 0, and ( > 0, that is,
excited states are degenerate). Let there be three identical particles without spin.
Answer parts a, b and c if the particles are bosons.
(a) Write down the canonical partition function.
(b) What is the probability of nding N particles in the single-particle ground state?
(c) What is the average occupation number of the single-particle ground state?
(d) Consider now the extremely high temperature limit, answer parts a, b, and c.
(e) Answer parts a, b, c, and d if the particles are fermions.
3. Cooling of a White Dwarf Star.
Just after a white dwarf star is formed it begins a long slow radiational cooling process which
will eventually reduce it to a cold dark ember. In this problem you will nd its temperature as a
function of time.
You may assume that :
There are no longer any heat sources in the star,
The thermal conductivity is so high that the temperature is essentially uniform throughout
the star,
c ODT

U Physics Department. (May 2008) 80


The heat capacity is that of the nearly degenerate (kT <<
F
) electron gas and has the form
C
V
= V T
n
where V is the volume and is a constant,
The surface of the star is a perfect absorber of radiation at all frequencies.
(a) What is the value of n in the expression for C
V
?
(b) Find the expression for the derivative of the total energy with respect to temperature T;
dE/dT.
(c) Find an expression for the derivative of the total energy of the star with respect to time;
dE/dt.
(d) Find the time evaluation of the temperature.
c ODT

U Physics Department. (May 2008) 81


Nov 2007
Nov 2007
ANALYTICAL MECHANICS
1. Let p and q be canonical variables.
(a) Which of the following transformations are canonical?
i. P = p
2
+ q
2
, Q = p
2
q
2
ii. P = p
2
+ q
2
, Q = 1/2 arctan(q/p)
iii. P = p
2
+ q
2
, Q = pq
(b) Find generating function F
1
(q, Q, t) for the canonical transformation(s) in part (a).
(c) For a harmonic oscillator with the following Hamiltonian
H =

2
(p
2
+ q
2
) (1)
write the equations of motion in the form

P = f(P, Q) and

Q = g(P, Q). You may choose the
second pair of P and Q from the above list.
(d) Solve the equations obtained in part (c) for P and Q.
(e) Find solution for q using the results obtained in part (d).
2. A three particle system consists of masses m
i
and coordinates (x
1
, x
2
, x
3
) as follows:
i mass (x
1
, x
2
, x
3
)
1 3m (a, 0, a)
2 4m (a, a, a)
3 2m (a, a, 0)
(a) Calculate the total mass and the center of mass of this system.
(b) Calculate the inertia tensor of this system with respect to the origin of the coordinate system.
(c) What is the rotational energy T
rot
, if the system rotates around the (1, 1, 1) axis with the
angular velocity .
(d) Find the three principle momenta of inertia of this system. (Hint: One of the principle
momenta is I
1
= 10ma
2
. Verify this)
(e) Find the principle axis corresponding to the principle momentum I
1
= 10ma
2
.
3. Consider two masses m
1
and m
2
connected by a massless spring of force constant k and unstretched
length l
0
, free to slide over a frictionless horizontal table. Assume that r
1
and r
2
are the positions
of m
1
and m
2
relative to the origin of some inertial frame.
(a) Using polar coordinates (r, ) for r, write down the Lagrangian in terms of the radius vector
of the center of mass

R and relative position vector r.
(b) Derive the equations of motion and nd their solutions.
(c) What can you conclude about the motion of the reduced mass ? What are the constants of
motions?
You may need the following:
c ODT

U Physics Department. (Nov 2007) 83


The orbit equation in a central force eld:
d
2
u
d
2
+ u =
F(1/u)
l
2
u
2
(2)
where u = 1/r

_
dx

ax
2
+ bx + c
=
1

a
arccos
_

_
2ax + b

b
2
4ac
__
(3)
c ODT

U Physics Department. (Nov 2007) 84


Nov 2007
ELECTROMAGNETIC THEORY
1. (a) A conducting sphere with radius a is held at constant potential V
0
. Find the potential outside
of this sphere. What is the potential inside?
(b) A conducting spherical shell of radius b has surface charge density =
0
cos where
0
is
constant and is the polar angle. Find the potential inside and outside of this shell.
(c) Now, suppose the conducting sphere (with radius a) of part (a) is placed concentrically in
the spherical shell (with radius b and surface charge density =
0
cos ) of part (b) (a < b).
Find the potentials in the regions i) r < b, and ii) a < r < b.
Some information you may use:
P
0
(x) = 1 P
1
(x) = x P
2
(x) =
1
2
(3x
2
1) P
3
(x) =
1
2
(5x
3
3x)
=

_
A
m
r
m
+ B
m
r
(m+1)
_
P
m
(cos )
_

0
P
m
(cos )P
n
(cos ] sin d =
2
2m + 1

mn

2
=
1
r
2

r
_
r
2

r
_
+
1
r
2
sin

_
sin

_
+
1
r
2
sin

2
2. (a) Inside of a uniformly polarized sphere with radius a and having electric dipole density

P
(polarization), show that the electric eld is given by

E =
1
3
0

P
(b) Vector potential of a spinning spherical shell with radius a, angular velocity and uniform
surface charge density
0
is given as:

A(r) =

0
a
0
3
( r) (1)
where is the angular velocity vector and r is the eld point.
Inside of a uniformly magnetized sphere with radius a and magnetic dipole density

M (mag-
netization), show that the magnetic eld is given by

B =
2
0
3

M. (You may use above infor-


mation.)
You may use:

A =
1
r sin
_

(sin A

)
A

_
r+
1
r
_
1
r sin
A
r



r
(rA

)
_

+
1
r
_

r
(rA

)
A
r

(2)
(c) Outside of an electrically and magnetically polarized matter (with

P and

M), electric and
magnetic eld vectors are given as:

E =
1
4
[3( p r) r p]
r
3
and

B =

0
4
[3( m r) r m]
r
3
(3)
where p and m are the total electric and magnetic dipoles.
c ODT

U Physics Department. (Nov 2007) 85


Using above information and the results of part (a) and part (b), nd that the electromagnetic
momentum (

P
m
=
0
_
(

E

B)dV ) of a uniformly polarized sphere of both electrically (

P)
and magnetically (

M), is given as:
4
9

0
a
3
(

M

P) (a is the radius of the sphere).
Hints: Find the momentum separately for the inside and outside of the sphere and add
[3( p r) r p] [3( m r) r m] = 2( p m + 3 r[ r ( p m)] (4)
You may set z-axis along ( p m) to evaluate the integral.
c ODT

U Physics Department. (Nov 2007) 86


3. A vector potential for magnetic dipole radiation can be expressed as

A(x) =
ik
0
4
( n m)
exp
ikr
r
_
1
1
ikr
_
(5)
where m is magnetic dipole moment.
(a) Using the vector potential given above nd the magnetic and the electric eld for a magnetic
dipole radiation as

H =
1
4
_
k
2
( n m) n
exp
ikr
r
+ (3 n( n m m)
_
1
r
3

ik
r
2
_
exp
ikr
_
(6)

E(x) =
Z
0
k
2
4
( n m)
exp
ikr
r
_
1
1
ikr
_
(7)
where Z
0
=
_

0
/
0
is the impedance of free space.
Hint:

(a

b) =a(

b)

b(

a) + (

b

)a (a

)

b (8)
(b) Show that in the far eld zone the electric and magnetic elds satisfy the typical behaviour
of radiation elds as .
Find the electric and magnetic elds in the near zone kr 1. Interpret the behaviour of elds in
the near zone.
c ODT

U Physics Department. (Nov 2007) 87


Nov 2007
METHODS OF MATHEMATICAL PHYSICS
1. Using the Frobenius method solve the following dierential equation in the interval x [1, 1]
(1 x
2
)
d
2
y
dx
x
dy
dx
+ n
2
y = 0 (1)
(a) Including the end points check the convergence of your solution.
(b) By restricting n to integer values write the solution for the rst four values.
(c) Is the solution set found by restricting n to integer values orthogonal? If not, how can you
make them orthogonal.
2. (a) Using the recursion relation
a
k+2
a
k
=
k(k + 1) l(l + 1)
(k + 1)(k + 2)
(2)
where k = 0, 1, . . . and l = 0, 1, 2, . . . derive the general expression for the Legendre polyno-
mials:
P
l
(x) =
[l/2]

n=0
(1)
n
(2l 2n)!
2
l
(l 2n)!(l n)!n!
x
l2n
(3)
(b) Hermite equation is dened as
H
n
// 2xH
n
/ = 2nH
n
(4)
where n = 0, 1 . . .. First write it in self-adjoint form and then show the orthogonality relation
_

H
m
(x)H
n
(x) exp
x
2
dx = N
mn

mn
(5)
Note: Do not use the Rodriguez formula.
(c) Using the recursion relation
exH
n
2nH
n1
= H
n+1
(6)
evaluate N
m
.
Hint:
_

exp
x
2
=

3. Show that if f(x) is analytic and bounded for [z[ = [x + iy[ < 1, then
f() =
1

_ _
|z|<1
f(z)dxdy
(1 z)
2
(7)
where C and [[ < 1.
Hint: First express the area integral in polar coordinates, then transform one of the integrals to
a suitable line integral of a rational function that can be evaluated using the calculus of residues.
c ODT

U Physics Department. (Nov 2007) 88


Nov 2007
QUANTUM MECHANICS
1. Consider the following Hamiltonian of a 1D oscillator
H =
p
2
2m
+
1
2
m
2
x
2
+ x
3
(1)
where is a small perturbation parameter signifying the deviation of the oscillator from harmonic-
ity.
(a) Compute the energy eigenvalue E
n
of the nth level to the smallest non-zero order of .
(b) Consider a transition between levels n+1 and n. What is the frequency of the photon emitted
(or absorbed) in such a transition? Plot qualitatively the emission or absorption spectrum
(intensity versus frequency graph) of this oscillator.
Hint: Annihilation operator for harmonic oscillator is
a =
_
m
2
_
x +
i
m
p
_
(2)
2. Although one can have quite a good description of Hydrogen atom with the Coulomb Hamiltonian,
one needs to include, for instance, the spin eects for a more realistic description.
(a) One can include the spin eect by taking into account the fact that in the rest frame of the
electron, one has electric elds as well as magnetic elds, although we have only the electric
eld (Coulomb) in the rest frame of the nucleus. Using the general Lorentz transformations
for the

E and

B elds

=
_

E + (



B)
_


2
+ 1



E) (3)

=
_

B (



E)
_


2
+ 1



B) (4)
with

= /c, = (1
2
)
1/2
. Work out the magnetic eld in electron rest frame in the
non-relativistic limit (with terms of order /c retained), and using this, the additional term
in the Hamiltonian, the so-called spin-orbit term H
1
.
(b) Estimate the ratio H
1
)/H
0
in terms of the constants (like etc.) in the eigenstate basis of
H
0
, and show that you can treat H
1
as a perturbation over H
0
.
(c) Discuss the degeneracy issues, and the possible relevant eigenstate bases of H
0
in the presence
of spin. Relate these two bases to each other, via the Clebsch-Gordan series expansion method.
(You do not have to determine the CG coecients explicitly. It is sucient that you derive
the equation satised by the CG coecients by imposing the condition that both bases are to
be normalized.)
(d) By using the most suitable of these bases, compute the corrections to
E
0
n
=

2
2n
2
(m
e
c
2
) (5)
(where H
0
[n
0
) = E
0
n
[n
0
)) due to H
1
, to lowest non-vanishing order. (You can use

1
r
3
)
nlm
=
_
a
3
0
n
3
l
_
l +
1
2
_
(l + 1)
_
1
(6)
c ODT

U Physics Department. (Nov 2007) 89


by carefully reducing the 1/r
3
) in the basis of your choice, involving spin to the [nlm)-basis.
You have to take into account the possible values of j, the total angular momentum quantum
number. Here a
0
is the Bohr radius: a
0
=

mc
; =
e
2
c
=
1
137
is the ne structure constant.)
c ODT

U Physics Department. (Nov 2007) 90


3. Consider the Dirac equation for the stationary states, namely the eigenvalue equation for the Dirac
Hamiltonian,
H
D
(x) = E(x) ; (x, t) = exp

Et
(x) (7)
for a charged Dirac particle in a Coulomb potential, (r) = e/r, so that potential energy is
V = e = e
2
/r.
(a) Work out the Dirac Hamiltonian using the minimal substitution principle . Next, you are
asked to derive the non-relativistic limit of this equation by retaining relativistic corrections
to order v/c,through the following steps:
(b) Parametrize the stationary Dirac spinor in terms of the 2-component spinors , as
(x) =
_
(x)
(x)
_
(8)
and E as E = E

+ mc
2
, (that is E

=
_
c
2
p
2
+ m
2
c
4
+ V mc
2
). Then derive the coupled
2-component exact equations for and in terms of E

.
(c) From the second equation, derive the exact relation which expresses in terms of . Use
series expansion, with
E

V (r)
2mc
2
as the expansion parameter. (Recall: V (r) becomes large
close the origin. That is why we do not neglect as compared to 2mc
2
, from the begining.)
Then substitute this approximate expression, I(), in the rst equation to obtain an
uncoupled (higher order) equation for , in the form H
(D)
NR
= E

NR
, where E

NR
is the NR
limit of E

(obtained by expanding
_
c
2
p
2
+ m
2
c
4
in terms of p
2
/m
2
c
2
to lowest order, in
E

=
_
c
2
p
2
+ m
2
c
4
+ V mc
2
).
(d) You will get terms like i
V

r
xr in H
(D)
NR
. Check whether this term is Hermitian. If not, you have
to Hermitianize it. After this important step, you have a Hamiltonian which must describe
the Hydrogen atom with all the relativistic corrections to lowest order. Check whether this
is really is the case, that is H = H
0
+

3
i=1
H
i
, where H
0
= H
Coulomb
; H
1
: (correction from
relativistic kinetic energy); H
2
: (spin-orbit term); H
3
: (Darwin term).
Hint: Dirac matrices are given as,
=
0
0
,
0
=
I 0
0 I
(9)
c ODT

U Physics Department. (Nov 2007) 91


Nov 2007
STATISTICAL MECHANICS
1. Consider three spins that sit on the corners of an equilateral triangle. Each spin can assume two
states: up (s = +1) or down (s = 1). The system is in equilibrium at temperature T. Suppose
that in the absence of an external magnetic eld the energy of the system derives from the spin-spin
interactions. The interaction energy between a pair of spins is given by E
ij
= Js
i
s
j
where J > 0
and the Hamiltonian of the system is given by H = J(s
1
s
2
+ s
1
s
3
+ s
2
s
3
).
(a) Write out the canonical partition function explicitly.
(b) Find the probability that a particular corner of the triangle has spin up (s = +1) on it.
(c) Find the average energy of the system.
(d) Find the (average) magnetization if the system is subjected to a uniform external magnetic
eld, B.
2. Consider a non-relativistic Fermi gas of N non-interacting particles of mass m and spin s. The gas
is conned to a volume L
D
in D-dimensional space and is in equilibrium at temperature T. Let

D
be the volume of the unit sphere in D dimensions.
(a) Let g

denote the average number of particles at energy . Plot g

versus for T 0
+
.
= (k
B
T)
1
and k
B
Boltzmann constant.
(b) Express N in terms of the appropriate Fermi-Dirac integral. Note the Fermi-Dirac integral of
order n is dened as
F
n
(z) =
1
n!
_

0
x
n1
(z
1
exp
x
+1)
1
dx (1)
(c) Estimate the increase in energy for T just above 0
+
by estimating the fraction of particles
excited and the average excitation energy of the excited particles. No detailed calculations
are required. Just give some reasonable estimates on the order of magnitudes in terms of the
parameters of the system.
(d) Let C
V
denote the heat capacity at constant volume. Find
C
V
Nk
B
for T by using a general
theorem. No proof of the theorem is required. Just state the theorem and its consequence(s).
3. Consider an ideal gas of ultra relativistic bosons in three dimensions of spin zero and mass m. The
energy wave-vector relation for a single particle is = c[

k[, where c is the speed of light.


(a) From the relation for the mean number of particles per unit volume N)/V , in terms of
fugacity z and temperature T, show that the system undergoes Bose-Einstein condensation
below some non-zero critical temperature T
c
. Give an expression for T
c
in terms of N)/V ,
various fundamental constants, and a dimensionless integral.
(b) For this system, evaluate mean energy per volume, E)/V , in the Bose-condensed phase at
T < T
c
. Show that E)/V is proportional to T raised to some power.
c ODT

U Physics Department. (Nov 2007) 92


May 2007
May 2007
ANALYTICAL MECHANICS
1. A bead of mass m slides without friction on a wire in the shape of a semicircle with equations
x = a cos , y = a sin , /2 /2, under the action of gravitational force. Find the force
acting on the bead by the wire.
2. Consider the transformation
Q = q exp
Rt/2m
, R = p exp
Rt/rm
(1)
where R and m are constants.
(a) Show that the transformation is canonical..
(b) Find the generating function F
2
(q, Q, t) for the given transformation.
(c) The Hamiltonian for the damped harmonic oscillator is given as
H =
p
2
2m
exp
Rt/rm

k
2
q
2
exp
Rt/rm
,
where k is constant. Obtain the time independent Hamiltonian by using the given transfor-
mation.
(d) Use the Hamilton-Jacobi method to nd q(t) and p(t).
3. A pendulum is constructed by putting a bob of mass m at the end of a massless spring of equilibrium
length l
0
and spring constant k, which can stretch along its length, but does not bend. The motion
is restricted to a vertical plane.
(a) Write down the Lagrangian for this system.
(b) Using the result of part a), write down equations of motion.
(c) Find the coordinates of equilibrium position .
(d) Find the normal modes and corresponding eigenfrequencies of this system.
c ODT

U Physics Department. (May 2007) 94


May 2007
ELECTROMAGNETIC THEORY
1. (a) A long coaxial cable have a charge density and + on its outer conducting shell and on the
surface of the inner conducting rod, respectively. Find the electric eld and potential between
the shell and the rod away from the ends. The radius of the rod is r
R
and the shell is r
S
.
(b) The coaxial cable of part (a) is connected to a battery with e.m.f.
0
and a resistance R as
shown. Find the magnetic eld away from its ends between the shell and the rod assuming
that the current ows uniformly through the surfaces of the conductors.
(c) For part (b) of the problem, nd the Poynting vector and using it show that the power
transported between the inner rod and outer shell is I
2
R.
Some useful information:

S =
1

E

B
2. A simple model for an atom may be a point nucleus having charge +q and surrounded by uniform
spherical charge distribution with total charge q.
(a) Show that such an atom in a uniform electric eld

E has atomic polarizability = 4
0
a
3
where a is the radius of the atom. (Note that the atomic polarizability is dened by the
equation: p =

E, where p is the dipole moment of the atom.)


Instead of above crude model, assume that the electrons bound to the atom are damped
oscillators with natural frequency
j
, damping force m
j
dx
dt
and oscillator strength f
j
for the
jth electron (Note that

j
f
j
= Z for an atom where Z is the atomic number and the equation
of motion for such an electron can be written as:
m
e
d
2
x
dt
2
+ m
e
dx
dt
+ m
e

2
x =

E
0
exp
it
. (1)
) Solve followings.
(b) Show that for a material having such atoms when acted by EM wave with temporal part

E =

E
0
exp
it
will have polarization given by the real part

P =
Ne
2
m
e
_
_

j
f
j

2
j

2
i
j

_
_
E (2)
of where N is the number of atoms per unit volume. (Note that polarization is the average
dipole moment per unit volume.) Assume steady state.
(c) Using part (b) nd the complex dielectric constant of this material.
Some useful information:
1
4
0
= 8.99 10
9
N m
2
/C
2
Bohr radius= a
0
= 5.29 10
11
m
q
e
= e(electronic charge) = 1.6 10
19
C
m
e
= 9.11 10
31
kg
c ODT

U Physics Department. (May 2007) 95


3. Radiation of electric eld from an accelerated charge can be expressed as

E
rad
=
e
c
2
r
[ r ( r a)]
retarded
. (3)
(a) Find the Poynting vector for emitted radiation as

S =
c
r
E
2
rad
r (4)
(b) Using the Poynting vector nd the total emitted power as
P =
2e
2
3c
3
[a[
2
(5)
This formula is called Larmor formula for a nonrelativistic accelerated charge.
(c) Using the Larmor formula nd the instantaneous and average emitted radiation power from
a simple harmonic oscillator with a spring constant k with an attached mass m and charge q.
(d) Write the Lorentz invariant generalization of Larmor formula.
(e) Using the Lorentz invariant generalization of Larmor formula obtain the total emitted power
as
P =
2e
2
3c
3

6
_
[a[
2
[ a[
2

(6)
c ODT

U Physics Department. (May 2007) 96


May 2007
METHODS OF MATHEMATICAL PHYSICS
1. Given the dierential equation
(1 x
2
)
d
2
y
dx
2
(2l + 3)x
dy
dx
+
_
l(l + 2) + R
2
0
(w
2

1
R
2
0
_
y(x) = 0 , (1)
where x [1, 1], l = 0, 1, . . . and R
0
is a constant.
(a) Find a series solution about the origin.
(b) For y(x) = finite, nd the allowed values of w.
(c) Find the condition for the solutions to form an orthonormal set.
2. A certain set of polynomials: A
n
(x), n = 0, 1, . . . are dened by the generating function
T(x, t) = exp
t
2
+2xt
=

n=0
A
n
(x)t
n
(2)
(a) UsingT(x, t) derive two basic recursion relation.
(b) Using these recursion relations nd the second order dierential equation that A
n
(x) satises.
(c) Is this equation self-adjoint? If not determine the weight factor.
3. In quantum mechanics, the following integral shows up in problems regarding time dependent
perturbation theory, scattering and WKB barrier penetration. Assuming that E
0
and are positive
real constants, evaluate
I

2i
+
_

exp
it
d
E
0
i/2
fort > 0 . (3)
Explain your reasoning carefully in every step you take. Discuss also what happens when t < 0.
c ODT

U Physics Department. (May 2007) 97


May 2007
QUANTUM MECHANICS
1. Let [ n ) be the spin-up state along n for a particle with spin 1/2. Here n = n
x
x+n
y
y +n
z
z is an
arbitrary unit vector and the state satises the eigenvalue equation
S
n
[ n ) =

2
[ n ) (1)
where S
n
=

S

n is the component of spin along n. You dont need to compute the state [ n ) in
order to solve this problem.
(a) Show that for any arbitrary operator A, we have [A, S
n
]) = 0 in this state.
(b) Use the result above for A = S
x
, S
y
, and S
z
; solve the equations obtained; and nally compute
S
x
), S
y
) and S
z
). What is

S)? (A relation you may need: )S


n
) = /2.)
(c) Compute the uncertainties S
x
, S
y
and S
z
.
(d) Write down the uncertainty relation for the product S
x
S
y
. Then, show that this relation
is satised in this particular state.
2. Consider a harmonic oscillator having the Hamiltonian
H =
p
2
2m
+
1
2
m
2
x
2
(2)
Let [n)(n = 0, 1, 2, . . .) represent the usual normalized eigenstates of H. The oscillator is initially
(at t = 0) in the state given by
[(t = 0)) =
1

2
([n) [n + 1)) (3)
(a) Write down [(t)) the state at time t. Show that the state is periodic, so that after a
time T the state returns back to the initial state together with an overall phase factor, i.e.,
[(t + T)) = exp
i
[(t)). What is T and what is the phase factor?
(b) Compute the average position, x)
t
, and the average momentum, p)
t
, at time t.
(c) Compute the uncertainties in position and momentum, x
t
and p
t
, at time t and show that
the uncertainty relation is satised at all times.
Note: a =
_
m
w
_
x + i
p
m
_
3. The simplest candidate for a relativistic generalization of free Schrodinger equation is the Klein-
Gordon (KG) equation which is obtained by direct application of the operator version of relativistic
energy-momentum dispersion relation on an arbitrary function of space and time.
(a) Construct this equation and prove why it is not acceptable as the correct relativistic quantum
mechanical equation of motion.
(b) Work out the non-relativistic limit of the KG equation, and check whether you get the correct
non-relativistic equation in positive and negative energy cases separately.
(c) Repeat the same analysis in part (a) and (b) when the particle carries charge q, in the presence
of electromagnetic interaction
c ODT

U Physics Department. (May 2007) 98


May 2007
STATISTICAL MECHANICS
1. Consider a two-dimensional classical ideal gas of N particles. Each particle has mass and the
temperature is T.
(a) Find the partition function.
(b) Find the Helmholtz free energy.
(c) Find the force exerted on one side of the square wall by the gas.
2. Consider a three-dimensional ideal gas of ultra relativistic electrons. The single particle energies
are given by = ck.
(a) Find the Fermi energy
F
.
(b) Find the density of states ().
(c) Find the energy E of the gas at absolute zero.
(d) Find the pressure exerted by the gas at T = 0 K.
3. Consider a photon gas and a classical ideal gas in equilibrium in a region of space. The photon
gas is described by the massless relativistic dispersion relation (p) = cp , for particles with two
polarization states.
(a) Can we set = 0 for the photon gas?
(b) Find the grand canonical partition function for the photon gas.
(c) Calculate the pressure P
rad
that the photon gas exerts at temperature T.
(d) What must the temperature T be so that the photon gas pressure P
rad
and the ideal gas
pressure are equal.
c ODT

U Physics Department. (May 2007) 99


Nov 2006
Nov 2006
ANALYTICAL MECHANICS
1. The equation of motion for the Dung oscillator is
q + q + q + q
3
= 0 (1)
where and are arbitrary constant.
(a) Set up a Lagrangian for this system.
(b) If the mass m = 1, nd an expression for the total energy in terms of q and q. Is the total
energy constant? Explain your answer.
(c) Obtain a Hamilton function for this system. Is it equal to the total energy?
(d) Show that the quantity
I = e
4
3
t
[
q
2
2
+
q q
3
+

2
q
2
18
+
q
4
4
] (2)
is a rst integral of this system if 9 = 2
2
.
2. A typical diatomic molecule, like CO, may be regarded as equivalent to two masses m
1
and m
2
connected by a massless spring of force constant k and of unstretched length l, oscillating along
the line joining the two masses. Assume that the motion is only in the horizontal direction.
(a) Write down the Lagrangian for the system.
(b) Find the normal frequencies and normal modes of small oscillation.
(c) Rewrite the Lagrangian in terms of the normal coordinates.
(d) Find the equations of motion and solve them.
3. (a) Is the transformation
X =
1

m
(
_
2P
1
sinQ
1
+ P
2
),
Y =
1

m
(
_
2P
1
cosQ
1
+ Q
2
),
P
x
=

m
2
(
_
2P
1
cosQ
1
Q
2
),
P
y
=

m
2
(
_
2P
1
sinQ
1
+ P
2
) (3)
canonical?
(b) Find the Hamiltonian of the motion for a particle moving in a plane magnetic eld described
by the vector potential

A = (
Y B
2
,
XB
2
, 0), in terms of the new variables Q
1
, Q
2
, P
1
, P
2
introduced above ( Use the denition =
eB
mc
).
(c) Use the results of part (b) solve the Hamiltonian equations.
(d) Write down the solution of the Hamiltonian equations in terms of the variables X, Y, P
x
, P
y
.
Give the physical interpretation of the obtained solution.
c ODT

U Physics Department. (Nov 2006) 101


Nov 2006
ELECTROMAGNETIC THEORY
1. (a) A sphere with radius a is placed in an electric potential eld which initially is
0
= E
0
z.
Draw qualitatively the charge density with respect to on the sphere (see gure) if it is:
i. conducting, neutral and isolated;
ii. conducting, charged and isolated;
iii. dielectric material.
(b) Using the solution of
2
= 0 in spherical coordinates nd the potential everywhere for part
(a), case(i).
(c) Find the potential if the conducting neutral isolated sphere with radius a is placed in a region
where initially there exists a potential
0
(r, , ) = r
2
e
i
sin 2.
(d) How would the result of part (b) change if instead of conducting material a dielectric sphere
is placed in
0
= E
0
z. Explain in a few statements. You may use:
Y
00
=
1

4
,
Y
11
=
_
3
8
Sine
i
,
Y
10
=
_
3
4
Cos,
Y
22
=
1
4
_
15
2
Sin
2
,
Y
21
=
_
15
8
SinCose
i
,
Y
20
=
_
5
4
(
3
2
Cos
2

1
2
),
Y
mn
(, ) =

2m + 1
4
(mn)!
(m + n)!
P
n
m
(Cos)e
in
_
2
0
d
_

0
Y

m

n
(, )Y
mn
(, ) sin d = mm

nn

(r, , ) =

m
n=m

n=m
(Ar
m
+ Br
(m+1)
)Y
mn
(, )
c ODT

U Physics Department. (Nov 2006) 102


2. An oscillating linear quadrupole composed of two equal charges q placed at a distance a from a
center charges +2q as shown in the gure.
(a) Using the quadrupole tensor Q

=
_
(3x

r
2

)(

x )d
3
x, nd the quadrupole moments.
(b) Find the power emitted per solid angle.
(c) Plot the radiation pattern qualitatively.
(d) Find the total radiated power.
3. Consider the Lagrangian density of electromagnetic eld is modied by adding a mass term. This
Lagrangian is known as the Proca Lagrangian,
L =
1
16
F

+

2
8
A

1
c
J

(1)
where = m

c/.
(a) Using Euler-Lagrange equation nd the equation of motion as

+
2
A

=
4
c
J

(2)
(b) Using the Lorentz gauge show the equation of motion in part a can be written as
[]A

+
2
A

=
4
c
J

(3)
(c) Write the above equation in the static limit as

2
A

2
A

=
4
c
J

(4)
(d) If the source is a point charge q at rest at the origin. Then only nonvanishing component
is A
0
= . In this case show that scalar potential in Yukawa form ( = q
e
r
r
) satises the
equation in part c.
c ODT

U Physics Department. (Nov 2006) 103


Nov 2006
METHODS OF MATHEMATICAL PHYSICS
1. Analysze the convergence properties of the series

k=0
a
2k
x
2k
where
a
k+2
=
k(k 1) + C
0
k C
1
(k + 1)(k + 2)
a
k
, (1)
for
(a) the interval (1, 1),
(b) the end points x = 1,
(c) For what values of C
0
and C
1
does it converge in the entire interval [1, 1]?
2. (a) Find the following inverse Laplace transform,
L
1
k
2
(s
2
+ k
2
)
2
(2)
(b) Find the Laplace transform of Laguerre polynomials, L
n
(at), where a is a constant. Laguerre
polynomials satisfy the dierential equation
tL

n
(t) + (1 t)L

n
(t) + nL
n
(t) = 0 (3)
3. Evaluate
I =
_

x
2
x
4
2x
2
cos2 + 1
dx (4)
where 0 < < 2 and ,= .
c ODT

U Physics Department. (Nov 2006) 104


Nov 2006
QUANTUM MECHANICS
1. Consider a 2-particle system described by the Hamiltonian
H =

p
1
2
2m
1
+

p
2
2
2m
2
+ V ([

r
1


r
2
[) (1)
We dene a new set of observables (

r ,

p ,

R,

P ) in terms of the ones (

r
1
,

p
1
,

r
2
,

p
2
) as

r =

r
1


r
2
,

p =
m
1

p
2
m
2

p
1
m
1
+ m
2
,

R =
m
1

r
1
+ m
2

r
2
m
1
+ m
2
,

P =

p
1
+

p
2
(2)
(a) Construct the Hamiltonian in terms of the new observables (

r ,

p ,

R,

P ) and the new mass
parameters M, dened as, M = m
1
+ m
2
,
1

=
1
m
1
+
1
m
2
.
(b) Starting from the basic commutators, [r
1i
, p
1j
] = i
ij
, [r
2i
, p
2j
] = i
ij
(and all other com-
mutators you can think of are zero) work out [r
i
, p
j
] and [R
i
, P
j
] (as well as [r
i
, P
j
] and
[R
i
, p
j
]).
(c) Work out

L
1
=

r
1


p
1
,

L
2
=

r
2


p
2
and

L
tot
=

L
1


L
2
in terms of the new
observables, and express

L
tot
in terms of an

L
r
=

r

p and

L
CM
=

R

P .
2. In seeking for a relativistic covariant equation with positive denite probability density. Dirac has
proposed
i

t
= (
c
i

x
i
+ mc
2
) (3)
He has taken
i
,
i
constant coecients as matrices and as a column matrix.
(a) Using the fact that this equation must give the correct relativistic energy-momentum relation
construct the algebra, which matrices
i
and
i
should satisfy in D = 1 and D = 2 space
dimensions.
(b) By working out the additional physical and mathematical properties of
i
and
i
determine
the minimum dimension of the spinor space and construct the
i
and
i
matrices explicitly
in D = 1 and D = 2 dimensions.
Express the Dirac Hamiltonian in terms of these matrices and the momentum operator p
i
=

x
i
where i = 1 for D = 1 and i = 1, 2.
c ODT

U Physics Department. (Nov 2006) 105


3. Consider a free particle in 1D having the Hamiltonian H =
p
2
2m
. At the initial time (t = 0) the
particle is in an arbitrary initial state.
(a) Compute the uncertainties in position and momentum at time t(x
t
, p
t
) in terms of some
initial expectation values.
(b) Let C
t
represent the position-momentum correlator at time t, which is dened as
C
t
=
1
2
< xp + px >
t
< x >
t
< p >
t
(4)
Compute C
t
in terms of some initial expectation values.
(c) Use the uncertanity principle on the result in part (a) to derive the following inequality
x
2
0
p
2
0


2
4
+ C
2
0
, (5)
which is a more general form of the Heisenberg uncertainty principle. Argue why this inequal-
ity should also be correct for other systems in which the particle is not free (for systems where
H
.
=
p
2
2m
).
(d) Show that the expression
x
2
t
p
2
t
C
2
t
, (6)
is independent of time for the free particle.
c ODT

U Physics Department. (Nov 2006) 106


Nov 2006
STATISTICAL MECHANICS
1. Consider a two-level system with energies 0 and where the energy of the excited state given as
=

V

, where V is volume and , are constants.


(a) Calculate the canonical partition function Z.
(b) Calculate the internal energy E of the gas of N such distinguishable atoms.
(c) Calculate the pressure P of the gas.
2. Consider an ideal gas of ultra relativistic bosons of spin zero and mass m in three dimensions. The
energy wave vector relation for a single particle is = c[k[ where c is the speed of light.
(a) From the relation for the mean number of particles per unit volume
<N>
V
, in terms of fugacity
z and temperature T. Show that the system undergoes Bose-Einstein condensation below
some non-zero critical temperature T
c
. Give an expression for T
c
in terms of
<N>
V
, various
fundamental constants and a dimensionless integral.
(b) For this system evaluate mean energy per volume
<E>
V
in the Bose-condensed phase at T < T
c
.
Show that
<E>
V
is proportional to T raised to some power.
3. Consider ideal (nonrelativistic) Fermi (F) and Bose (B) gasses in two dimensions. Let A, N, T and
(= h(2mkT)
1
2
) denote correspondingly the area, the number of particles, the temperature and
the mean thermal wavelength. Fermi-Dirac and Bose-Einstein integrals are respectively denoted
by f
n
() = F
n
()/(n) 0 < and g
n
() = G
n
()/(n) 0 < 1 with (n) = (n 1)!
and = exp() where = (kT)
1
, k, Boltzmann constant, , chemical potential.
(a) Find the relationship between the chemical potentials,
B
and
F
, of the two gases.
(b) Express the internal energies, U
F
and U
B
, in terms of the Fermi-Dirac / the Bose-Einstein
integrals and .
(c) Show that the specic heats, C
V
(N, T), of the two gases are identical for all N and T.
Hints: It may be useful to know,
(a)
f
1
() = ln(1 + ), (1)
(b)
g
1
() = ln(1 ), (2)
(c)
f
2
(
F
) =
1
2
ln
2
(1 +
F
) + g
2
(
B
). (3)
c ODT

U Physics Department. (Nov 2006) 107


May 2006
May 2006
ANALYTICAL MECHANICS
1. Two particles, each of mass m, move in one dimension along the x axis, with positions x
1
and
x
2
. Both particles move in the same quadratic potential, V (x) =
1
2
kx
2
. In addition, the particles
are attached to each other by the potential,
1
2
k
1
(x
1
x
2
)
2
(a) Write the Lagrangian of the system.
(b) Derive the equations of motion
(c) Find normal modes, x
1
(t) and x
2
(t).
(d) Are the energy and momentum conserved?
2. Consider particles in three dimensions x , y, z with Hamiltonian
H =
p
2
x
+ (p
y
+ ax)
2
+ (p
z
+ bz)
2
2m
(1)
where m is mass of particle, a and b are some constants.
(a) Find Hamilton equations of motion.
(b) Find Lagrangian of the particle.
(c) Find the equations of motion (Lagrange-Euler)
(d) Solve the equations of motion for x, y, z in terms of the time t and initial coordinates and
velocities.
3. A particle of mass m is moving in the xy-plane about the xed mass M under the action of an
attractive central force f(r) e
r
. It is assumed that no other force is acting.
(a) Show that the equations of motion are
m r mr

2
= f(r), mr
2

= p

= (2)
where = constant.
(b) Show that the dierential equation of the path followed by m is
d
2u
d
2
+ u =
mf(1/u)

2
u
2
(3)
where u = 1/r.
(c) Find the force law if the path followed by the particle m is a spiral, r = k, where k is a
constant.
(d) Determine r and at any time t 0.
(e) Find the time it takes for the particle moving on this orbit to fall down from the radial distance
r
0
to the force center M.
c ODT

U Physics Department. (May 2006) 109


May 2006
ELECTROMAGNETIC THEORY
1. A small circuit loop of wire of radius carries a current I = I
o
cos(wt) = Re(I
o
e
iwt
) located in the
xy plane as shown in the gure.
z
x
y
I cos(wt)
(a) Calculate the magnetic dipole moment in terms of spherical coordinates. (Hint: z = cos r
sin

)
Vector potential of the system is given

A =
ik
0
4
( n m)
e
ikr
r
(1
1
ikr
),
(b) Calculate the magnetic and electric elds at the far eld zone.
(c) Determine the emitted power per solid angle.
(d) Explain the main features of radiation pattern.
(e) Calculate the average radiated power.
2. A perfectly conducting sphere of radius R moves with constant velocity v along x-axis through a
uniform magnetic eld B along y-axis.
x
z
y
v
B
(a) Using Lorentz transformation given below nd the electric and magnetic elds relative to the
moving frame of sphere.

E = (

E +

c


B)

2
+ 1

c
(

c
.

E) (1)
c ODT

U Physics Department. (May 2006) 110



B = (

B

c


E)

2
+ 1

c
(

c
.

B) (2)
(b) Find the induced surface charge density on the sphere.
b
a
x
y
3. (a) Starting from Coulombs law (d

E =
dq
4
0
(

x

x

)
|

x

x

|
3
, where dq = (x

)d
3
x

,

x and

x

are
the eld and the source point coordinates ) derive the dierential equations of electrostatic.
(.

E =

0
,

E = 0).
Also nd the Poissons equation
2
=

0
where is the scalar electric potential.
(b) Solve Poissons equation for source free medium (Laplace equation) in Cartesian coordinates.
(c) An innite rectangular pipe having metallic sides placed along z-axis as shown in the cross
sectional view. The sides y = a, y = 0 and x = 0 are grounded while x = b has potential

0
(y). Find the potential inside the pipe.
(d) Find the explicit result if
0
(y) = V
0
sin(
2y
a
). where V
0
is constant.
(e) Find the explicit result if
0
(y) = V
0
. Draw qualitatively equipotentials and electric eld lines
for this part.
c ODT

U Physics Department. (May 2006) 111


May 2006
METHODS OF MATHEMATICAL PHYSICS
1. Let
I =
_
d[(1
2m
r
)

t
2
+
1
(1
2m
r
)
r
2
+ r
2

2
], (1)
where m is a real constant, r(), t() and () are the dependent variables with r() 2m and a
dot over a letter denotes dierentiation with respect to the independent variable .
(a) Calculate the Euler-Lagrange equations of motion which extremizes this integral.
(b) Integrate the equations coming from the variations of I with respect to t and and show that
these dene two integration constants E and L, respectively, as E = (1
2m
r
)

t and L = r
2

.
(c) Assuming that r = E at r = 2m, show that the last equation of motion (the one coming from
the variation of I with respect to r) can be integrated once and is given by
r
2
= E
2
(1
2m
r
)( +
L
2
r
2
), (2)
where = (1
2m
r
)

t
2

1
(1
2m
r
)
r
2
r
2

2
= constant.
NOTE:This question is related to the geodesics of the Schwarzschild black hole solution of
General Relativity. Here one has set =

2
. In fact E is the total energy, L is the angular
momentum, m is the mass of the black hole, is called the proper time and = 1 for a
timelike particle.
2. Using Greens functions nd the solution of
d
dx
(x
dy
dx
)
4
x
y(x) = x
2
(3)
subject to the boundary condition, y(0) = y(1) = 0.
3. In quantum mechanics the 3 dimensional isotropic harmonic oscillator leads to the following
dierential equation for the radial function R(r)
d
2
R
dx
2
+
2
x
dR
dx
+ ( x
2

( + 1)
x
2
)R(x) = 0, = 0, 1, 2, ... (4)
where x and are dimensionless quantities dened as;
x =
r
_
h/m
, and =
E
_
h/2
. (5)
(a) Examine the nature of the singular point at x = .
(b) Show that the behavior of the solution as x is R e
x
2
/2
.
(c) Find a power series expansion of the solution about x = 0.
(d) Examine the nature of the series at x = 0 and x = .
(e) Find the values for for which the series terminates.
c ODT

U Physics Department. (May 2006) 112


May 2006
QUANTUM MECHANICS
1. (a) For a free particle, compute the number of states having an energy in the interval (E, E+dE)
and the momentum

p in the solid angle d.
(b) A hydrogen atom in its ground state is placed in an electric eld

(t) =

0
sint (

0
is a
uniform eld, i.e., space independent and >
me
4
2
3
). Find the probability per unit time (=
transition rate) that the atom will be ionized (that the electron is ejected).
Hints:

i
=
100
= (
3
a
0
)
3/2
e

r
a
0
, E
i
=
me
4
2
2
.
You can take the wave function of the ejected electron as a plane wave,
f
= L
3/2
e
i

k .

x
.
You can use the Fermis golden rule expression for the absorption probability per unit
time
i[f]
=
2

[V
fi
[
2
(E
f
)[
E
f
=E
i
+
.
In calculating the density of the nal states, pay attention to the fact that the matrix
element depends not only on the nal state energy, but also on the direction of momentum.
Thus, you have to rst compute the transition rate with ejection of the electron into the
element of the solid angle d and then obtain the total transition rate by integrating
overall directions. Note also that the density of states for the ejected free electrons is to
be computed using periodic boundary conditions.
2. Dirac equation for a free electron is given as
(i

d
dx

mc)(

x , t) = 0, with

= 2g

I, and (1)

0
=
_
I
0
0
I
_
=
_
0

0
_
. (2)
It is known that, the interaction with an external electromagnetic eld A

= (,

A) can be intro-
duced via the gauge invariant substitution p

= p
e
c
A

in the free Dirac equation.


(a) Construct the continuity equation in the interacting case by identifying the probability density
and the probability current. Discuss the free case also.
(b) Construct the velocity operator

v =
d
d

x
dt
using the Heisenberg picture (or equivalently Ehren-
fest theorem), and compare with the free case.
Check whether v is constant of motion. Interpret.
Compute the commutator among the components of velocity operator [ v
i
, v
j
]. Interpret
this result from the perspective of Heisenberg uncertainty principle.
Check whether p is constant of motion. Interpret. Is there a puzzle in the commutation
relations of velocity and momentum operators?
(c) Show that v
i
,

H
D
+ e = 2c
2
( p
i

e
c
A
i
). What is the free particle limit of this expression?
(d) For the free particle case, determine

x (t) in the Heisenberg picture, in terms of

x (0) and
the constants of motion, you have previously identied. Identify the relativistic quantum
mechanical eects in the expression explicitly.
c ODT

U Physics Department. (May 2006) 113


(e) Using the information you obtained in part (c) for the free case, prove that


c
b
p
H
has non-
vanishing matrix elements only between the state of equal momentum and opposite energies.
3. Consider an electron with charge e moving in the two dimensional xy-plane under a magnetic
eld along the z-direction. The Hamiltonian of the electron is given by
H =
1
2m
(

p +
e
c

A)
2
=
1
2m
(
2
x
+
2
y
) (3)
where

is the kinetical momentum and

A is the vector potential (

)

A =

B. You may want
to use the cyclotron frequency
c
=
eB
mc
(w
c
> 0) in some expressions below. (Assume that the
gauge for the vector potential is arbitrary. But if you really need it, you can choose a specic gauge
for

A).
(a) Compute the commutator [
x
,
y
].
(b) Show that for some constant D, the operator a = D(
x
i
y
) becomes a ladder operator, i.e.,
it satises [a, a
+
] = 1. Compute the constant D.
(c) Express the Hamiltonian in terms of the ladder operators a and a
+
and compute all eigenvalues
of H.
(d) Let

Q =

+
e
c

r

B = (
x
+ m
c
y) x + (
y
+ m
c
x) y. (4)
Compute [Q
x
, Q
y
] and then show that b = D(Q
x
iQ
y
) is also a ladder operator (i.e.[b, b
+
] =
1) which commutes with a a
+
. Using this operator, show that all energy levels of H are
innitely degenerate.
c ODT

U Physics Department. (May 2006) 114


May 2006
STATISTICAL MECHANICS
1. (a) Find the relation between the uctuation of energy, < (E)
2
>=< (E < E >)
2
>, and
the specic heat at constant volume, in the canonical ensemble. Note that < .... > indicates
averaging.
(b) A surface with N
0
adsorption centers has ????? gas molecules adsorbed on it. Each adsorbed
gas molecule has a one-dimensional quantum harmonic oscillator energy spectrum, that is,
E
n
= (n + 1/2)h, n = 0, 1, 2, ..... characterized by the same linear frequency . Use the
grand canonical partition function to calculate the temperature dependence of the chemical
potential, . Express your result in terms of N
0
, N, and the universal constants.
(Note that parts (a) and (b) are independent)
2. Consider the atomic hydrogen in equilibrium at temperature T. The temperature is low enough
that the electron is in its ground state as far as its orbital motion is concerned. There are four
possible states for the atom as a result of hyperne interaction between electron and proton spins.
The singlet state (with energy = 0) occurs when the proton and electron spins anti-align to form
a spin 0 atom. The triplet states (with energy > 0 ) occur when the proton and electron spins
aligned forming a spin 1 atom. Consider a gas of N atoms and focus only on these internal states.
(a) Without extensive calculation, what are the internal entropy and energy as T 0?
(b) Again without extensive calculation, what are the internal entropy and energy as T 0?
(c) What is the partition function for these internal states at arbitrary temperature? (You can
take N independent atoms as distinguishable).
(d) What are the free energy, entropy and energy of N atoms at arbitrary temperature?
3. Consider N massless non-interacting spin-s fermions, in a three dimensional box of volume V.
(a) Find the Fermi energy as a function of N, V and s.
(b) Find the pressure, for zero temperature
(c) Draw a graph of the occupation of states as a function of energy, at temperature of 0.1
E
F
k
B
,
showing the eect of temperature clearly.
c ODT

U Physics Department. (May 2006) 115


Nov 2005
Nov 2005
ANALYTICAL MECHANICS
1. A simple pendulum of mass m is attached to the point with mass M that may move without friction
on a horizontal direction. Assuming that the pendulum can move only in plane of the picture,
g
M
m

a) derive the equation of motion of the system;


b) nd all integrals of motion and indicate what conservation law does each integral corresponds
to;
c) calculate the frequency of small oscillations of the system near the equilibrium, sketch the
oscillation mode for the case M = m.
2. Use the Lagrangian formalism to nd equation of motion of a massive bead that may slide without
friction on a circular ring of radius R which is rotated with angular velocity about the horizontal
diameter.
g
m

a) Find the Lagrangian and Lagrange-Euler equation.


b) Find the Hamiltonian H of the system.
c) Find energy E.
d) Are the H and E equal?
e) Is either of them conserved in time?
3. In the presence of an electromagnetic eld, the kinetic part of the Hamiltonian is
1
2m
[ pe

A[
2
where
is the electromagnetic vector potential, m and e are the mass and charge of the particle. Consider
the situation in which we have a one dimensional harmonic oscillator in a constant magnetic
potential, but suppose that the zero point of the spring is v =
eA
m
moving to the right with speed
so the Hamiltonian is time dependent
H =
1
2m
( p e

A)
2
+ (x +Vt)
2
(1)
Solve the Hamilton-Jacobi problem for this system.
c ODT

U Physics Department. (Nov 2005) 117


You may need the followings:
_
_
a
2
x
2
dx =
x
2
_
a
2
x
2
+
a
2
2
arcsin
_
x
a
_
arcsin u(x)

=
u

(x)

1 u
2
where


d
dx
c ODT

U Physics Department. (Nov 2005) 118


Nov 2005
ELECTROMAGNETIC THEORY
1. a) Write the Maxwells equations in matter in terms of currents and charges.
b) If the medium is non conducting with constant electric permittivity and magnetic permeability
show that E and B statises the wave equations for source free regions (p=0, J=0).
c) Write an expression for phase velocity.
d) Write the plane wave solutions for E and B.
e) Show how E and B are related.
f) The electromagnetic waves are at the plane interface of two dielectrics media having the same
magnetic permeability. Write the boundary conditions.
g) Derive relationships between the angles of incidence and reection and refraction for part (f).
2. A charge density s(q) (q is the polar angle) is placed on the surface of a spherical thin shell of
radius R. Center of the shell is at the origin.
a) Find the electric eld and potential inside and outside of the shell if s(q) is constant and equal
to s
o
b) Find the electric potential both inside and outside if s(q) = s
o
cos 2q.
c) Find the magnetic scalar potential eld both inside and outside if the sphere of part a is rotated
with a constant angular velocity w about an axis passing through the center.
d) Find the magnetic eld both inside and outside for part c.
e) Explain why one can use magnetic scalar potential instead of magnetic vector potential for part
c of the problem.
P
o
(x) = 1 P
1
(x) = x P
2
(x) =
1
2
(3x
2
1) P
3
(x) =
1
2
(5x
3
3x)
_

0
P
m
(cos )P
n
(cos ) sin d =
2
2m + 1

mn
3. A dipole is placed with its center at origin as shown. The distance between the charges is d.
c ODT

U Physics Department. (Nov 2005) 119


a) Show that the potential for the far points is given as:

1
4
0
p r
r
3
where p is the dipole moment and r is the distance between the dipole and point of interest.
b) Show that the electric eld of this dipole for the far points can be written as:

E =
3( p n) n p
4
0
r
3
where n is the unit vector from dipole to the point of interest.
c) Two dipoles p
1
and p
2
are placed with their centers being on the same axis and they lie on
the same plane as shown. The angle between each dipole and the axis is given as q. Find the
interaction potential energy. (Assume that the distance r between the dipoles is much larger
than their size.)
d) What would be the interaction potential energy if a third dipole p
3
is placed on the same axis
with an angle

and at a distance r

, far away from p


2
.
c ODT

U Physics Department. (Nov 2005) 120


Nov 2005
METHODS OF MATHEMATICAL PHYSICS
1. For a function f(z) analytic everywhere in the upper-half plane and on the real axis with the
additional property f(z) a as [z[ (a is real). Show that the Cauchy principle value
integrals give
f
R
(x) = a +
1

P
_
+infty

f
I
(x

dx

x
, (1)
f
I
(x) =
1

P
_
+infty

f
R
(x

dx

x
(2)
2. In quantum mechanics the 3-dimensional isotropic harmonic oscillator leads to the following dif-
ferential equation for the radial function R(r)
d
2
R
dx
2
+
2

dR
dx
+
_
x
2

l(l + 1)
x
2
_
R(x) = 0, l = 0, 1, 2, ... (3)
where x and are dimensionles quantities dened as;
x =
r
_
h/m
, and =
E
_
h/2
. (4)
a) Examine the nature of the singular point at x = .
b) Show that the behaviour of the solution as x is R exp
x
3/2
.
c) Find a power series expansion of the solution about x = 0.
d) Examine the nature of the series at x = 0 and x = .
e) Find the values for for which the series terminates.
3. Consider the following homogeneous, linear integral equation
y(x) =
_
b
a
K(x, t)y(t)dt . (5)
Suppose that there exists only one eigenfunction
n
(x) corresponding to each eigenvalue
n
of this
equation, where n?Z
+
(i.e. n = 1, 2, ...) and
n
,= 0 for all n. Assume also that the eigenfunctions
are normalized to unity in the interval x?[a, b], thus
_
b
a

n
(x)
m
(x)dx =
nm
(6)
a) Take the inhomogeneous integral equation
(x) = f(x) +
_
b
a
K(x, t)(t)dt (7)
where f(x) ,= 0 is a piecewise continuous function in the interval x?[a, b] and is such ,=
n
that for all n?Z
+
. Solve this equation in terms of the known quantities, i.e. f(x), , the
eigenvalues
n
and the eigenfunctions
n
(x).
c ODT

U Physics Department. (Nov 2005) 121


b) As a specic example to the assumptions made in the begining of the question, nd the eigen-
values and the normalized eigenfunctions of the homogeneous integral equation
y(x) =
_
1
0
xty(t)dt . (8)
c) Now solve the inhomogeneous integral equation
(x) = x +
_
1
0
(t)dt , (9)
where is an arbitrary real number with ,= 3.
c ODT

U Physics Department. (Nov 2005) 122


Nov 2005
QUANTUM MECHANICS
1. A particle with spin 1 has magnetic moment given by = k

S where

S is the spin operator and
k is a constant. When this particle is placed inside a magnetic eld

B, its Hamiltonian becomes
H = u

B = k

S

B (1)
a) Show that
d
dt
_

S
_
t
= )
t


B (2)
here )
t
denotes the expectation value at time t for any initial state.
b) Let [1, m) denote the eigenstates of S
z
where m = 1, 0, 1 denote the corresponding quantum
number. Compute the eigenstates of S
x
in terms of [1, m).
c) Suppose that the magnetic eld is along x-direction,

B = B x and the particle is prepared in
the initial state [(t = 0)) = [1, 1). Compute the state at time t.
2. The interaction of a Dirac particle with an external electromagnetic eld described by the 4-
potential A
=(,

A)
can be introduced by means of the gauge invariant sustitution,
P

q
c
A

. (3)
a) Derive the Dirac Hamiltonian for the interaction case,
i

t
= H
D
(4)
and identify H
D
in this case.
b) Derive the non-relativistic limit of the interacting Dirac Equation by recalling that in this
limit, the kinetic energies and the eld interaction energies are small in comparison with the
rest energy mc
2
(recall these values, say for the electron in Hydrogen atom, and verify these
assumptions).
Hint: First parametrize the 4-dimensional Dirac spinor in terms of the 2-dimensional spinors
and as
(x, t) = exp

mc
2
t
_
(x, t)
(x, t)
_
. (5)
c) Identify the interaction terms explicitly in the limiting equation (known as Pauli equation).
Involving the 2-dimensional spinor for a constant uniform magnetic eld.
Determine the Land g-factor which is dened via =
gq
2m
e
c

S ( mu: magnetic moment,



S: spin).
Hint: Check that for the special

B mentioned in part (c),

A can be determined as

A =
1
2

Br
and this satises the Coulomb gauge condition, ??

A = 0.
3. Consider a particle in one dimension whose Hamiltonian is given by H =
p
2
2m
+V (x). By calculating
[[H, x], x] prove

a
[a

[x[a

)[
2
(E
a
E
a
) =

2
2m
(6)
where [a

) is an energy eigenket with eigenvalue E


a
.
c ODT

U Physics Department. (Nov 2005) 123


Nov 2005
STATISTICAL MECHANICS
1. An ideal gas composed of N particles, each of mass m, is enclosed in a vertical cylinder of height
L placed in a uniform gravitational eld of acceleration g and is in thermal equilibrium.
a) Evaluate the partition function of the gas, Q
N
.
b) Find the internal energy, U.
2. Calculate:
a) The partition function in 3-dimensions for a particle whose energy varies with wave vector as,
(k) = ak
3
,
b) The equation of state of the completely degenerate fermionic gas of N such particles.
3. The density of states of the electrons in a 3-dimensional material is assumed constant=C. Let the
total number of electrons be equal to N.
a) Calculate the chemical potential, , as a function of the temperature when the system
i) is non-degenerate
ii) is completely degenerate
Express in terms of the Fermi energy in each case.
b) Show that the heat capacity at constant volume is proportional to the temperature when the
system is highly degenerate.
Hint: The Fermi-Dirac integral is
F
n
(z) = (n)f
n
(z) =
_

0
x
n1
z
1
exp
x
+1
dx 0 z <
f
n
(z) =

m=1
(1)
m1
z
m
m
n
For z << 1
f
n
() =

n
(n + 1)
_
1 +

j=2,4,6,...
2n(n 1)...(n + 1 j)
_
1
1
2
j1
_
(j)

j
_
For large z
where ln z and (j) is the Riemann zeta function of order j.
c ODT

U Physics Department. (Nov 2005) 124


May 2005
May 2005
ANALYTICAL MECHANICS
1. The point of support of a simple pendulum of mass m and length is driven horizontally x =
a sin t. The system is in a gravitational eld.
a) Find the Lagrangian for the pendulum.
b) Find the Euler-Lagrange equation of motion.
c) Derive the Hamiltonian from the Lagrangian.
d) Find the equation of motion from the Hamilton equations of motion and compare with the
result in (b)
2. Hamiltonian of a particle with charge e and mass m moving in a plane perpendicular to a constant
magnetic eld

B and subjected to harmonic oscillator potential given by
H =
p
x
2m
+
(p
y
ax)
2
2m
+
1
2
m
2
y
2
where a =
e[

B[
c
. (1)
The solution of Hamilton equations can be obtained more easily if one rst applies the canonical
transformation to the coordinate and momenta.
a) Show that the transformation
x = X, p
x
= P
x
, y = P
y
, p
y
= Y (2)
is canonical.
b) What is the Hamiltonian in terms of the new variables? What are the new Hamilton equations
of motion?
c) Solve the equations of motion for X and Y.
3. A smooth piece of wire is twisted into the shape of a helix with cylindrical equations r = R, z =
where R and are constants. A bead of mass M, attached to the point O by a spring with spring
constant k, is constrained to move on the wire as shown in the gure.
g
z
x
y
M
r
O

c ODT

U Physics Department. (May 2005) 126


a) Write down the Lagrangian for the system by using the cylindrical coordinates (r, , z) and nd
the Lagranges equations.
b) Solve the equations that you have found in (a) and describe the motion of the bead.
c) What is the reaction force acting on the bead by the wire?
d) Write down a conserved quantity for the system. Explain your answer.
c ODT

U Physics Department. (May 2005) 127


May 2005
ELECTROMAGNETIC THEORY
1. A point charge q is placed at z = a on z-axis.
a) Write the potential at a point (r, , ).
b) Find that, for r > a, can also be written as
q
4
0

m=0
(
a
r
)
m
P
m
(cos), (1)
where r is the distance of the point to the origin.
c) Generalize your result for an axial charge distribution along z-axis given by (

x

) = (z

)(x

)(y

)
of nite length ((z

) is a charge of nite length with (z

) = 0 for [z

[ > z
0
), for points r > z
0
.
(Result is: (r, ) =
1
4
0
_
(z

)
r

m=0
(
z

r
)
m
P
m
(cos)dz

)
d) Use the general expression in part b and axial multiple moments dened as: P
m
=
_
(z

)z
m
dz

,
nd the potential for a charge conguration of +q at z =
a
2
; q at z =
a
2
and 2q at z = 0.
e) If the charge at 2q at the center of the charge conguration at part d is taken away, what
would be the monopole, dipole and quadrupole terms? Find the electric eld for this simple
conguration.
You may use: P
0
(x) = 1 P
1
(x) = x P
2
(x) =
1
2
(3x 1) P
3
(x) =
1
2
(5x
3
3x)
(1 + x)
p
= 1 + px +
p(p1)
2!
x
2
+ ........ +
p(p1)...(pk+1)
k!
x
k
+ ...
2. Two point charges of q are located at two ends of a line of length 2a. At their center +2q charge
is located. The charge system rotates with a constant angular velocity about the z-axis through
its center as shown in the gure.
-q
a
-q
2q
2a
z
x
y
a) Find the electric dipole moment.
b) Find the magnetic dipole moment.
c) Find the electric quadrupole moment of the system.
d) What type of radiation is emitted by the system?
e) What is the frequency of the radiation?
3. An electron (e) pair is created by pair production near the parent nuclei with charge Ze. Electron
and positron move relativistic speeds with Lorentz factor =
1

1(

c
)
2
10
5
, where c is the speed
c ODT

U Physics Department. (May 2005) 128


of light. Electron and positron move constant velocity with opposite directions as seen in the gure.
-e
-v t
b
v t
y
Ze
+e
x
a) Using the transformation of electromagnetic elds nd the electric and magnetic elds with
respect to the laboratory frame along y-axis.
b) Calculate the Lorentz force on parent nuclei.
c ODT

U Physics Department. (May 2005) 129


May 2005
METHODS OF MATHEMATICAL PHYSICS
1. a) Establish the following contour integral representation for the Hermite Polynomials:
H
n
(x) =
(1)
n
n! e
x
2
2i
_
c
e
z
2
dz
(z x)
n+1
Do not forget to show your contour.
Hint. Rodriguez formula for the Hermite polynomials is given as
H
n
(x) = (1)
n
e
x
2 d
n
(e
x
2
)
dx
n
.
b) Evaluate the integral to obtain a general expression for the Hermite polynomials.
2. Laguerre polynomials satisfy
tL

n
(t) + (1 t)L

n
(t) + nL
n
(t) = 0.
Evaluate the Laplace transform of L
n
(at), where a is the constant.
3. Find the minimum value of the integral
I =
_
1
0
_
dy
dx
_
2
dx,
for a function y = y(x) subject to the conditions y(0) = 0, y(1) = 0 and
_
1
0
y
2
dx = 1 .
c ODT

U Physics Department. (May 2005) 130


May 2005
QUANTUM MECHANICS
1. Show that the operator D
z
() = e
iS
z
/
really rotates a physical system by looking at its eect
on S
x
), S
y
) , S
z
).
2. Consider a one-dimensional simple harmonic oscillator (SHO).
a) Compute x and p operators as a function of time (for t 0) in the Heisenberg picture in terms
of the corresponding operators at t = 0.
b) Using these results, compute (x)
2
(t) and (p)
2
(t) as a function of time, in terms of the
corresponding quantities at t = 0 and some other expectation values at t = 0.
c) Suppose at t = 0 the state vector is given by [(t = 0) >= e
ibp/
[0 > , where [0 > is the
ground state of the SHO, b is some real number with the dimensions of length and p is the
momentum operator.
i) Compute (x)
2
, (p)
2
and the product (x)(p) at t = 0 for this state.
ii) Using the results you have found in (b) and (c-i) compute the product (x)(p) at time t
and interpret your result.
Hints: You can use
H[n >= E
n
[n > , E
n
= (n +
1
2
) ,
a[n >=

n[n 1 > , a

[n >=

n + 1[n + 1 > ,
a =
_
m
2
_
x + i
p
m
_
.
3. The orbital and spin degrees of freedom of electrons in atoms become correlated by the spin-orbit
interaction. For the purpose of investigating only this interaction, the Hamiltonian can be expressed
as
H = 2

L

S
where

L and

S are the orbital and spin angular momenta of the electron respectively and is a
constant with the dimensions of frequency.
a) Consider the states with orbital angular momentum quantum number only. What are the
eigenvalues and the corresponding eigenstates of H? What is the degeneracy of each level?
b) Suppose that an electron with spin-up is captured by the atom at t = 0 in an orbital angular
momentum eigenstate Y
m

so that the initial state of the electron is


[(t = 0) >= Y
m

[ > .
What is the average energy of this state? What are the probabilities of obtaining j = 1/2
if J
2
is measured? (

J =

L +

S is the total angular momentum and j is the quantum number
associated with J
2
.)
c) What is the state [(t) > at a later time t? Is the motion represented by [(t) > periodic? If
so what is the period?
d) What is the probability that the electron can be found in a spin-down state at time t? What is
the maximum value of this probability?
c ODT

U Physics Department. (May 2005) 131


e) Calculate the expectation values of all components of spin, < S
z
>
t
, < S
x
>
t
and < S
y
>
t
at
time t.
Hints: The common eigenstates of J
2
and J
z
for the orbital angular momentum quantum
number are
[j = +
1
2
; m
j
= m +
1
2
>
c
=

+ m + 1 Y
m

[ > +

m Y
m+1

[ >

2 + 1
, (1)
[j =
1
2
; m
j
= m +
1
2
>
c
=

m Y
m

[ >

+ m + 1 Y
m+1

[ >

2 + 1
. (2)
c ODT

U Physics Department. (May 2005) 132


May 2005
STATISTICAL MECHANICS
1. A monatomic classical gas in an cylinder with cross-sectional area A and height h (ie, volume
V=Ah). N atoms, each with mass of the gas are subjected to a linear potential
U(z) = bz (1)
where z is the vertical coordinate. Assume that the temperature T is uniform in the cylinder.
a) Find the Helmholtz free energy of this system.
b) From the free energy compute the pressure of the system by varying h.
c) Find the number density n(z) as a function of z, using the probability of nding a molecule at
height between z and z + dz.
d) Using the ideal gas law, compute the pressure as a function of height z.
e) Find the average pressure p.
2. A crystal made up of N non-interacting spin 1 atom is placed in a magnetic eld. The energy of
each atom in the eld is then, B
0
, 0 or B
0
. The crystal is at a temperature T.
a) Find the partition function of this system.
b) Compute the free energy of this system.
c) Find the mean magnetization per atom (
1
N
F
H
).
d) What is the behavior at low temperature? At high temperature?
3. Consider an ideal Fermi gas with energy spectrum p
s
, contained in a box of volume V in a
space of n dimensions.
a) Express the density of states, D()d.
b) Show that
P
k
B
T
(
1

)
n
s
s
n
F
n
s
+1
(z) (2)
where the Fermi function of order is dened by
F

(z) =
_

0
x
1
dx
z
1
e
x
+ 1
, 0 < z . (3)
c) Prove that for this system PV =
s
n
U.
k
B
: Boltzmann constant, T : temperature, = (k
B
T)
1
, P : pressure, U : energy, : chemical
potential, z = e

.
c ODT

U Physics Department. (May 2005) 133


Nov 2004
Nov 2004
ANALYTICAL MECHANICS
1. A particle of mass M is constrained to move on a horizontal plane. A second particle of mass m
is constrained to a vertical line. The two particles are connected by a massless string which passes
through a hole in the plane. The motion is frictionless.
a) Find the Lagrangian of the system and show that the angular momentum is conserved.
b) Find the Hamiltonian of the system and derive the equation of motion for r using Hamiltons
equations.
c) Initially, the mass M moves in a circle with kinetic energy E
o
. If the string is then slowly pulled
by a vertical force until the radius of the circle will be half of its initial value, how much work
was done?
2. Let the Hamiltonian of one dimensional system is given by
H(p, q) =
p
2
2m

1
q
2
a) Show that the following quantity is a constant of motion
C(q, p, t) =
p q
2
H t
b) Show that the following transformation is canonical
q
x
= Q
x
cos +
P
y
m
sin (1)
q
y
= Q
y
cos +
P
x
m
sin (2)
p
x
= mQ
y
sin + P
x
cos (3)
p
y
= mQ
x
sin + P
y
cos (4)
c) If old Hamiltonian is
H(q
x
, q
y
, p
x
, p
y
) =
p
2
x
+ p
2
y
2m
+
1
2
m
2
_
q
2
x
+ q
2
y
_
,
c ODT

U Physics Department. (Nov 2004) 135


what is the new Hamiltonian in terms of the new variables Q
x
, Q
y
, P
x
, P
y
?
3. A particle with mass m moves under the inuence of an attractive central force

F(r) =
n
[r[
2
(5)
where n is unit vector along r and > 0.
a) Show that the force is conservative and calculate the corresponding potential U(r) such that

F(r) =

U(r).
b) Explain why the total energy E=T+U and the angular momentum

L = r p are constants of
the motion. Show that the motion proceeds in the plane orthogonal to the vector

L.
c) Find the Lagrangian of a particle in the central eld using polar coordinates (, ) as shown in
the gure. Obtain the expressions for E and =

in terms of these variables. Combine the


two results to get an equation of the form
E =
m
2

2
+ U
eff
(, ) = const.
and nd the eective potential U
eff
(, ).
d) Draw systematic picture of the potential, i.e. determine its extremum, nd its asymptotic
behavior for 0 and of the eective potential and make a sketch of the graph of
U
e
(, ) for ,= 0.
e) Discuss the features of the motion for E < 0 and E 0.
c ODT

U Physics Department. (Nov 2004) 136


Nov 2004
ELECTROMAGNETIC THEORY
1. Consider the unpolarized incident electromagnetic waves scattered from a small dielectric sphere
() of radius a with = 1.
a) Calculate the scattering cross section per solid angle.
b) Find the polarization after the scattering.
2. Using method of images, discuss the problem of potential of a point charge q inside a hollow
grounded, conducting sphere of inner radius a. Find
a) the potential inside the sphere,
b) the induced surface charge density,
c) the magnitude and direction of the force acting on q,
d) Is there any change in the solution if the sphere is kept at a xed potential V? If the sphere has
a total charge Q on its inner and outer surfaces?
3. An insulated coil is wound on the surface of a sphere of radius a in such a way as to produce a
uniform magnetic induction B
0
in the z direction inside the sphere and a dipole eld outside the
sphere. The medium inside and outside the sphere has a uniform conductivity gg and permeability
. Find the necessary surface current density and show that the vector potential describing the
magnetic eld has only an azimuthal component given by
A

=
B
0
a
2
2
r
<
r
2
>
sin (1)
where r
<
(r
>
) is smaller (larger) of r and a.
c ODT

U Physics Department. (Nov 2004) 137


Nov 2004
METHODS OF MATHEMATICAL PHYSICS
1. Using the Frobenius method solve the following dierential equation with the condition v(x) =
finite in the entire interval x [0, 1]:
4x(x 1)

2
v(x)
x
2
+ 2(10x 3)
v(x)
x
+ (16 +
2
x
4)v(x) = 0 (1)
a) Show that this condition could only be satised by certain characteristic values of and nd
these values.
b) Find the weight function w(x) so that the functions v(x, ) form an orthogonal set:
1
_
0
w(x)v(x, )v(x,

)dx = 0 , (2)
where ,=

2. Generating function for the Hermite polynomials is given as


T(x, t) = e
t
2
+2xt
=

n=0
H
n
(x)
t
n
n!
(3)
a) UsingT(x, t) derive the recursion relations
2xH
n
(x) = H
n+1
(x) + 2nH
n1
(x),
2nH
n1
(x) =
dH
n
(x)
dx
(4)
b) Using these recursion relations nd the second order dierential equation that H
n
(x) satises.
c) Is this equation self-adjoint? If not determine the weight factor.
3. a) Find the Greens function for the operator
L = x
d
2
dx
2
+
d
dx

4
x
(5)
satisfying the boundary conditions G(0, x

) = G(1, x

) = 0.
b) Using this Greens function solve the dierential equation
x
d
2
y
dx
2
+
dy
dx

4y
x
= x (6)
with y(0) = y(1) = 0.
c ODT

U Physics Department. (Nov 2004) 138


Nov 2004
QUANTUM MECHANICS
1. Consider the observables momentum p and position x for a free particle.
a) Write the Heisenberg equations of motion for this system. Obtain the solution for the position.
b) Write the commutation relations for position at equal times and at dierent times.
c) Apply the uncertainty relation and obtain < (x
i
)
2
>
t
< (x
i
)
2
>
t=0
, interpret the result.
2. a) Consider a spin 1/2 particle. Find the eigenvalues and the corresponding eigenstates of the
operator

S n, where n is a unit vector in the xz -plane making an angle < 90
o
with the
+z-axis, in terms of the eigenstates of S
z
.
b) Suppose a particle is in the eigenstate corresponding to the eigenvalue 1 of part (a).
i) Find the probability of observing S
z
to have value /2.
ii) Find the expectation value of S
z
.
iii) Find the expectation value of S
x
.
iv) Check if these answers are reasonable by considering special cases.
c) A beam of spin 1/2 particles initially in the state given in part b) enters a magnetic eld

B
that points in the z-direction. Each of the particles has a magnetic moment given by

S where
is a constant.
i) Find the state of the particles as a function of time.
ii) Calculate the expectation value of S
x
and show that the spins precess around the eld
direction.
3. A spin 3/2 nucleus situated at the origin is subjected to an external inhomogeneous electric eld.
The basic electric quadrupole interaction may be taken to be
H
int
=
eQ
2s(s-1)
2
__

x
2
_
0
S
2
x
+
_

y
2
_
0
S
2
y
+
_

z
2
_
0
S
2
z
_
(1)
where is the electrostatic potential satisfying Laplaces equation, Q is the electric quadrupole
moment of the nucleus and s is the spin of the nucleus. ( )
0
means evaluated at the origin.
a) Show that the interaction can be written as
H
int
= A
_
3S
2
z


S
2
_
+ B
_
S
2
+
S
2

_
, (2)
and express A and B in terms of
_

2
/x
2
_
0
etc.
b) Determine the energy eigenkets and the corresponding energy eigenvalues.
c) Is there any degeneracy?
c ODT

U Physics Department. (Nov 2004) 139


Nov 2004
STATISTICAL MECHANICS
1. A monatomic gas consist of atoms with two internal energy levels; a non-degenerate ground state
and a non-degenerate low-lying excited state at an energy E above the ground state. Find the
specic heat of this gas.
2. Find the isothermal compressibility of K of a free electron gas at very low temperatures in terms
of the number density n.
Hint: K
1
= V
_

2
F
V
2
_
T
3. a) Consider two parts of an otherwise isolated system in which an amount of energy in the form
of heat (Q) ows from A to B and A and B are suciently large to maintain their respective
temperatures xed at T
A
and T
B
. Let g
i
and g
f
be the number of states accessible to the
system before and after the energy exchange. Given that [Q[ = 2J, T
A
=300 K and T
B
=200 K,
what is g
f
/g
i
?
b) Two systems S
1
and S
2
are brought into thermal contact. The total energy of the combined
system is xed at E=10
17
J. The number of accessible states to each increases with its energy
according to g
1
=C
1
(E
200
1
) and g
2
=C
2
(E
100
2
) where C
1
and C
2
are constants. What is the
equilibrium temperature?
Take Boltzmann constant: 1.38 10
23
J/K.
c ODT

U Physics Department. (Nov 2004) 140


May 2004
May 2004
ANALYTICAL MECHANICS
Will be added later...
c ODT

U Physics Department. (May 2004) 142


May 2004
ELECTROMAGNETIC THEORY
1. A hallow right circular cylinder of radius a has its axis coincident with the z axis and its ends at
z = 0 and z = L the potential on the end faces is zero. And the cylindrical surface is made two
equal half-cylinders, one at potential V and the other at potential V , so that
V (, z) =
_
V for /2 < < /2
V for /2 < < 3/2
_
(1)
Find the potential inside the cylinder.
2. An electric dipole with constant electric dipole moment magnitude is located at a point in the xy
plane and rotates with constant angular frequency
a) Determine the time dependent electromagnetic elds at large distances from the dipole.
b) Determine the radiated average power angular distribution and the total radiated power.
3. a) Show that the covariant form of inhomogeneous Maxwell equations could be written in terms
of eld strength tensor and 4-current in the following:

u
F
u
=
4
c
J

(2)
b) Show that the covariant form of homogeneous Maxwell equations can be written in terms of
dual strength tensor in the following form:

F
u
= 0 (3)
c ODT

U Physics Department. (May 2004) 143


May 2004
METHODS OF MATHEMATICAL PHYSICS
1. Consider the equation of the forced, damped harmonic oscillator,
x(t) + 2 x(t) +
2
0
x(t) = f(t), where
0
> > 0 (1)
and dierentiation with respect to time t is indicated by a dot.[Here units have been chosen such
that mass m = 1, is essentially the coecient of frictional drag,
0
is the square root of the
(positive) spring constant and f(t) is external driving force.]
a) Find the two linearly independent solutions, x
1
(t) and x
2
(t), to the homogeneous equation (the
equation when f(t) = 0).
b) When f(t) ,= 0 but f(t) 0 as [t[ ,= , show that the general solution can be written in the
form
x(t) = Ax
1
(t) + Bx
2
(t) +
_

G(t, t

)f(t

)dt

, (2)
where constants A and B must be chosen to satisfy the boundary conditions.
c) Calculate the Greens function G(t, t

) explicitly.
d) Let x(0) = 0, x = 0, f(t) = f
0
e
t
for some constants f
0
, > 0 and nd x(t) then.
2. The Bessel functions of the rst kind of integral order n are obtained from a generating function
as
g(x, t) = e
x
2
(t
1
t
)
=

n=
J
n
(x)t
n
(3)
and they satisfy
J
n
(x) = (1)
n
J
n
(x) fornZ. (4)
a) Show that
cos(xsin) = J
0
(x) + 2

n=1
J
2n
(x)cos(2n), (5)
sin(xsin) = 2

n=1
J
2n1
(x)sin[(2n 1)]. (6)
b) Using the identities in part a, show also that
J
n
(x) =
1

_

0
cos(n xsin)d, n = 0, 1, 2, ..... (7)
c) Prove that
J
0
(x) =
1
2
_
2
0
e
ixsin
d =
1
2
_
2
0
e
ixcos
d, n = 0, 1, 2, ..... (8)
d) The Fourier transform of a function f(x) is given by
Ff(x) =
1

2
_

f(x)e
ix
dx. (9)
For [ x [< 1, nd the Fourier transform F(J
0
(x)) .
c ODT

U Physics Department. (May 2004) 144


3. a) Establish the following counter integral representation for the Hermite polynomials:
H
n
(x) =
(1)
n
n!e
x
2
2i
_
c
e
z
2
dz
(z x)
n+1
(10)
Do not forget to show your contour.
Hint: Rodriguez formula for the Hermite polinomials is given as
H
n
(x) = (1)
n
e
x
2 d
n
(e
x
2
)
dx
n
(11)
b) Evaluate this integral to obtain a general expression for the Hermite polinomials.
c ODT

U Physics Department. (May 2004) 145


May 2004
QUANTUM MECHANICS
1. a) A certain state [ > is an eigenstate of L
2
and L
z
. L
2
[ >=
2
( +1)[ >, L
z
[ >= m[ >.
For this state calculate < L
x
> and < L
2
x
>.
b) Let x(t) be the coordinate operator for a free particle in 1D in the Heisenberg picture. Evaluate
[x(t), x(0)].
c) Consider the observables momentum p and position x for a free particle.
i) Write the Heisenberg equations of motion for this system. Obtain the solution for the
position.
ii) Write the commutation relations for position at equal times and dierent times,
iii) Apply the uncertainty relation and obtain < (x
i
)
2
>
t
< (x
i
)
2
>
t=0
, interpret the result.
2. a) For a spin
1
2
system nd the eigenvalues of the operator

S .

n where

S =

2

, and

n is the unit
vector expressed in terms of the spherical polar coordinates:

n = (sincos, sinsin, cos).
b) Construct the eigenstates [

n ; > of (

S .

n ) corresponding to these eigenvalues.
c) Deduce the eigenstates of the operators S
x
, S
y
, S
z
from the general result you have obtained
in part a.
d) Construct the uncertainties (S
x
)
2
, (S
y
)
2
, (S
z
)
2
in the eigenstate [

n ; t >.
e) Verify the Heisenberg uncertanity relation
(A)
2
(B)
2

1
4
[ < [A, B] > [
2
(1)
for all possible pairs (A,B) with A, B being any one of the operators S
x
, S
y
, S
z
.
3. Consider a Dirac particle subject to a three-dimensional spherical well potential
V (r) =
_
V
0
for r < r
0
0 for r > r
0
_
(2)
a) Obtain the exact four-component energy eigenfunctions for j =
1
2
(even) bound states, where
(even) means even orbital parity for the upper two components.
b) Set up an equation that determines the energy eigenvalues.
c) What happens if the strength of the potential is increased so that V
0
becomes comparable to
or larger than 2mc
2
?
Note:
< m

1
2
m
s
[jm > =

+ m +
1
2
2 + 1
for j = +
1
2
, m
s
=
1
2
=

m +
1
2
2 + 1
for j = +
1
2
, m
s
=
1
2
=

m +
1
2
2 + 1
for j =
1
2
, m
s
=
1
2
=

+ m +
1
2
2 + 1
for j =
1
2
, m
s
=
1
2
(3)
c ODT

U Physics Department. (May 2004) 146


May 2004
STATISTICAL MECHANICS
1. A system has energy levels E
n
= nE
0
where n = 1, 2, 3...... For this system in equilibrium nd
a) the Helmholtz free energy A,
b) the average Energy E,
c) the entropy S. Express all three quantities as function of temperature T
d) Repeat parts (a), (b), (c) for a system where n can only have the values 0 and 1.
2. A macroscopic system, consisting of N 1 independent components, is in thermal equilibrium
with a reservoir at temperature T. Each component with 3 internal states is described by a single-
particle Hamiltonian of the form
H =
_
_
E1 V 0
V E1 V
0 0 E1
_
_
(1)
a) Let the canonical single-particle density matrix be = exp(H). Using , construct the
single-particle partition function (Z
1
).
Hint: Z
1
= Trace()
b) Construct the single-particle partition function using another method of your choice.
3. a) Evaluate the root-mean-square uctuation of the magnetization of a system of N independent
spin
1
2
atoms in equilibrium at temperature T within an external magnetic eld B. Express
your results in terms of hyperbolic functions.
b) Use the grand canonical ensemble to show that the variance in particle density =
N
V
, assuming
xed (V ) but variable particle number (N), can be expressed as
2

= (V )
1

[
T,V
. Note that

=<
2
> < >
2
.
c ODT

U Physics Department. (May 2004) 147


Dec 2003
Dec 2003
ANALYTICAL MECHANICS
1. Let a single particle with mass m = 1 move under the Hamiltonian H =
p
2
2
.
a) For this case solve the Hamilton-Jacobi equation for the generating function S (q, , t)
b) Find the canonical transformation q = q (, ) and p = p (, ) where and are the trans-
formed momentum and coordinate respectively.
c) Find the form of trajectory.
d) If there is a perturbation Hamiltonian, H

=
1
2
q
2
then, will no longer be constant. Express
the transformed Hamiltonian K (using the same transformation found in part (a)) in terms of
and and t.
e) Find corresponding Hamilton equations for the transformed Hamiltonian K.
f) Solve Hamilton equations in part (e) for (t) and (t).
g) Show that the perturbed solutions q [ (t) , (t)], p [ (t) , (t)] is simple harmonic.
Hint: In calculation you may need the integrals,
_
dx
x
2
+ 1
= arctan x,
_
tan xdx = ln (cos x)
2. A particle of mass m moves in one dimension q in a potential energy eld V (q) and is retarded by
a damping force 2m q proportional to its velocity. If Lagrangian of this system is given by
L = e
2t
_
1
2
m q
2
V (q)
_
a) Find the Hamiltonian.
b) For the generating function
F
2
(q, P, t) = e
t
qP
nd the transformed Hamiltonian K (q, P, t).
c) For an oscillator potential V =
1
2
m
2
q
2
show that the transformed Hamiltonian yields to a
constant of the motion
K (Q, P, t) =
P
2
2m
+
1
2
m
2
Q + QP.
d) Obtain the solution q (t) for the damped oscillator from the constant of motion in part (c) in
the damped case < .
3. Consider a particle of mass m moving in a plane.
a) Find the central force which results for a particle in the following orbit
r = a (1 + cos )
b) Find the Lagrangian of this particle in terms of the plane polar coordinates r and and their
time derivatives.
c) Write down the equations of motion for r and and show that the angular momentum is
constant of motion.
c ODT

U Physics Department. (Dec 2003) 149


d) If the attractive potential is given by U =

r
2
, nd the eective potential and the r dependence
of U
e
(r).
e) Using the result obtained in part (c) analyse the stability of the orbit.
Hint: the stability of the circular orbit is determined by,
_
F

(r)
F (r)
+
3
r
_
r=
> 0 .
c ODT

U Physics Department. (Dec 2003) 150


Dec 2003
ELECTROMAGNETIC THEORY
1. A point charge q is brought to a position a distance d away from an innite plane conductor held
at zero potential. Using the method of images, nd:
a) the surface charge density induced on the plane.
b) the force between the plane and the charge by using Coulombs law for the force between the
charge and its image
c) Calculate the period of the oscillation.the total force acting on the plane by integrating
2
/
over the hole plane
d) the work necessary to remove the charge q from its position to innity
2. A plane polarized electromagnetic wave of frequency w in free space is incident normally on the
at surface of a nonpermeable medium of conductivity and dielectric constant .
a) Write the boundary conditions on the at surface.
b) Calculate the amplitude and phase of the reected wave relative to the incident wave for arbi-
trary and .
c) Find the reection coecient (ratio of reected to incident intensity) for very poor and very
good conductor.
3. A charge e moves relativistic speeds in simple harmonic motion along the z axis, z (t

) = a cos (t

).
Find the instantaneous power radiated per unit solid angle.
c ODT

U Physics Department. (Dec 2003) 151


Dec 2003
METHODS OF MATHEMATICAL PHYSICS
1. Solve the following partial dierential equation using the Fourier transform method:
u(x,t)
t
= D

2
u(x,t)
x
2
subject to the boundary conditions
u(x,0)
x
0 as x
and the initial conditions
u(x, t) = exp(x
2
), x (, ).
2. Let
I =
_
c
[dz[
[z a[
2
(1)
where c is the circle c = z[ [z[ = , > 0, ' traversed in the counterclockwise direction and
a is any complex number such that [a[ , = .
a) Use [z[
2
= z z =
2
to convert the integral I to a line integral of rational function.
b) Using your answer to part (a), compute the integral I for all possible a such that [a[ , = .
3. The Hermite polynomials H
n
(x) satisfy the ordinary dierential equation
y - 2xy + 2ny = 0
and obey the recurrence relation
H

n
(x) = 2nH
n1
(x).
Assume that there exists a generating function g(x, t) =

n=0
H
n
(x)
t
n
n!
for these polynomials.
a) Dierentiate g(x, t) with respect to x and obtain a rst order partial dierential equation for
g(x, t).
b) Integrate the rst order partial dierential equation you found in part (a) with respect to x,
holding t xed.
c) Given that H
2n
(0) = (1)
n (2n)!
n!
and H
2n+1
(0) = 0, determine the constant of integration in
part (b) and explicitly derive g(x, t).
d) Dierentiate g(x, t) with respect to t and obtain a second recurrence relation satised by H
n
(x).
c ODT

U Physics Department. (Dec 2003) 152


Dec 2003
QUANTUM MECHANICS
1. The hyperne interaction in atoms, which couples the spins of the electrons and the nucleus, is
an extremely weak interaction. It is magnetostatic in origin: The magnetic eld created by one
particle due to its spin magnetic moment changes the spin orientation of the other particle. For the
Hydrogen atom in its ground state the interaction Hamiltonian can be written as H
hf
=
a

S
p

S
e
,
where a is a constant and

S
p
and

S
e
are the spin operators for the proton and the electron
respectively. If there are no other interactions aecting the spins of the particles, we can ignore
the rest of the Hamiltonian. We also ignore the orbital wavefunctions of the particles and consider
only the spin states. Both the proton and the electron has spin 1/2.
a) Show that the total spin operator,

S =

S
p
+

S
e
, is a conserved quantity for this Hamiltonian.
b) Express the common eigenstates of S
2
and S
z
in terms of the uncoupled states, [) , [) , [) , [)
where the rst arrow is for the proton and the second one is for the electron.
c) Find the eigenstates and the eigenvalues of H
hf
. What is the degeneracy of each energy level?
Calculate the angular frequency, , of the radiation that would come out due to transitions
between these levels.
d) Suppose that a proton has just captured an electron and at the time of the capture (t = 0),
the electron had spin-down and the proton had spin-up. In other words, the spin-state of the
system is [(t = 0)) = [). Calculate the spin state, [(t)), of the proton-electron system at a
later time t.
e) What is the probability that the electron is in a spin-up state at time t?
f) For the ground state of the hydrogen atom a is 9.42 10
25
J. Calculate roughly the time for
the complete reversal of the spin of the electron. (h = 6.6 10
34
J s.)
2. a) Starting with the relativistic energy-momentum relation E
2
= c
2
p
2
+m
2
c
4
derive the relativistic
Schrodinger equation. Obtain the probability density and the probability current. Can this
equation be considered as a satisfactory single-particle equation?
b) Derive the Dirac equation. Obtain the probability current and the probability density. Is this
equation a satisfactory single-particle equation?
c) What are the eigenvalues of the velocity operator in Dirac theory? Explain your result in view
of the uncertainty relation.
d) Show that the Dirac equation predicts the correct magnetic moment of the electron.
3. It is well known that for a particle of charge q in an external magnetic eld

B, the Hamiltonian is
given by
H =
1
2m
_
p
q
c

A
_
2
a) Derive the expression of the probability current density

J which satises the continuity equation

t
[[
2
+

J = 0
b) Consider a uniform magnetic eld along the z axis (

B = Bz). Verify that the magnetic potential

A dened by


A =

B can be chosen as

A =
1
2

Br and further satises



A = 0.
c ODT

U Physics Department. (Dec 2003) 153


c) Evaluate the commutator [
x
,
y
] for the kinetic velocities which are dened by
i
=
1
m
_
p
i

q
c
A
i
_
.
Using this result, show that Q =
c
qB
_
p
x

q
c
A
x
_
and P = p
y

q
c
A
y
can be chosen as a suitable
canonically conjugate pair.
d) Express the Hamiltonian in terms of the two conjugate pairs (Q, P) and (z, p
z
). Comparing
this expression with that of the simple harmonic oscillator, write down the energy eigenvalues
of the problem under consideration automatically [Recall: H =
p
2
2m
+
1
2
m
2
x
2
with [x, p] = i
and E
n
=
_
n +
1
2
_
], without solving the problem explicitly.
c ODT

U Physics Department. (Dec 2003) 154


Dec 2003
STATISTICAL MECHANICS
1. Using the grand canonical ensemble, evaluate the chemical potential (T, P) for an ultrarelativistic
gas ( = c[

P[ in a box of volume V).


2. Find the approximate expression for the partition function Z(T, L) for a single quantum particle
in an innite one-dimensional square well of width L in the limit of low temperatures. Obtain the
heat capacity C
L
in this limit.
Hint:
E(n) =
n
2

2
2mL
2
, n = 1,2,3,..
3. Consider a system of very weakly interacting identical molecules embedded in a crystal lattice.
Suppose that each molecule can be modeled as a rigid zipper with N links. Each link has two
states: open and closed, with respective energies E and 0. The zipper can only unzip from the left
end (i.e., the right end is always closed), and a given link can only open if all links to the left are
already open. The system is in equilibrium at temperature T.
a) Show that the canonical partition function (Z) of each molecule can be summed in the form
Z = [1 exp (NE)] / [1 exp (E)]
where = (k
B
T
1
and k
B
is the Boltzmann constant.
b) Find the average number of open links in the system.
c) Discuss the limiting values of the average number of open links in extreme (low and high)
temperatures. Clarify what you meant by high and low temperature limits.
4. An ideal gas of free fermions is enclosed in a 3-dimensional volume V. The fermions have momentum
(p)-energy () relationship p=A
2/3
where A is a constant and the energy of any fermion is
restricted between the values 0 and
m
. These fermions possess the property (as do photons and
phonons) that their number is not conserved. Calculate the average energy of the gas in extreme
(low and high) temperature limits, paying particular attention to its temperature dependence (i.e.,
do not be concerned with the values of the temperature-independent constants). Clarify what you
meant by high and low temperature limits.
c ODT

U Physics Department. (Dec 2003) 155


May 2003
May 2003
ANALYTICAL MECHANICS
1. A double pendulum consists of two identical rods hinged at their one ends to each other is mounted
from the ceiling as shown in the gure. The rods have m and length and constant mass density
(mass per unit length). The system is placed in a gravitational eld (acceleration g). The pendulum
is constrained to move in a vertical plane. Assume that
2
1
1 and
2
2
1. Find:
a) The Langrangian of the system in terms of
1
and
2
, and their time derivatives.
b) Find the normal mode frequencies of the system.
c) Find the Hamiltonian of the system.
2. The transformation equation between two sets of coordinates are
Q = ln(1 + q
1/2
cos p), P = 2(1 + q
1/2
cos p)q
1/2
sin p.
a) Show that this transformation is canonical.
b) Find the symmety function F
3
(p, Q) of this transformation.
3. Assume we have two types of potentials;
a) Powerlaw type: U(r) =
k
n 1
1
r
n1
,
b) Yukawa type: U(r) =
k
r
e
r/a
.
Investigate the stability of circular orbit in both cases.
Hint: The condition for stability is
_
F

(r)
F(r)
+
3
r
_
r=r
0
> 0,
where F(r) is the force, and F

is its rst derivative.


c ODT

U Physics Department. (May 2003) 157


May 2003
ELECTROMAGNETIC THEORY
1. Consider a possible solution to Maxweels equation given by
A(x, t) = A
0
e
i(k.xt)
,
(x, t) = 0,
where A is the vector potential and is the scalar potential. A
0
, k, and are the constant vector
potential, the constant wave vector, and the constant angular frequency, respectively.
a) Find the electromagnetic eld components E and B in terms of given quantities above.
b) Write the Maxwell equations in the source free region (i.e., the current density J = 0 and the
charge density = 0 ).
c) Substitute the electromagnetic eld components found in part (a) into the Maxwell equations
in part (b) and nd constraints for E, B, and k.
2. Two point charges q and q are located on the z axis at z = a and z = a, respectively.
a) Find the multiple moments
b) Suppose now that the charges are surrounded by a spherical shell of radius b (b > a) and kept
at constant potential V . Using the Greens function nd the potential everywhere inside the
spherical shell.
3. An electron moves in a circular orbit in a synchrotron under the action of homogeneous magnetic
eld B
0
. Calculate the energy, which is lost in electromagnetic radiation per revolution if the speed
v of the electron is assumed to be constant.
Hint:
d(m
0
v)
dt

= F,
dU
dt

=

0
q
2
v
2

4
6c
.
c ODT

U Physics Department. (May 2003) 158


May 2003
METHODS OF MATHEMATICAL PHYSICS
1. Solve the following dierential equation using the Fourier Transform method:
u(x, t)
t
= D

2
u(x, t)
x
2
,
subject to the boundary conditions,
u(x, 0)
x
0, x ,
and the initial conditions,
u(x, 0) = e
x
2
, x (, ).
2. a) Given the generating function g(x, t) = e
x/2(t1/t)
expand it in a Laurent series about t = 0 as
g(x, t) =

n=
J
n
(x)t
n
,
and obtain the series expansion of J
n
(x). Specify the exact range of t over which the Laurent
expansion of g(x, t) holds (for x nite).
b) Show that J
n
(x) is not independent of J
n
(x) by nding the relation that describes one in
terms of the other.
c) Dierentiate g(x, t) with respect to x and t, respectively, and obtain two separate recurrence
relations that relates J
n1
(x) and J
n+1
(x) to J

n
(x) and J
n
(x), respectively. (Prime denotes
dierentiation with respect to x here).
d) Prove that J
n
(x) can be represented as
J
n
(x) =
1
2i
_
C
g(x, t)
dt
t
n+1
by a suitable closed contour. Describe the most general contour that can be used.
3. Evaluate
_

0
tan(x + ia) dx, a ', a ,= 0,
by turning it into a contour integral around a unit circle. Discuss separately the case a = 0.
c ODT

U Physics Department. (May 2003) 159


May 2003
QUANTUM MECHANICS
1. Consider a particle on a onedimensional lattice;
a) What are the eigenstates and eigenvalues of the translation operator T, dened by T n) = n + 1)
?
b) Consider the Hamiltonian
Hn) =
1
2
2
n + 1) +n 1) 2 n) .
Does T commute with H?
c) Let k) be given by
n[k) = e
ikn
.
Show that k) is an eigenket of Hamiltonian. What is the relationship between the eigenvalue
E
k
and k) ? What is E
k
as k 0 ?
2. Consider a particle which for t < 0 evolves in onedimensional potential so the Hamiltonian is
given by
H =
p
2
2m
+
1
2
kx
2
.
The particle is in the ground state of this Hamiltonian. For t > 0 a small perturbation is introduced.
So, the full Hamiltonian becomes H = H
0
+ H
i
(t), where H
i
(t) = x
2
e
t
.
a) Construct creation and annihilation operators a

and a, as linear combination of p and x with


the property that
_
a, a

= 1,
and nd expressions of H
0
and H
i
(t) in terms of operators, a

and a.
b) To the lowest nonvanishing order in , nd the probability that at a later time (t 1/), the
particle will be found in the 2
nd
excited states of H
0
.
c) To the lowest order in , extend the calculation in (b) to nd the probability that the particle
will be found in the n
th
excited state of H
0
for all values of n.
3. Consider the mutual eigenstate of J
2
and J
z
represented by j, m), where J is an angular momentum
operator. Calculate the following expectation values in this state
a) J
x
), J
y
)
b) J
x
, J
y
.
Let J
n
= n.J denote the component of the angular momentum along the unit vector n =
sin x + cos z.
c) Calculate J
n
) and J
n
.
d) What is the uncertainity relation between the observables J
n
and J
y
? Show that this relation
is satised in this state.
Hint:
J

j, m) =
_
j(j + 1) m(m1)j, m1).
c ODT

U Physics Department. (May 2003) 160


May 2003
STATISTICAL MECHANICS
1. Suppose that the energy levels of a particle is given by E
n
= n(E
0
/2) and the system consistes of
Nparticles. Calculate
a) The partition function,
b) The Helmholtz free energy,
c) Energy of the system,
d) The ppecic heat C
v
,
e) Describe the high and low temperature limits of C
v
.
2. Consider an ideal Bose gas of N particles with singleparticle energy spectrum k
s
(k is the
wave vector) conned to a box of volume V in d spatial dimensions.
a) Under what conditions (a relationship between s and d) does one expect BoseEinstein conden-
sation (BEC)?
b) Does BEC occur for the extremely relativistic gas in d = 2?
c) Show that if BEC does occur, the critical temperature (T
c
) is proportional to a power density,
such that T
c
(N/V )

and deduce in terms of s and d.


d) Show that if BEC does occur then, C
v
(T/T
c
)

for T < T
c
, and deduce in terms of s and
d. (C
v
is the specic heat at constant volume).
Hint:
x
1
dx
z
1
e
x
1
= g

(z)(), (0 z < 1, > 0; z = 1, > 1).


3. You need a calculator for the following question!
Heat capacity at constant volume (C
v
) of three dierent onedimensional solids (A, B, and C),
each consisting of N 1 noninteracting atoms, are examined as a function of temperature (T).
The experimental results are given in the plot below. Suppose, the atoms in each material have
only two energy levels: a ground state at 0 meV, and a doubly degenerate excited level at
i
meV,
with i = A, B, C.
a) Determine the values of
A
,
B
, and
C
using the plot below.
b) Find S
A
(Nk
B
)
1
(S is the entropy) at 10 K.
c ODT

U Physics Department. (May 2003) 161


Nov 2002
Nov 2002
ANALYTICAL MECHANICS
1. Let us consider a pendulum of mass m whose point of support A oscillates in plane (x, y), as in
the gure, according to law
r = e sin(2t + ),
e = (e
x
, e
y
) = (cos , sin ), (1)
whera , , are some xed parameters.
a) Find the simplest equivalent form of corresponding Lagrangian.
b) Find an expression for the angular expression for the angular momentum of such a pendulum.
c) At which conditions the innite motion is possible?
d) Find the corresponding Hamiltonian and Hamiltonians equations.
e) Is energy conserved?
2. A particle with mass m and energy E is moving in a onedimensional potential
V (x) = ax
2
+ bx + c,
where a, b and c are some real parameters.
a) At which values of parameters a, b, c and E is the innite motion is possible?
b) Find the turning points.
c) What is dimensionality of parameters a, b and c?
d) Find the period of oscillation T for case a > 0, b = c = 0.
e) Explain the dependence of T on parameters a and E.
3. A particle with mass m, energy E and angular momentum M is moving in a central potential
V (r) =

r
n
,
where and n are some real parameters.
a) How many integration constatns must the general solution of the corresponding equation of
motion contain?
b) Explain its physical meaning.
c) Find the generalization of the third Kepler law (relation between typical time and space scales).
d) What are the typical combinations of parameters of this problem could be used for time mea-
suring in a such a potential?
c ODT

U Physics Department. (Nov 2002) 163


Nov 2002
ELECTROMAGNETIC THEORY
1. Derive the continuity equation for charge density from Maxwells equations using
a) vector notation,
b) component notation.
Compare the usefulness of the two systems of notations. Also, discuss the physical meaning of the
charge continuity equation.
2. If a progressive wave is traveling in a linear, isotropic, homogeneous, nonconducting dielectric
medium with dielectric parameter and permeability , what is the dispersion relation (D(, k) =
0)? And what is the group velocity in this case? Also, what is the dispersion relation in a conducting
medium?
3. An electric dipole with constant electric dipole moment magnitude is located at a point in the xy
plane and rotates with constant angular frequency .
a) Determine the time dependent electromagnetic elds at large distances from the dipole.
b) Determine the radiated average power angular distribution and the total radiated power.
c ODT

U Physics Department. (Nov 2002) 164


Nov 2002
METHODS OF MATHEMATICAL PHYSICS
1. In eld theory one often encounters divergent sums like

n=1
n
2
.
To obtain meaningful quantities we modify such expressions by introducing a regularization function
and a parameter such as;

n=1
n
2
e
n
, (1)
where e
n
is a regularization function and is a regularization parameter. This way we hope
to control the divergences in the original series and hope to get a nite quantity after a suitable
subtraction process. In the light of these evaluate the above sum (3)
a) by using the EulerMclaurin sum formula
b) by using the geometric series,
c) show that the two results are identical.
Hint:
B
0
= 1, B
1
=
1
2
, B
2
=
1
6
, B
3
= 0, B
1
=
1
30
2. Evaluate
_
2
0
sin
2

5 + 3 cos
d
3. Consider a set of functions
g(x, t) =

n=1
f
n
(x)
t
n
n!
,
that satises the dierential equation
(A + Bt)
g(x, t)
t
= (Cx + Dt)g(x, t)
a) Show that f
n
(x) satises the recursion relation
Af
n+1
(x) = (Cx Bn)f
n
(x) + Dnf
n1
(x).
b) Expanding g(x, t) in a power series about t = 0, show that
f
0
(x) = g(x, 0).
c) Using the recursion relation, show that
f
1
(x) =
C
A
xg(x, 0),
f
2
(x) =
_
C
2
A
2
x
2

BC
A
2
x +
D
A
_
g(x, 0).
c ODT

U Physics Department. (Nov 2002) 165


Nov 2002
QUANTUM MECHANICS
1. It is known that the Yukawa potential
V (r) = V
0
e
r
r
(1)
is producing in Born approximation the following amplitude
f() =
2mV
0

2
1

2
+ q
2
, (2)
where m is the mass of scattered particle, and q is the momentum transfered.
a) Explain the physical meaning of the factor
mV
0

2
, appearing in Eq. (2).
b) Under which kinematical conditions the corresponding cross-section does not depend on energy
of scattered particle?
c) If
d
d

= 10
26
cm
2
, for proton scattering with zero energy, estimate V
0
in [MeV ] assuming
1/r = 10
13
cm.
2. A box containing a particle is divided into a right and left compartments by a thin partition. If the
particle is known to be on the right (left) side with certainty, the state represented by the position
eigenket [R) ([L)), where we have neglected spatial variations within each half at the box. The
most general state vector can then be written as
[) = [R)R[) +[L)L[),
where R[) and L[) can be regarded as wave functions. The particle can tunnel through the
partition; this tunneling eect is characterized by the Hamiltonian
H = ([L)R) +[R)L[),
where is a real number with the dimension of energy.
a) Find the normalized energy eigenkets and the corresponding energy eigenvalues.
b) In the Schrodinger picture the base kets [R) and [L) are xed and the sate vector moves with
time. Suppose the system is represented by [) as given above at t = 0. Find the state vector
[, t
0
= 0; t) for t > 0 by applying the appropriate time evolution operator to [).
c) Suppose at t = 0 the particle is on the right side with certainty. What is the probability for
observing the particle on the left side as a function of time?
d) Write down the coupled Schrodinger equations for the wave functions
R[, t
0
= 0; t), L[, t
0
= 0; t).
Show that the solutions to the coupled Schrodinger equations are just what you expect from
(b).
e) Suppose the printer made an error and wrote H as
H = ([L)R[).
By explicitly solving the most general timeevolution problem with this Hamiltonian, show that
the probability conservation is violated.
c ODT

U Physics Department. (Nov 2002) 166


3. An electron in a onedimensional potential well
V (x) =
1
2
kx
2
+
1
2
m
2
x
2
is immersed in a constant uniform electric eld of magnitude E which points in the xdirection.
The corresponding perturbation to the Hamiltonian is H

= eEx.
a) Find the exact energy eigenvalues of the total Hamiltonian
H =
p
2
2m
+
1
2
kx
2
+ eEx
without solving the corresponding Schrodinger equation.
b) Calculate the rst and second order corrections to the energy. Compare your results with part
(a).
Hint: Energy eigenvalues of the Hamiltonian for harmonic oscillator are given by
E
n
= (n + 1/2);
creation and annihilation operators are dened as follows
a =
_
m
2
_
x +
i p
m
_
, a

=
_
m
2
_
x
i p
m
_
,
and
a[n) =

n[n 1), a

n + 1[n + 1).
c ODT

U Physics Department. (Nov 2002) 167


Nov 2002
STATISTICAL MECHANICS
1. a gas of N indistinguishable classical noninteracting atoms are held in a neutral atom trap by the
potential V (r) = r. The gas is in thermal equilibrium at temperature T.
a) Find the single particle partition function Z for a trapped atom.
b) Find the entropy of gas.
c) The gas is cooled reversibly by decreasing to

while there is no heat exchange with the


surroundings. Find the nal temperature T

in terms of ,

and T.
2. Suppose that one has a gas of N free spin1/2 fermions of mass m on a planar surface of area A.
Obtain an explicit expression for the chemical potential of this gas as a function of temperature.
3. A twostate system has energy levels of energy 0 and . Find;
a) the canonical partition function,
b) the free energy,
c) the average energy,
d) the entropy
e) the specic heat, as a function of temperature.
f) What are the implications of the T 0 and T limits of entropy for level occupations?
Useful Relations:
B

(T) =
2hc
2

5
1
e
hc/kT
1
,
B

(T) =
2h
c
2

3
e
h/kT
1
2hc
2
= 119.268 10
18
,
hc/kT = 78412.345 10
10
,
hc/kT = 6.955935558 10
6
,
hc/kT = 69559.356 10
10
.
c ODT

U Physics Department. (Nov 2002) 168


May 2002
May 2002
ANALYTICAL MECHANICS
1. Let us consider onedimensional motion of particle with mass m and energy E in the potential
V (x) = x
2
+ ax + b,
where a and b are some real parameters.
a) For which values of a, band E the nite motion is possible?
b) Determine the turning points.
c) Calculate the period of the oscillation.
2. Lagrangian of the system has the following form
L = m q
2
+ aq
2
sin 2t 2aq q(cos t)
2
,
where m, a and are some real parameters.
a) Is it possible to simplify the Lagrangian?
b) Find the Hamiltonian of the system.
c) Find the Hamiltonians equations.
3. Consider the motion of a projectile of mass m in a vertical plane (xy plane). Assume that
x = y = 0 and x = y = v
0
at t = 0.
a) Write down the Lagrangian.
b) Find both the dependence of the coordinates on time and the equation of trajectory.
c) Solve the part (b) again by HamiltonJacobi method.
c ODT

U Physics Department. (May 2002) 170


May 2002
ELECTROMAGNETIC THEORY
1. a) Derive the canonical momentum and the generalized force for the case of the charged particle
in electromagnetic eld and A. The Langrangian is
L =
mc
2

q +v.A.
b) Consider the Langrangian density of the electromagnetic eld in the form
L =
1
2

0
_
c
2
B
2
E
2
_
A.J + .
We know that electromagnetic elds are invariant under gauge transformation
A = A

+, =


t
.
Determine the Langrangian density under a gauge transformation. Is it invariant? If not,
discuss the consequences this would have.
2. The surface of a hollow conducting sphere of inner radius a is divided into an even number of equal
segments by set of planes their common line of intersection is the zaxis and they are distributed
uniformly in the angle . The segments are kept at the potentials V alternatively. Set up a series
representation for the potential inside the sphere for the general case 2n segments.
3. Consider the continuity equation and introduce the vector eld
.(t, x) =
(t, x)
t
= J
and the super potential (t, x) with the following properties
.(t, x) =
1
c
2
(t, x)
t
= A.
a) Show that (t, x) satises the homogeneous wave equation.
b) If a help vector C is dened as C = . Show that the magnetic and electric eld can be
obtained as follows
B =
1
c
2
C
t
, E = C.
c ODT

U Physics Department. (May 2002) 171


May 2002
METHODS OF MATHEMATICAL PHYSICS
1. The generating function for the Bessel coecients
g(z, t) =

n=
J
n
(z)t
n
is given as
g(z, t) = exp
_
z
2
_
t
1
t
__
using this, obtain the recursion relations
2n
z
J
n
(z) = J
n1
(z) + J
n+1
(z)
2J

n
(z) = J
n1
(z) J
n+1
(z).
2. Evaluate the integral
_
2
0
sin
2

3 + 2 cos
d.
3. Show that the Greens function for the dierential operator L =
d
2
dx
2
with
y(0) = y(1) = 0
could be written as
a)
G(x, t) =
_
x(1 t), 0 x < t
t(1 x), t x < 1
_
,
or
b)
G(x, t) =
2

n=1
sin(nx) sin(nt)
n
2
c) Show that they are equivalent.
c ODT

U Physics Department. (May 2002) 172


May 2002
QUANTUM MECHANICS
1. Answer each of the following questions with a brief and where possible quantitative statement
a) A beam of natural atoms pass through a Stern-Gerlach apparatus. Seven equally spaced lines
are observed. What is the total angular momentum of the atom?
b) What is magnetic moment of an atom in the state
3
P
0
? (Disregard nuclear eects.)
c) In a hydrogen gas discharge both the spectral lines corresponding to the transition 2
2
P
1/2
1
2
S
1/2
and 2
2
P
3/2
1
1
S
1/2
are observed. Estimate the ratios of their intensities.
d) What is the cause for the existence of two independent term level schemes, the singlet and the
triplet systems in atomic helium.
Hint: The representation of the atom in terms of J, L and S is
2S+1
L
J
.
2. Let Hamiltonian of a particle with spin S in constant electric (E) and magnetic (B) elds has the
following form
H = a
1
S.L + a
2
S.p + a
3
S.E+ a
4
S.B+ a
5
L.E+ a
6
L.B,
where a
i
, i = 1, 2, 3, 4, 5, 6 are constants. Which terms of this Hamiltonian violates,
a) P invariance?
b) T invariance?
c) PT invariance?
3. An electron is scattered elastically by a
3
He nucleus which contains one proton. Consider electro-
static interaction of the electron only. Derive an expression for the dierential crosssection; show
your steps clearly and give physical interpretation of the factors in your expression.
c ODT

U Physics Department. (May 2002) 173


May 2002
STATISTICAL MECHANICS
1. Calculate the isothermal compressibility
_

1
= V
_

2
F
V
2
_
T
_
of a one-dimensional classical ideal
gas of N indistinguishable particles, each of mass m , conned to a line of length L, using the
grand canonical partition function.
2. Let the density of states of the electrons in some material be assumed constant= c, and the
total number of electrons be equal to N. Calculate the chemical potential as a function of
temperature T when the system is nondegenerate (i.e., when exp

1).
Hint:
P =
2
1/2
m
3/2
(kT)
5/2
3
2

3
_
x
3/2
dx
exp[x ] 1
.
3. If T T
c
at xed density, quantum eects can be neglected and the thermal properties of an ideal
Bose gas reduces to the ideal classical gas. What is the rst quantum correction to the classical
equation of state
PV
NkT
= 1, when the temperature is reduced (at constant density)?
c ODT

U Physics Department. (May 2002) 174


Nov 2001
Nov 2001
ANALYTICAL MECHANICS
1. A particle with mass m and energy E is moving in the potential V (x), which is presented on the
following gure a and b are some parameters, a, b > 0.
a) For which values of energy E the oscillations are possible?
b) Find the turning points.
c) Find the formula for the period T of the oscillation.
d) Analyze the answer from the dimensional consideration.
e) Explain the dependence of T on the parameters a, b and on the mass m.
2. Let us change slightly the Coulomb potential
V (r) =

r
=

r
1+
where is a small parameter. Let us consider the motion of particle with mass m in such a
potential.
a) Is the energy conserved now?
b) Is the angular momentum conserved?
c) Is elliptic momentum possible now?
d) How the third Keplers law must be modied now?
e) Is a circle orbit possible?
3. In one freedom, consider the transformation
Q =
_
mq + ip

2m
_
e
it
, P = i
_
mq ip

2m
_
e
it
, (i =

1).
a) Show directly that this transformation is canonical.
b) Find a generating function of type F
1
(q, Q, t).
c) Consider the generating function F
1
(q, Q, t) for the harmonic oscillator problem with the Hamil-
tonian
H =
1
2m
p
2
+
1
2
m
2
q
2
.
Find the new Hamiltonian K(Q, P, t) and solve Hamiltonians equations of motion.
d) Determine the time evolution of the variables (q, p).
c ODT

U Physics Department. (Nov 2001) 176


Nov 2001
ELECTROMAGNETIC THEORY
1. Derive the Larmor formula by calculating the radiated power of an accelerating point charge due to
electric dipole emission. Apply the Larmor formula to linear harmonic motion and circular motion.
Hint: The time domain (nonFourier) version of the dipole elds:
E =
1
4
0
rc
2
( p r) r, B =

0
4rc
( p r)
Linear harmonic motion:
x(t) = x
0
cos(
0
t),
Circular harmonic motion:
x(t) = R
0
cos(
0
t), y(t) = R
0
sin(
0
t).
2. a) State gauge transformation and discuss their signicance.
b) Show that E and B are invariant under gauge transformations.
c) Give two examples of choice of a gauge (or gauge condition).
d) If initially chosen potentials do not satisfy the Lorentz condition, nd a gauge transformation
such that the transformed potentials satisfy the Lorentz condition.
3. Consider a hollow grounded sphere of radius b with a concentric ring of charge of radius c and total
charge Q. The charge density of the ring can be written as
(x

) =
Q
2a
2
(r

c)(cos

)
Find the electrostatic potential inside the spherical shell.
Hint: The Green function for a spherical shell bounded by r = a and r = b is given as
G(x, x

) = 4

l=0
l

m=l
Y

lm
(

) Y
lm
(, )
(2l + 1) [1 (a/b)
2l+1
]
_
r
l
<

a
2l+1
r
l+1
<
__
1
r
l+1
>

r
l
>
b
2l+1
_
with
Y
lm
(, ) =

2l + 1
4
(l m)!
(l + m)!
P
m
l
(cos )e
im
.
c ODT

U Physics Department. (Nov 2001) 177


Nov 2001
METHODS OF MATHEMATICAL PHYSICS
1. Assuming that f(z) =
g(z)
(z z
0
)
m
where g(z) is analytic at and around z
0
,
a) Show that
Res[f(z)]
z=z
0
=
1
(m1)!
lim
zz
0
d
m1
dz
m1
[g(z)]
b) Show that
_

0
dx
(x
2
+ 1)(x
2
+ 4)
2
=
5
288
2. Find the closed curve with the least perimeter that encloses a given area A.
Hint: Area inside a closed curve in the xyplane is
A =
1
2
_
b
a
(xy

y) dt
3. The psifunction is dened in terms of the gamma function as
(z) =
d
dz
[ln (z)]
The nth derivative
(n)
(z) of (z) is then

(n)
(z) =
d
n
dz
n
(z) =
d
n+1
dz
n+1
[ln (z)]
a) Show, using (z + 1) = z(z) that

(n)
(z + 1) =
(n)
(z) +
(1)
n
n!
z
n+1
b) Then show that

(n)
(k + 1) = (1)
n
n!
_
1
k
n+1
+
1
(k 1)
n+1
+ . . . +
1
1
_
+
(1)
c) Show, using (
1
2
+ z)(
1
2
z) =

cos(z)
that

(n)
= (
1
2
+ z) (1)
n

(n)
(
1
2
z) =
d
n
dz
n
tan(z).
c ODT

U Physics Department. (Nov 2001) 178


Nov 2001
QUANTUM MECHANICS
1. An electron placed in a uniform magnetic eld B = B

k has S
y
=

2
at t = 0.
a) Show that,
d
dt
< S >=
e
mc
< S > B
b) Find the rst time that electron has again S
y
=

2
.
c) What is the probability to measure S
y
=

2
at time t.
d) Evaluate < S
x
(t) > both in the Schrodinger and Heisenberg pictures.
2. Derive the dierential cross section for the elastic scattering of a fast electron by the ground state
of the hydrogen atom. Ignore the eect of identity.
Hint: Hydrogen ground state wave function is =
_
1
a
0
_
3/2
2e
r/a
Y
00
.
3. Determine the energy levels of a Dirac particle in a homogeneous magnetic eld along the zaxis
B = B

k, where

k is unit vector in zaxis direction. For this aim perform the following calculations:
a) Reduce the Dirac equation for the bispinor =
_

_
to the twocomponent form.
b) Apply the result obtained in part 1 to the stationary state problem for a constant, purely
magnetic eld (A
0
= 0, A) is independent of time and eliminate in equations.
c) Use the ansatz

(x) = e
i(P
y
y + P
z
z)
f(x) in the equation obtained in part b to reduce to the
Schrodinger equation for the harmonic oscillator, and without solving this equation determine
the energy levels.
Hint: Choose vector potential as A = (0, Bx, 0) and use Coulomb gauge A = 0, for Dirac
matrices use the following representation,

0
=
_
I 0
0 I
_
; =
_
0
0
_
.
c ODT

U Physics Department. (Nov 2001) 179


Nov 2001
STATISTICAL MECHANICS
1. Show that when the density of a gas consisting of particles of mass m is suciently low and its
temperature is suciently high so that, the thermal wavelength mean distance between particles,
one can use Boltzmann statistics as a good approximation irrespective of whether particles obey
Fermi or Bose statistics.
2. The molar heat capacity at constant volume of an ideal gas of bosons of mass m at a temperature
below the BoseEinstein condensation temperature T
c
is given by,
C
v
= 1.93 R (T/T
c
), T < T
c
.
Find the pressure for this gas at T < T
c
.
Given: N/V = 2.61(2m kT
c
/h
2
)
3/2
.
3. Consider an ideal Bose gas composed of particles which have internal degrees of freedom. It
is assumed for simplicity that only one internal energy level
1
has to be taken into account.
Determine the BoseEinstein condensation temperature of this gas as a function of energy
1
. (Do
not calculate integrals.)
c ODT

U Physics Department. (Nov 2001) 180


May 2001
May 2001
ANALYTICAL MECHANICS
1. A point mass m slides without friction along a wire bent into a vertical circle of radius a. The wire
rotates with constant angular velocity about the vertical diameter, and the apparatus is placed
in a uniform gravitational eld g parallel to the axis of rotation (see the gure).
a) Construct the Langrangian for the point mass using the angle (angular displacement measured
from the upward vertical) as generalized coordinate and nd the equation of motion.
b) Find the condition for an equilibrium circular orbit.
c) Demonstrate that this equilibrium circular orbit is stable against small displacements along the
wire and nd angular frequency of small oscillations about the equilibrium.
2. In one freedom, consider the transformation
Q =
_
mq + ip

2m
_
, P = i
_
mq ip

2m
_
, (i =

1)
a) Show directly that this transformation is canonical.
b) Find a generating function of type F
1
(q, Q, t).
c) Consider the generating function F
1
(q, Q, t) for the harmonic oscillator problem with the Hamil-
tonian
H =
1
2m
p
2
+
m
2
2
q
2
.
Find the new Hamiltonian K(Q, P, t) and solve Hamiltons equations of motion. Determine the
time evaluations of the variables (q, p).
d) Is there a Lagrangian for the new Hamiltonian K? If so, give it.
3. A particle of mass m moves in a attractive central force eld.
a) Using the Lagrangian formulation, nd the equations of motion in terms of the plane polar
coordinates r and .
b) Use the equations of motion, and the substitution u =
1
r
show that the orbital equation cam be
written in the form
d
2
u
d
2
+ u =
m

2
1
u
2
F(1/u)
where is a constant.
c) If the particle is describing an orbit r = a(1 + cos ), a = constant, under the action of the
central force, nd the force law.
d) What is the total energy of the orbit?
c ODT

U Physics Department. (May 2001) 182


May 2001
ELECTROMAGNETIC THEORY
1. A plane polarized electromagnetic wave is incident on a perfect conductor at an angle . The
electric eld is given by E = E
0
e
i(k.rt)
. E is in the plane of incidence. Starting with the
boundary conditions imposed on electromagnetic eld at a surface of a conductor, derive the
following properties of the reectred wave:
a) direction of propagation,
b) amplitude,
c) polarization,
d) phase.
2. Two electric dipole oscillators vibrate with the same frequency , but their phases dier by /2.
The amplitudes of the dipole moments are both equal to p, but the two vectors are at an angle
0
to each other, (let p
1
be along the xaxis and p
2
in the xyplane). Find
a) the average angular distribution,
b) the average total intensity of the emitted radiation in the radiation zone.
Hint: For an oscillating dipole p at the origine, the Beld in the radiation zone is given by
B = k
2
e
ik.r
r
(
r
r
p).
3. Two large, nonconducting, parallel plates separated by a distance d and oriented as shown in the
gure, move together along the xaxis with velocity v, not necessarily small compared to c. The
upper and lower plates have uniform surface charge densities + and respectively in the rest
frame of the plates.
a) Find the magnitude and direction of the electric and magnetic elds between the plates (neglect
the edge eects).
b) Check that E

.B

= E.B and E

2
B

2
= E
2
B
2
i.e., these eects expressions have thesame
value in both frames.
c) Show that E.B and E
2
B
2
are invariant under Lorentz Transformations in general.
c ODT

U Physics Department. (May 2001) 183


c ODT

U Physics Department. (May 2001) 184


May 2001
METHODS OF MATHEMATICAL PHYSICS
1. Find the extramals of the problem

_
[S(x)y

(x)
2
p(x)y

(x)
2
+ q(x)y(x)
2
]dx = 0
subject to
_
r(x)y(x)
2
dx = 1,
where y(x
1
), y(x
2
), y

(x
1
), y

(x
2
) are prescribed.
2. A function f(z) that has an isolated pole of order m has the Laurent series expansion
f(z) =

n=0
a
n
(z z
0
)
n
+
b
1
z z
0
+ . . . +
b
m
(z z
0
)
m
a) Show that
Re s
z=z
0
[f(z)] =
1
(m1)!
lim
zz
0
d
m1
dz
m1
[(z z
0
)
m
f(z)].
b) Find the residues of
f(z) =
z
(z 1)(z + 1)
2
at the points z = 1 and z = 1.
c) Evaluate the integral
I =
_
c
z dz
(z 1)(z + 1)
2
where C is the eightcontour shown in the gure.
3. Using Schois integral reprentations for Legendre functions
P

(z) =
1
2i
_
c
(t
2
1)

dt
2

(t 1)
+1
where C is a closed contour that encloses t = z and t = 1 but not t = 1.
a) Show that the generating function
g(, t) =

t=0

(z)
is obtained as
g(, t) =
1

1 2z +
2
.
c ODT

U Physics Department. (May 2001) 185


b) Dierentiating g(, t) with respect to , obtain the recurrence relation
z(2 + 1)P

(z) = ( + 1)P
+1
(z) + P
1
(z).
c) Expanding g(, t) obtain P
0
(z) and P
1
(z).
d) Using the recurrence relation compute P
2
(z) and P
3
(z).
c ODT

U Physics Department. (May 2001) 186


May 2001
QUANTUM MECHANICS
1. An electron occupies the ground state of a onedimensional potential
V (x) = 0, L/2 x L/2
V (x) = , elsewhere
A uniform electric eld = x is applied at t = 0 and removed at t = T. Use time dependent
perturbation theory to nd the probability that the particle will be in the rst excited state at
t > T. Useful formulei:
sin Acos B =
1
2
sin(A + B) +
1
2
sin(AB),
_
xsin ax dx =
1
a
2
sin ax
x
a
cos ax.
2. An electron moves in the presence of a uniform magnetic eld in the zdirection (B = B z). The
Hamiltonian of the system is
H =
1
2m
(p
e
c
A)
2
+ e
where and A are scalar and vector potentials.
a) Calculate the commutator [
x
,
y
] where
i
= p
i

e
c
A
i
.
b) Using the result obtained in step in step (1) evaluate
[V
x
, V
y
], [V
y
, V
z
], [V
z
, V
x
],
where V is the velocity of electron.
c) By compairing the Hamiltonian and commutation relation obtained in (1) reduce given problem
to the onedimensional oscillator problem and nd energy eigenvalues without solving corre-
sponding Schrodinger equation.
c ODT

U Physics Department. (May 2001) 187


3. Dirac equation in a scalar eld can be written as
i

t
= [c.p + mc
2
+ V (V)].
Consider the onedimensional potential shown in the gure.
V = V
0
z > 0
V = 0 z < 0
Calculate the reected and transmitted currents for a positive energy electron of momentum p and
energy E < V
0
incident from the left with spinup along the zdirection. Describe and explain
what happens if V
0
is increased such that V
0
> E + mc
2
.
c ODT

U Physics Department. (May 2001) 188


May 2001
STATISTICAL MECHANICS
1. A system has energy levels E
n
= nE
0
, where n = 0, 1, 2, . . .. Find the Helmholtz free energy A,
the average energy E and the entropy S for the system in equilibrium.
2. A surface with N
0
absorption centers has N (N N
0
) gas molecules adsorbed on it. Find the
chemical potential of the adsorbed molecules in terms of the partition function of a single adsorbed
molecule a(T). (You may neglect the intermolecular interactions between the adsorbed molecules).
3. Solve problem 2 using the Grand Canonical approach.
c ODT

U Physics Department. (May 2001) 189

Vous aimerez peut-être aussi